CH3 Current Electricity
CH3 Current Electricity
Cu RRENT ELECTRICITY
R
SI
3.1 CURRENT ELECTRICITY If the current is steady i.e., the rate of flow of charge
1. What is current electricity ? does not change with time, then
1= lim
t.Q = dQ 1 milliampere = 1 mA = 10-3 A
Ilt -7 a M dt
1 microampere = 1 !iA = 10-6A
(3.1)
3.2 PHYSICS-XII
R
T = 21tr
taken as the direction of electric current. However, a v
negative charge moving in one direction is equivalent to
Frequency of revolution, v = 2=~
an equal positive charge moving in the opposite T Zrtr
direction, as shown in Fig. 3.1. As the electrons are Current at any point of the orbit is
SI
negatively charged particles, so the direction of electronic I = Charge flowing in 1 revolution
current (i.e., the current constituted by the flow of x No. of revolutions per second
electrons) is opposite to that of the conventional
or I = e v = 3!!.- .
current. 21tr
Units Used
Conventional current Electronic current
~ •• Electric charge is in coulomb (C), time in second
(s), and current in ampere (A)
Constant Used
IT
~I
Charge on an electron, e = 1.6 x 1O-19c.
Fig. 3.1 Flow of negative charge is equivalent to the flow of
positive charge in the opposite direction. Example 1. 1020 electrons, each having a charge of
1.6 x 10-19 C, passfrom a point A towards another point Bin
5. Is electric current a scalar or vector quantity ? 0.1 s. What is the current in ampere? What is its direction?
H
Electric current is a scalar quantity. Although Solution. Here n = 1020, e = 1.6 x 10 -19 C, t = 0.1 s
electric current has both magnitude and direction, yet Current,
it is a scalar quantity. This is because the laws of
ordinary algebra are used to add electric currents and
the laws of vector addition are not applicable to
O
J.ne addition of electric currents. For example, in The direction of current is from B to A.
(Fig. 3.2, two different currents of 3 A and 4 A flowing Example 2. Show that one ampere is equivalent to aflow of
in two mutually perpendicular wires AO and BO meet 1018 elementary charges per second.
6.25 x [CaSE D 92C]
at the junction 0 and then flow along wire Oc.
Solution. Here 1=1 A, t = 1 s, e = 1.6 x 10-19 C
The current in wire OC is 7 A which is the scalar
M
R
Number of electrons,
It 0.5 xl 18
Dimension of a =[t~] = ~; = Ar1
n=- = 19 = 3.125 x 10 .
e 1.6 x 10-
SI
electron revolves around the nucleus in a circular path of Dimension of c = [q] = AT
radius 5.1 x 10 -11m at afrequency of6.8 x 1015 revolutions
per second. Calculate the equivalent current. (ii) Current, 1= dq = ~ (at2 + bt + c) =2at + b
dt dt
Solution. Here r = 5.1 x 10-11m, At t = 5 s, I= 2 x 5 x 5 + 3 = 53 A.
v =6.8 x 1015 rps, e =1.6 x 10-19 C
Current,
rp roblems For Practice
1= e v = 1.6 x 10-19 x 6.8 x 1015 = 1.088 x 10-3 A. 1. One billion electrons pass from a point P towards
IT
another point Q in 10-3 S . What is the current in
Example 6. In a hydrogen atom, an electron moves in an
ampere? What is its direction?
orbit of radius 5.0 x 10-11 m with a speed of 2.2 x 106 ms-1.
(Ans. 1.6 x 10-7 A, direction of
Find the equivalent current. (Electronic charge = 1.6 x 10-19
current is from Q to P)
coulomb). [Roorkee 84]
2. If 2.25 x 1020
electrons pass through a wire in one
Solution. Here r = 5.0 x 10-11m, minute, find the magnitude of the current flowing
H
v=2.2x106ms-1, e=1.6xlO-19C through the wire. [Punjab 02] (Ans. 0.6 A)
Period of revolution of electron, 3. A solution of sodium chloride discharges
T=2rrr =2rrx5.0x10-
11
s 6.1 x Hy6 N a + ions and 4.6 x 1016Cl" ions in 2 s. Find
the current passing through the solution.
v 2.2 x 106
O
Current, 1= e v = 1.6 x 10-19 x 7 x 1015 0.6 x Hy6 revolutions per second around the nucleus.
Determine the average current at any point on the
= 1.12 x 10-3 A.
orbit of the electron. (Ans. 0.96 mA)
Example 7. Figure 3.3 shows a plot of current I through 6. An electron moves in a circular orbit of radius 10 em
the cross-section of a wire over a time interval of10 s. Find with a constant speed of 4.0 x 106 ms-1. Determine
the amount of I(A) the electric current at a point on the orbit.
charge that flows (Ans. 1.02 x 10-12 A)
through the wire 5
during this time 7. In a hydrogen discharge tube, the number of
protons drifting across a cross-section per second is
period.
[CBSE00 lSC]
1.1 x 1018, while the number of electrons drifting in
the opposite direction across another cross-section
5 10 t (s)
is 3.1 x 1018 per second. Find the current flowing in
Fig. 3.3 the tube. (Ans. 0.672 A)
3.4 PHYSICS-XII
R
4. q = It = 2.0 x10-6 x5 x60 = 6.0 x10-4 C. form lower potential to higher potential i.e., in the
5. [= ve = 0.6 X 1016 X 1.6 x 10-19 opposite direction of the electric field. Clearly, a charge
= 0.96 X 10-3 A = 0.96 mA flow circuit is analogous to the water flow circuit.
6
6. T = 21tr = 21t x O.!O s :. V = ~ = 4 x 10 s-1
SI
V 4 x 10 T 21t X 0.10 A-:~ Source of emf
19 B - (Charge pump)
l __ 4x106 x 1.6 x 10- -102 10-12A
=ve - -. X • R
21t xO.10
(n + ne) e
7. [= l + I = --,-P--- Fig. 3.5 A closed charge flow circuit.
P" t
(1.1 X 1018 + 3.1 X 1018) x 1.6 X 10-19 3.4 ELECTROMOTIVE FORCE : EMF
= = 0.672A
1
7. Define emf of a battery. Is it really aforce? When is
IT
the emf of a battery equal to the potential difference
3.3 MAINTENANCE OF STEADY CURRENT
between its terminals? Define emf of 1 volt.
IN A CIRCUIT
Electromotive force. A battery is a device which
6. With the help of a mechanical analogy, explain how maintains a potential difference between its two ter-
f+-
~ theflow of electric current is maintained in an electric circuit. minals A and B.
---B ·~-------------I
Maintenance of steady current in an electric circuit.
H
The flow of electric current in a circuit is analogous to
F, •
the flow of water in a pipe. As shown in Fig. 3.4,
suppose we wish to maintain a steady flow of water in : A Fe B I
-+
Suppose the force on a positive charge q is F". As the
-11\
charges build up on the two terminals A and B, a
'n\
potential difference is set up between them. An electric
I
-+
field E is set up in the electrolyte from A to B.This field
-+ -+
h
Water exerts a force Fe = q E on the charge q, in the opposite
pump -+
direction of Fn. In the steady state, the charges stop
l~A======~ ..'
accumulating further and F" = Fe .
The work done by the non-electrostatic force during
the displacement of a charge q from B to A is
W=Fn d
Fig. 3.4 A closed water flow circuit. where d is the distance between the terminals A and B.
CURRENT ELECTRICITY 3.5
The work done per unit charge is In case of a closed circuit, we can define emf in
e = w = Fn d another way as follows:
q q The emf of a source may be defined as the energy supplied
by the source in taking a unit positive charge once round the
e
The quantity = W / q is called the electromotive force complete circuit. Again, we note that
or emf of the battery or any other source.
The electromotive force of a source may be defined as
emf= Work done or e= W ~
Charge q
the work done by the source in taking a unit positive charge
from lower to the higher potential. Literally, emf means the force which causes the
flow of charges in a circuit. However, the term emf is a
If the two terminals of the battery are not connected misnomer. The emf is not a force at all. It is a special
R
externally, then case of potential difference, so it has the nature of work
Fn = Fe = qE done per unit charge.
Fnd = Fed = q Ed = qV SI unit of emf is volt. If an electrochemical cell supplies
an energy of 1 joule for the flow of 1 coulomb of charge
where V = Ed is the p.d. between the two terminals. Thus,
through the whole circuit (including the cell), then its emf is
SI
e = Fn d = qV =V said to be one volt.
q q
3.5 EMF VS. POTENTIAL DIFFERENCE
Hence the emf of a source is equal to the maximum potential
8. Give important points of differences between
difference between its terminals when it is in the open circuit
electromotive force and potential difference.
i.e., when it is not sending any current in the circuit.
Differences between electromotive force and
Basically, an electrochemical cell consists of two potential difference.
electrodes P and N immersed in an electrolyte, as
shown in Fig. 3.7 Electromotive force Potential difference
IT
1. It is the work done by a It is the amount of work
C I R D
AA source in taking a unit done in taking a unit
YYY
charge once round the charge from one point
complete circuit. of a circuit to another.
I P N I
R
2. It is equal to the maxi- Potential difference
mum potential diffe- may exist between any
~
potential difference is
caused.
The two electrodes exchange charges with the
6. It is equal to the sum of Every circuit
electrolyte. Consequently, the positive electrode P potential differences component has its own
develops a positive potential V+ (V+ > 0) with respect to across all the compo- potential difference
its adjacent electrolyte marked A The negative nents of a circuit inclu- across its ends.
electrode N develops a negative potential- V_ (V_ >0) ding the p.d. required
to send current through
with respect to the adjacent electrolyte B. When. no the cell itself.
current flows through the cell, the electrolyte has the
7. It is larger than the p.d. It is always less than the
same potential throughout, so that the potential dif- across any circuit emf.
ference between the· two electrodes P and N is element.
V+ - (- V_) = V+ + V_ = e, the emf. 8. It is independent of the It is always less than the
external resistance in emf.
Obviously, V+ + V_ > o. the circuit.
3.6 PHYSICS-XII
9. State Ohm's law. Define resistance and state its S1unit. Variable ~ or-+-
Ohm's law. On the basis of his experimental resistor ~
observations, a German physicist George Simon Ohm Potential ~ or ~
derived a relationship between electric current and
potential difference in 1828. This relationship is known
divider I t
as Ohm's law and can be stated as follows: Meters --0-- -0- -@- -<D-
The current flowing through a conductor is directly
Voltmeter Ammeter Galvanometer
proportional to the potential difference applied across its
ends, provided the temperature and other physical Fig. 3.9 Symbolsfor resistors and meters.
R
conditions remain unchanged.
Thus, Potential difference ex: Current
10. Briefly explain how can we measure the resistance
of a wire.
Vex: I
Measurement of resistance. Fig. 3.10 shows a simple
V=RI
SI
or circuit for measuring the resistance of a wire. Here the
battery and ammeter are connected in series with the
The proportionality constant R is called the resis-
wire and the voltmeter in parallel with it. The ratio of
tance of the conductor. Its value is independent of V
the voltmeter reading (V) and the ammeter reading (/)
and I but depends on the nature of the conductor, its
gives the resistance (R) of the wire.
length and area of cross-section and physical con-
ditions like temperature, etc. Ohm's law may also be Battery
expressed as + ,\I-----{
V=R
IT
r
R
T~raph ~etween the
potential difference V
applied across a conductor v~
to the current I flowing
through it is a straight line, Fig. 3.8 V-I graph for an Voltmeter
H
as shown in Fig. 3.8. ohmic conductor. Fig. 3.10 To measure resistance of wire.
Resistance. The resistance of a conductor is the property
by virtue of which it opposes the flow of charges through it. 3.7 FACTORS AFFECTING THE
The more the resistance, the less is the current I for a RESISTANCE : RESISTIVITY
given potential difference. It is equal to the ratioof the poten- 11. What are the factors on which the resistance of a
O
tial difference applied across the conductor to the current conductor depends? Define resistivity and state its SI unit.
flowing through it. Thus Factors affecting the resistance. At a constant
R=V temperature, the resistance of a conductor depends on
I the following factors :
M
51 unit of resistance is ohm (Q). If the potential 1. Length. The resistance R of a conductor is directly
difference (V) is 1volt and current (I) is 1ampere, then proportional to its length i.e.,
the resistance (R) is 1ohm. . R ex:I
I volt
10h m=---- 2. Area of cross-section. The resistance R of a uniform
1ampere conductor is inversely proportional to its area of cross-
section A, i.e.,
or
I
Rex:-
Thus, the resistance of a conductor is said to be 1 ohm if A
a current of 1 ampere flows through it on applying a 3. Nature of the material. The resistance of a
potential difference of 1 uolt across its ends. conductor also depends on the nature of its material.
Any material that has some resistance is called a resistor. For example, the resistance of a nichrome wire is
Pictorial symbols for resistors and meters are given in 60 times that of a copper wire of equal length and area
Fig. 3.9. of cross-section.
CURRENT ELECTRICITY 3.7
I I
R ex: - or R =p -
A A --+
A
where p is the constant of proportionality called resistivittj
or specific resistance of the material of the conductor. It --+
A
depends on the nature of the material of the conductor (a) (b)
and on the physical conditions like temperature and
Fig. 3.11 Current density.
pressure but it is independent of its size or shape.
Resistivity or specific resistance. If in the above Current density,
R
equation, we take . I I
}=-=
I = 1unit and A = 1square unit An A cos 8
then R =p -> ->
Thus, the resistivity or specific resistance of a material
or I = jA cos 8 = j .A
SI
may be defined as the resistance of a conductor of that This equation again shows that electric current,
material, having unit length and unit area of cross-section. being scalar product of two vectors, is a scalar quantity.
Or, it is the resistance offered by the unit cube of the material The 51 unit of current density is ampere per square
of a conductor. metre (Am -2) and its dimensions are [AL-2].
51 unit of resistivity. We can write
NOT E The current I through a particular surface 5 in
RxA ->
p=-- a conductor is the flux of j through that surface and is
I
. f ohm x metre/ given by the surface integral
51 unit 0 p = ------
IT
metre I=ff.as
= ohm meter (Q m) 5
->
Thus, the 51 unit of resistivity is ohm metre (Q m). where dS is a small element of the given surface area.
1
inside a conductor is defined as the amount of chargeflowing or G=-
R
per second through a unit area held normal to the direction of
the flow of charge at that point. It is a vector quantity The 51 unit of conductance is ohm-lor mho or
having the same direction as that of the motion of the siemens (S)
positive charge. It is a characteristic property of any Conductivity. The reciprocal of the resistivity of a
M
->
point inside the conductor and is denoted by j . material is called its conductivity and is denoted by 0.
Thus,
As shown in Fig. 3.11(a), if a current 1is flowing
.. 1
uniformly and normally through an area of cross- Con ductivity = ----
section A of a conductor, then the magnitude of current Resistivity
density at any point of this cross-section will be 1
or 0 =-
.
q/ t I p
t=r+=r:
A A The 51 unit of conductivity is ohm -1 m -1 or mho m-1
If the area A is not perpendicular to the direction of or Sm -1.
current and normal to this area makes angle 8 with the
Vector form of Ohm's Law. If E is the -nagnitude of
direction of current as shown in Fig. 3.11(b), then the
electric field in a conductor of lengti I, then the
component of A normal to the direction of current
potential difference across its ends is
flow will be
~=Acos8 V= EI
3.8 PHYSICS-XII
Also from Ohm's law, we can write Table 3.1 Electrical resistivities of some substances
V = IR = Ipl
I
~ EI=-pl
A
I
A
...
or E = jp
~ A. Conductors
As the direction of current density j is same as
~ Silver 1.6 x 10-8 0.0041 1
that of electric field E, we can write the above
Copper 1.7 x 10-8 0.0068
R
equation as
~ ~ Aluminium 2.7 x 10-8 0.0043 3
E =p j
Tungsten 5.6 x 10-8 0.0045 6
~ ~
or j = CJ E Iron 10 x 10-8 0.0065 8
SI
The above equation is the vector [orm of Ohm's Platinum 11 x 10-8 0.0039 10
law. It is equivalent to the scalar form V = RI. Mercury 98 x 10-8 0.0009 2
13. How can we classify solids on the basis of their Manganin 48 x 10-8 0.002 x 10-3
(alloy of Cu.
resistivity values ?
Ni, Fe, Mn)
Classification of solids on the basis of their
IT
B. Semiconductors
resistivity values. The electrical resistivity of sub-
Carbon 3.5 x 10- 5 - 0.0005 4
stances varies over a very wide range, as shown in
(graphite)
Table 3.1. Various substances can be classified into
three categories: Germanium 0.46 -0.05 4
the lowest resistivities of all the metals, so their wires Glass 1010 _ 1014
are used for transporting electric current over large Hard Rubber 1013 _ 1016
O
if
The carbon resistors are widely used in electronic First significant figure
circuits of radio receivers, amplifiers, etc. They have
lr
Second significant figure
the following advantages :
(i) They can be made with resistance values rang-
I Decimal multiplier
R
Colour code for resistors. A colour code is used to
indicate the resistance value of a carbon resistor and its Red Red Red Silver
.J.- .J.- .J.- .J.-
percentage accuracy. The colour code used throughout
the world is shown in Table 3.2. 2 2 2 ± 10%
SI
Table 3.2 Resistor colour code
White W 9 109
Violf1jet
Yellow _ :wn
~r=Gold
How to remember colour code:
-t~~)--
O
bands is printed on the resistor which reveals the three bands of green, violet and red colours ; the
following facts : resistance value is
1. The first band indicates the first significant figure.
Green Violet Red No 4th band
2. The second band indicates the second significant .J.- .J.- .J.- .J.-
figure. 5 7 2 ± 20%
3. The third band indicates the power of ten with
R = 57 x 102 Q ± 20%.
which the above two significant figures must be
multiplied to get the resistance value in ohms.
4. The fourth band indicates the tolerance or possible
variation in percent of the indicated value. If the )--
fourth band is absent, it implies a tolerance of
±20%. Fig. 3.15
3.10 PHYSICS-XII
R
2. The colour of the end gives the second signi-
= [2.7 x 1018 + 1.0 x 1018] x 1.6 x 10-19
ficant figure.
3. The colour of the dot gives the number of zeroes = 3.7 x 1.6 x 10-1 =0.592 A
to be placed after the second figure. Effective resistance,
4. The colour of the ring gives the tolerance or
SI
R = V = 230 0 =388.50 =- 3.9 x 102 O.
percent accuracy of the indicated value. I 0.592
Illustration. Suppose for a given resistor, the body Example 10. A 10 V battery of negligible internal
colour is yellow, end colour is violet, dot colour is resistance is connected across a 200 V battery and a resis-
orange and the ring colour is silver. tance of 38 0 as shown in the figure. Find the value of the
current in circuit.
Body End I Dot Ring
[CBSE D13]
ow
Yei Vi~let I Orrge I er
Silr 10V
IT
4 7' 3 ± 10%
=-
R R 10 3
.. 1 1 [ No. of electrons,
5. Con ductivity = ---- or (J=- =-
Resistivity P RA
q It
. Current . I n=-=-
6. Current density = --- or ] =- e e
Area A
0.02 x 1 17
7. Colour code of carbon resistors. Refer to Table 3.2. ---Cl~9 = 1.25 x 10 .
1.6 x 10-
Units Used
Potential difference V is in volt (V), current I in (ii) Current density,
ampere (A), resistance R in ohm (0), resistivity p . I I 0.02
in Om, conductance in ohm -lor mho or siemens ] = A= 1tr2 = 3.14 x (0.1 x 10-3)2
(S), conductivity in 0 -1m -lor Sm -1 and current
density j in Am -2. = 6.37 x 105 Am -2.
CURRENT ELECTRICITY 3.11
Example 12. Current flows through a constricted con- Solution. Resistance of the arc lamp is
ductor, as shown in Fig. 3.17. The diameter 01 =2.0 mm R= V =80 =8D
and the current density to the left of the constriction is I 10
7 = 1.27 x 106 Am-2. (i) What current flows into the In order to use arc lamp with a source of 240 V, a
constriction ? (ii) If the current density is doubled as it resistance R' should be connected in series with it so
emerges from the right side of the constriction, what is that current through the circuit does not exceed 10 A.
diameter 02 ? Then
, I(R+R')=V or 1O(8+R')=240
,
,
, ,
r
= 24 - 8 = 16 D.
, II , ,, or R'
I
, , ,
,
R
, , , Example 15. Calculate the resistivity vf a material of a
,
, wire 10 m long, 0.4 mm in diameter and having a resistance
of2.0 D. [Haryana 02]
Fig. 3.17
Solution. Here I= 10 m, r = 0.2 mm = 0.2 x 10- 3 m,
Solution. Here 01 = 2.0 mm, 71 = 1.27 x 106 Am -2, R=2D
SI
72 =2 71 Resistivity,
(i) Current flowing into the constriction, RA Rxnr2
P==-I-= I
r n( r
=3.14 x [(5 x 10-2)2 -(4.5 x 10-2)2]
H
or 71 x n( ~1 =2jl x ~2 r. 72 =271] = 14.9x10-4 m2
R- i_1.7x10-8x5
= 0.707 x 2.0 mm = 1.414 mm. - P A - 14.9 x 10-4
Example 13. A current of 2 mA is passed through a = 5.7 x 10-5 D.
colour coded carbon resistor with first, second and third
rings of yellow, green and orange colours. What is the Example 17. Find the resistivity of a conductor in which a
voltage drop across the resistor? current density of 2.5 Am-2 is found to exist, when an
M
Electrical conductivity = 1 Example 22. The resistance of a wire is R ohm. What will
Resistivity be its new resistance if it is stretched to n times its original
1 I 3 length?
or 0=-=-=---------0-
P RA 2 x 0.02 x 10-6 Solution. In both cases, volume of the wire is same.
= 75 x 106 0-lm-1. V= Al = A'l'
Example 19. A wire of resistance 4 0 is used to wind a coil or A l'
-=-=n [.: I' = nil
of radius 7 em The wire has a diameter of 1.4 mm and the A' I
specific resistance of its material is 2 x 10- 7 Om Find the l'
number of turns in the coil. R' P =: l' A 2
-=-LL=-.-=n.n=n
R
Solution. Let n be the number of turns in the coil. p-I I A'
R
Then total length of wire used A
2
=21t Rx n =21tx7x 10-2 x n metre or R'=n R.
Total resistance, Example 23. A cylindrical wire is stretched to increase its
1 length by 10%. Calculate the percentage increase in
SI
R=p- or resistance.
A
Solution. New length, l' = 1+ 10% of I
n= 70.
=1+0.11=1.11
Example 20. A wire of 10 ohm resistance is stretched to
thrice its original length. What will be its (i) new resistivity, or £. = 1.1
and (ii) new resistance? [CBSE D 98C]
I
Solution. (i) Resistivity p remains unchanged AI=A'l'
A l'
because it is the property of the material of the wire. or
IT
A' I
(ii) In both cases, volume of wire is same. So
V= A'l' =AI ~=£.x~=(£.)2 =(1.1l=1.21
R I A' I
A' I I 1
or -=-=- [.: l'=1+21=3ij The percentage increase in resistance,
A l' 31 3
l' R' - R x 100 =
-R- (R')R -1 x 100 =(1.21-1)x 100 = 21%.
H
R' P A' l' A 3 3
-=--=-x -=-x -=9
R
p-I I A' 1 1 Example 24. Two wires A and B of equal mass and of the
A same metal are taken. The diameter of the wire A is half the
Hence R' =9R =9 x 10 =900. diameter of wire B. If the resistance of wire A is 240,
Example 21. A wire has a resistance of16 O. It is melted calculate the resistance of wire B.
O
and drawn into a wire of half its length. Calculate the Solution. Mass of wire = volume x density
resistance of the new wire. What is the percentage change in = area of cross - section x lengthx density
its resistance ? 2 2
m = 1trA IA d = 7trB IBd
Solution. In both cases, volume of the wire is same.
(rA J2 =(~)2
M
r
2
R' P A'
-=--=-x
l'
l' All
-=-x -=-
1 :~ 0: ':~: 0;~,(:. 0 ~'G)'0 1~
R
p-IIA'224 1trA
A 1 1
or RB = 16 RA = 16 x 240 = 1.5 O.
or R' = .! R =.! x 16 = 40.
4 4
Example 25. A piece of silver has a resistance of 10. What
Change in resistance
will be the resistance of a constantan wire of one-third length
R - R'
= --
12
x 100 = - x 100 = 75%. and one-half diameter, if the specific resistance of constantan
R 16 is 30 times that of silver ?
CURRENT. ELECTRICITY 3.13
R
ttd
perpendicular cross-section per second. Calculate
Example 26. On applying the same potential difference (i) the current and (ii) the current density in the
between the ends of wires of iron and copper of the same electron beam.
length, the same current flows in them. Compare their radii. [Ans. (i) 9.6 x 10-3 A (ii) 9.6 x 103Am -2]
Specific resistances of iron and copper are respectively
SI
1.0 x 10- 7 and 1.6 x 10- 8 nm Can their current-densities 7. Calculate the electric field in a copper wire of
be made equal by taking appropriate radii ? cross-sectional area 2.0 mm2 carrying a current of
1 A.The resistivity of copper = 1.7 x 10-8 nm.
Solution. On applying same potential difference,
(Ans. 0.85 x 10-2 Vm -1)
same current flows in the two wires. Hence the
resistances of the two wires should be equal. 8. A given copper wire is stretched to reduce its
diameter to half its previous value. What would be
1 1
But R=p-=p- its new resistance? [CBSE D 92C]
A nr2
(Ans. R' = 16 R)
IT
For the two wires of same length 1, we have
9. What will be the change in resistance of a
1 1 constantan wire when its radius is made half and
Rl =PI-2 and ~ =P2 -2'
rt r1 rt r2 length reduced to one-fourth of its original length ?
(Ans. No change)
As Rl=~
10. A wire of resistance 5 n is uniformly stretched until
PI P2 or its new length becomes 4 times the original length.
1=rf
H
Find its new resistance. (Ans.80n)
11. A metallic wire of length 1 m is stretched to double
riron = Piron = 1.0 x 10- 7 = 2.5.
its length. Calculate the ratio of its initial and final
~opper Pcopper 1.6 x 10- 8
resistances assuming that there is no change in its
O
No, current densities cannot be equal because they density on stretching. [CBSE D 94]
depend on nature of the metals. (Ans.1 : 4)
12. A wire of certain radius is stretched so that its
cproblems ForPractice radius decreases by a factor n Calculate its new
resistance. (Ans. n4 R)
M
square faces (ii) between the opposite rectangular R' = p 'l/ 4 = p _1_ = R
faces of the block. The resistivity of carbon is 1t (Y /2)2 ny2
3.5 x 10-50 cm. (Ans. 0.1750, 7.0 x 10- 5 0)
I .
17. Two wires A and B of the same material have their 10. R=p- =50
A
lengths in the ratio 1 : 5 and diameters in the ratio
3 : 2. If the resistance of the wire B is 1800, find the R'=p ~ = 16p ~ = 16 R= 16 x5= 800.
resistance of the wire A. (Ans. 160) A/4 A
I
18. A uniform wire is cut into four segments. Each 11. R=p-
A
segment is twice as long as the earlier segment. If
R
the shortest segment has a resistance of 4 0, find the R' = p ~ = 4p ~ = 4 R
resistance of the original wire. (Ans. 60 0) A/2 A
.. R: R' =1: 4.
19. Calculate the conductance and conductivity of a
wire of resistance 0.010, area of cross-section 12. V = A' I' = Al
10-4m2 and length 0.1 m. [Haryana 2000]
SI
(Ans. 100 S, 105 Sm -1) or V = 1t (.;;Y I' = 1t y2 I or I'= ~ I
HINTS 2
t][~r
V 30 4
I =- = 4 = 0.5 x 10 A.
R 60 x 10 13. Rz = 1) [
2. I = V = 10 V = ~ = 10-2A
R UO
10000 3 ]2 = 4.4 O.
IT
= 4.2 [1.5] [ 0.13 x 10-
2 1 0.155xlO-3
_ 3. _ ~ _ 10- x5 x 60 _ 1 875 1019
n - e - e - 1.6 x 10 19 -. x
14. Length of the wire used, I = lOOn D
4 . A s R-pl_~
-A-n';
I 100nD lOOp D
R=p ~ =P'-;r=-,;-
8
100 x 4.2 x 10-7 x3 x 10-2
H
:. y2 =~ = 64 x 10- x1.98x7 = 5.76x 1O-8m2
nR 22x7 -----"3 "2-- = 7.875O.
(O.4x10 )
or y = 2.4 x 10-4 m. I
15. As R=p-
I V A
5. Use R=p~and I=-.
nr R pi 1.68 x 10-8 x10xl03 5 2
O
A=-= =1.68xl0- m
q ne 6 x 1(y.6 x 16 x 10-19 R 10
6. (i) I=-=-=------
t t 1 Mass of copper required,
= 9.6 xl0-3 A. m = Volume x density = Al x density
(ii) Current density, = 1.68 x 10-5 x 10 x 103 x 8.9 x 103
M
.-i - 3
9.6 x 10- _ 9 6 103 Am-2
] - A - 1.0x 10-6 - . x .
= 1495.2 kg.
5 2
16. (i) R = ~ = 3.5 x 10- x50 x 10- = 0.175o.
V IR Ipl Ip 1x1.7x10-8 PALO x 10-2 x 1.0x 10-2
7. E=-=-=-=-=---..".-
I I IA A 20 x 10-6 5 2
(ii) R = ~ = 3.5 x 10- x 1.0x 10- = 7.0xl0-50.
=0.85 xl0-2Vm-1. PALO 2
x 10- x 50 x 10- 2
8. When the diameter of the wire is reduced to its half IA
value, area of cross-section becomes one-fourth and
the length increases to four times the original 17. RA
R8
= P~
_18_
= IA
18 dA
(.'!JL]2 =.! x
5 3
(~)2 =~
45
length. .
p nd~ /4
, I' 41 I
R =p A,=P'-1-=16p A =16R. 4 4
-A RA = 45 R8 = 45 xI80=160.
4
CURRENT ELECTRICITY 3.15
18. Let the lengths of the four segments be I, 2/, 41and flowing steadily in a circular loop. Their observations
8/. Then their corresponding resistances will be R, indicated that
2R, 4R and 8R
1. The sign of the charges is negative.
Given R=4Q 2. The ratio e / m of the charges is equal to that mea-
Resistance of the original wire sured for the electrons in other experiments.
= R + 2R + 4R + 8R = 15R = 15 x 4 = 60Q. It was thus established directly that current in
1 1 metals is carried by negatively charged electrons.
19. Conductance, G = - = - = 100 S.
R 0.01
Conductivity, 3.12 MECHANISM OF CURRENT FLOW IN A
cr=..!=_I_= 0.1 =10SSm-1. CONDUCTOR : DRIFT VELOCITY AND
R
p RA 0.01 x 10- 4 RELAXATION TIME
18. Explain the mechanism of the flow of current in a
3.11 CARRIERS OF CURRENT metallic conductor. Hence define the terms drift velocity
and relaxation time. Deduce a relation between them.
16. Mention different types of charge carriers In
SI
solids, liquids and gases. Mechanism of the flow of electric' charges in a
Carriers of current. The charged particles which by metallic conductor : Concepts of drift velocity and
flowing in a definite direction set up an electric current are relaxation time. Metals have a large number of free
electrons, nearly 1028 per cubic metre. In the absence of
called current carriers. The different types of current
any electric field, these electrons are in a state of
carriers are as follows:
continuous random motion due to thermal energy. At
1. In solids. In metallic conductors, electrons are
room temperature, they move with velocities of the
the charge carriers. The electric current is due to the
order of 105ms-1. However, these velocities are
drift of electrons from low to high potential regions. In
IT
distributed randomly in all directions. There is no
n-type semi-conductors, electrons are the majority
preferred direction of motion. On the average, the
charge carriers while in p-type semiconductors, holes
number of electrons travelling in any direction will be
are the majority charge carriers. A hole is a vacant state
from which an electron has been removed and it acts as equal to number of electrons travelling in the opposite
~ ~ ~
a positive charge carrier. direction. If u1' u2' .... , UN are the random velocities of N
H
2. In liquids. In electrolytic liquids, the charge free electrons, then average velocity of electrons will be
carriers are positively and negatively charged ions. For
~ ~ ~
example, CuS04 solution has Cu2+ and SO~- ions, ~ u1+u2+···+uN
u = =0
which act as the charge carriers. N
3. In gases. In ionised gases, positive and negative Thus, there is no net flow of charge in any direction.
O
an acceleration a given by
17. Why is it that electrons carry current in metals? ~
Metallic conduction. In metals, the atoms are ~ Force eE
a =--=--
closely packed. The valence electrons of one atom are Mass m
close to the neighbouring atoms and experience electrical where m is the mass of an electron. As the electrons
forces due to them. So they do not remain attached to a accelerate, they frequently collide with the positive
particular atom, but can hop from one atom to another metal ions or other electrons of the metal. Between two
and are free to move throughout the l.')ttice.These free successive collisions, an electron gains a velocity
electrons are responsible for conduction in metals. component (in addition to its random velocity) in a
~
The fact, that the negatively charged electrons carry direction opposite to E. However, the gain in velocity
current in metals, was "first experimentally confirmed lasts for a short time and is lost in the next collision. At
by the American physicists Tolman and Stewart in 1917. each collision, the electron starts afresh with a random
They measured the angular momentum of the charges thermal velocity.
3.16 PHYSICS-XII
,r
R
~
The average velocity v d of all the N electrons will be Fig. 3.18 Slow and steady drift of an electron in the opposite
->
direction of E. The solid lines represent the path in the
-> ->
absence of E and dashed lines in the presence of E.
SI
~ ~ ~ ~ ~ ~
= (u1 +a '1)+(u2 +a '2)+···+(uN +a 'N)
3.13 RELATION BETWEEN ELECTRIC CURRENT
N
~ ~ ~ AND DRIFT VELOCITY: DERIVATION OF
= u1 + ~ + ...+ UN + -; '1 + '2 + ...+ 'N OHM'S LAW
N N 19. Derive relation between electric current and drift
~ velocity. Hence deduce Ohm's law. Also write the
=0 +a ,
expression for resistivity in terms of number density offree
IT
where '=('1 +'2+·····+'N)/N is the average time electrons and relaxation time.
between two successive collisions. The average time that
Relation between electric current and drift velocity.
elapses between two successive collisions of an electron is
Suppose a potential difference V is applied across a con-
called relaxation time. For most conductors, it is of the
ductor of length I and of uniform cross-section A. The
order of 10-14 s. The velocity gained by an electron
electric field E set up inside the conductor is given by
during this time is
H
~ E= V
~ ~ e E, I
vd =a ,=---. ~
m
Under the influence of field E, the free electrons
~ ~
The parameter v d is called drift velocity of begin to drift in the opposite direction E with an
electrons. It may be defined as the average velocity gained
O
14 I ~I
the electron keeps colliding with the positive metal E Free electron
----+
ions. The velocity gained by it due to the electric field is
-e -e
lost in next collision. As a result, it acquires a constant
~ ~ -e -e
average velocity vd in the opposite direction of E. The --e -e
motion of the electron is similar to that of a small
Conventional
• •• Electronic
spherical metal ball rolling down a long flight of stairs.
current current
As the ball falls from one stair to the next, it acquires
acceleration due to the force of gravity. The moment it
collides with the stair, it gets decelerated. The net effect + 11-
Battery
is that after falling through a number of steps, the ball
begins to roll down the stairs with zero average
Fig. 3.19 Drift of electrons and electric field inside a conductor.
acceleration i.e., at constant average speed. Moreover,
CURRENT ELECTRICITY 3.17
R
electron density of the conductor.
:. Current, I=:J. = enAI or I=enAvd
t 1/ vd 2. The relaxation time r, the average time between
two successive collisions of an electron.
This equation relates the current I with the drift ~ ~
velocity vd. 20. Write relation between quantitiesj r 0 and E.
SI
The current density' j , is given by ~ ~
Relation between j , 0 and E . For an electron,
. I
] = - =envd q=-e
A
~
7 ~ ~ e E«
In vector form ] = en vd and vd =---
m
The above equation is valid for both positive and ~
negative values of q. 7
] = nqvd ~
=n(-e)
( e E
----;;;-
't) =-;;;-
ne 2
't ~
E
Deduction of Ohm's law. When a potential
IT
ne2't
= -1 = 0,
difference V is applied across a conductor of length I, ..
But -- conductivity of the conductor
the drift velocity in terms of V is given by m p
eE't eV't ~ ~ ~ ~
vd=-=-
m mi j =0E or E =p j
If the area of cross-section of the conductor is A and This is Ohm's law in terms of vector quantities like
~ ~
H
the number of electrons per unit volume or the electron
current density j and electric field E.
density of the conductor is n, then the current through
the conductor will be 21. What causes resistance in a conductor?
eV't Cause of resistance. Collisions are the basic cause of
I=enAvd =enA.-
mi resistance. When a potential difference is applied
O
22. Alloys of metals have greater resistivity than their Example 27. Assuming that there is one free electron per
constituent metals. Why ? atom in copper, determine the number of free electrons in
High resistivity of nichrome. In an alloy, e.g., 1 metre3 volume of copper. Density of copper is
nichrome (Ni - Cr alloy), Ni2+ and Cr3+ ions have 8.9 x 103 kgm-3 and atomic weight 63.5. (Avogadro's
different charge and size. They occupy random locations number, N = 6.02 x 1026 per kg-atom).
relative to each other, though their ionic sites form a Solution. If the atomic weight of a material is M kg
regular crystalline lattice. An electron, therefore, passes and the density is d kgm -3, then the volume of its
through a very random medium and is very frequently 1 kg-atom will be (Mid) m3.
deflected. So there is a small relaxation time and hence According to Avogadro's hypothesis, there are
large resistivity. In general, alloys have more resistivity 6.02 x 1026 atoms in 1 kg-atom of the material. This
than that of their constituent metals. number is called Avogadro's number (N). Thus
R
23. Explain the cause of instantaneous current in an Number of atoms in (Mid) m3 volume of a material
electric circuit. =N
Cause of instantaneous current. Although the drift .. Number of atoms in 1 m3 volume
speed of electrons is very small, typically 1 mm/s, yet N dx N
=--=--
SI
an electric bulb lights up as soon as we turn the switch Mid M
on. This is because electrons are present everywhere in
Assuming 1 free electron per atom in copper, the
an electric circuit. When a potential difference is
number of free electrons in 1 m3 volume of copper will be
applied to the circuit, an electric field is set up through-
dx N
out the circuit, almost with the speed of light. Electrons n=--
in every part of the circuit begin to drift under the M
influence of this electric field and a current begins to Now d =8.9 x 103 kg m-3, N =6.02 x 1026,
flow in the circuit almost immediately. M =63.5 kg
IT
The above situation is analogous to the flow of 8.9 x 103 x 6.02 x 1026 28 3
water in a long pipe. As soon as the pressure is applied n= = 8.4 x 10 m -
63.5
at one end of the water filled pipe, a pressure wave is
transmitted along the pipe with a speed of about Example 28. A copper wire has a resistanceof10 n and an
1400 ms - 1. When this wave reaches the other end, area of cross-section 1 m~. A potential difference of 10 V
water starts flowing out. But water inside pipe moves exists across the wire. Calculate the drift speed of electrons if
the number of electrons per cubic metre in copper is
H
forward with a much smaller speed.
8 x 1028 electrons. [CBSE D 96]
Solution. Here R= lOn, A=lmm2=10-6m2,
28
V = 10 V, n =8 x 10 electrons I m3
Formulae Used Now I = en A vd
O
R
n = number of Cu atoms =8.5 x 1028
I =1 A, o =6.25 x 107Sm-1
Now 1=1.5 A, A=10-7m2, e=1.6xlO-19C
As j = ~ = crE
I 1.5 A
v - - - ----::-;,-----:::.,,-----:=-
d - enA - 1.6 x 10-19 x 8.5 x 1028 x 10-7 E=_1_= 1
SI
Acr 2.0 x 10-6 x 6.25 x 107
15 = 11
• x 10-3 ms.-1
16 x 85 x 10 = 8 x 10-3 Vm-1.
(b) (i) At any temperature T, the thermal speed of a Example 31. A potential difference of100 V is applied to
copper atom of mass M is given by the ends of a copper wire one metre long. Calculate the
average drift velocity of the electrons. Compare it with the
_tkBT thermal velocity at 27°C. Given conductivity of copper,
vrms - M
c = 5.81 x 107 ~r 1 m-1 and number density of conduction
But ordinary temperature, T "'-300 K, electrons, n =8.5 x 1028m-3. [NCERT]
IT
Boltzmann constant, kB = 1.38 x 10-23 JK-I, Solution. Electric field,
Mass of a copper atom,
E= V =100V =100Vm-1
M_ 63.5 63.5 x 10-3 k I 1m
- 6.0 x 1023 g 6.0 x 1023 g
As j=crE=envd
3 x 1.38 x 10- 23x 300 x 6.0 x 1023 :. Drift speed,
H
63.5 x 10- 3 crE 5.81x 107 x 100
v =-=
d e n1.6 x 10-19 x 8.5 x 1028
= .J117354.33 =342.57 ms-1
= 0.43 ms-1.
From part (a), drift speed of electrons,
kB = 1.38 x 1O-23JK-1,
O
Now,
vd = 1.1 x 1O-3ms-1
T = 27 + 273 = 300 K
vd (electrons)
---'''------- =
1.1 x 10-3
= 3.21 x 10- .
6 me = 9.1 x 10-31 kg
vrms (Cu atoms) 342.57
Thermal velocity of electron at 27°C,
..! mv~
M
Solution. Here p = 1.7 x 10-8 nm, n = 8.5 x 1028m -3, :. Potential difference,
e =1.6x 1O-19 c, me =9.1x 10-31 kg, vd =1.6 x 10-4 ms-l. V = IR = 1.0 x 1.7 x 10-3 = 1.7 x 10-3 V.
m (ii) Current density,
As resistivity, p = _e_
ne21" . = .i = 1.0 = 1.0 x 106 Am -2.
:. Relaxation time, ] A 1.0 x 10-6
m 9.1 x 10-31 (iii) Free-electron density,
1" - __ e_ - --------:;;c-;;,-------;;;;-------;;-
- e2 np - (1.6 x 10-19)2 x 8.5 x 1028 x 1.7 x 10-8 dx N 8.9 x 103 x 6.02 x 1026
n =-- =---------
= 2.5 x 10-14s M 63.5
R
=v it = 1.6 x 10-4 x 2.5 x 10-14 :. Drift velocity,
= 4.0 x 10-l8m. j 1.0 x 106
V = - = -----;r;------::;o
d en 1.6 x 10- 19 x 8.4 x 1028
Example 33. An aluminium wire of diameter 0.24 em is
connected in series to a copper wire of diameter 0.16 em The = 7.4 x 10-5 ms-l•
SI
wires carry an electric current of 10 ampere. Find
(i) current-density in the aluminium wire (ii) drift velocity ~roblems For Practice
of electrons in the copper wire. Given: Number of electrons
per cubic metre volume of copper =8.4 x 1028. 1. The free electrons of a copper wire of cross-
Solution. (i) Radius of Al wire, sectional area 10-6 m 2 acquire a drift velocity of
10-4m/s when a certain potential difference is
r= 0.24 =0.12 em =0.12 xlO-2m applied across the wire. Find the current flowing in
2
the wire if the density of free electrons in copper is
Area of cross-section,
8.5 x 1028 electrons/m '. (Ans. 1.36 A)
IT
A = 1t? =3.14 x (0.12 x 10-2)2 = 4.5 x 10-6m2 2. Estimate the average drift speed of conduction
:. Current density, electrons in a copper wire of cross-sectional area
2.5 x 10-7 m 2 carrying a current of 2.7 A. Assume
.=.i= 10 =2.2x106Am-2.
the density of conduction electrons to be
] A 4.5 x 10-6
9x1028m-3. [CBSE OD 141
(ii) Area of cross-section of Cu wire is
H
(Ans. 0.75 mms ")
A = 1tX (0.08 x 10- )2 =2.0 x 10-6 m2
2
3. A current of 1.8 A flows through a wire of cross-
Also, sectional area 0.5 mm 2. Find the current density in
n = 8.4 x 1028m-3, e =1.6x 10-19 C, I =10 A the wire. If the number density of conduction
I 10 electrons in the wire is 8.8 x 1028m -3, find the drift
v - -- - ------:;-;::----~_;:_---__,_
O
R
1 0.2
aluminium wire is 1029 m - 3. vd = -en-A = -1-.6-x-1-0......,,19O-x-8-.4-x-1-0""'i2Q8
-x-7-.8-5-4-x-1-0-"7
(Ans. 4.976x106 Am-2, 1.28 x10-4ms-l)
= 1.895 x 10-sms-t.
9. A current of 30 ampere is flowing through a wire of
cross-sectional area 2 mm 2. Calculate the drift velo- 6. Drift velocity, vd = _l_
enA
SI
city of electrons. Assuming the temperature of the
2
wire to be 27°C, also calculate the rms velocity at
1.6 x 10-19 x 1029 x 1 x 10-6
this temperature. Which velocity is larger? Given
that Boltzman's constant = 1.38 x 10-23J K-1, den- =1.25 x 10-4ms-l.
sity of copper 8.9 g cm - 3, atomic mass of copper
. d . I 4 4
= 63. (Ans. 1.1 x 1O-3ms-l, 1.17 x 105ms-1) R eqUlre time, t = - = 4 = 3.2 x10 s.
vd 1.25 x 10-
10. What is the drift velocity of electrons in silver wire of
length 1 rn, having cross-sectional area 3.14 x 10-6m 2 7. I= !1. = ~ = 3.33 x 10-2 A
and carrying a current of 10 A ? Given atomic mass t 5 x 60s
IT
of silver = 108, density of silver = 10.5 x 103kg m -3, 1 1
charge on electron = 1.6 x 10-19 C and Avogadro's vd = enA = en (nr2)
number = 6.023 x 1026 per kg-atom. 3.33 x 10-2
(Ans. 3.399 x 1O-4ms-1) - 1.6 x 10 19 x 5 x 1022 x 106 x3.14 x(10 3)2
11. When a potential difference of 1.5 V is applied
= 1.326 x10-6 ms-l.
H
across a wire of length 0.2 m and area of cross-
8. As the two wires are connected in series, so current
section 0.3 mm 2, a current of 2.4 A flows through
through each wire, I = 10 A.
the wire. If the number density of free electrons
in the wire is 8.4 x 1028m -3, calculate the average (i) Current density in copper wire,
relaxation time. Given that mass of electron . I 10x4
= 9.1 x 10-31 kg ]= nd /4= 3.14 x(0.16xlO 2)2
O
=1.36A. =4.9x10-6m2
I 2.7 -1 1 10
2. vd = enA = 1.6 x 10-19 x9x1028 x2.5x10-7ms vd = -en-A = -1-.6-x-10-;Ot9"-x
-10....,2;n9-x-4-.9-x-1-0-,6
R
The rms velocity of electrons at 27°C (= 300 K) is e
given by
For a hole,
23
v = ~ 3 kB T = 3 x 1.38 x 10-
rms m 9 x 10-31 The mobilities of both electrons and holes are
1.17 x 105 ms-1 positive; although their drift velocities are opposite to
SI
=
each other.
The rms velocity is about 108 times the drift velocity.
51 unit of mobility = m2V-1s-1
10. Mass of silver wire,
Practical unit of mobility = cm2 V-Is-I.
m = Al P = 3.14 x 10-6 x 1x 10.5 x 103
1m2V-1s-1 =104 cm2 V-1s-1
No. of electrons per unit volume of silver,
Relation between electric current and mobility for a
6.023 x 1023 3.14 x 10.5 x io-3 conductor
n= x -----,---
108 3.14 x 10- 6 xl In a metallic conductor, the electric current is due to
IT
= 5.8557 x 1028 its free electrons and is given by
I I = enAvd
v -
d - enA But vd =I--leE I=enAl--le E
10 This is the relation between electric current and
= 1.6 x 10-19 x 5.8557 x 1028x3.14 x 10-6 electron mobility.
3.399 x 10-4 ms-1 Relation between electric current and mobility for a
H
=
semiconductor
11. E= V = 1.5 V = 7.5 Vm-1. The conductivity of a semiconductor is both due to
I 0.2m
electrons and holes. So electric current in a semi-
Current density, conductor is given by
O
't - -- - ------".,,-------,-,,-,,--
- ne E - 8.4 x 1028x(1.6 x 10 19)2x 7.5
2
Conductivity of a semiconductor. According to
= 4.51 x 10-16 s. Ohm's law,
I= V =~=EA ...(ii)
R pl/ A P
3.14 MOBILITY OF CHARGE CARRIERS
From equations (i) and (ii), we get
24. Define mobility of charge carrier. Write relations
between electric current and mobility for (i) a conductor EA
-=eAE(nl--le+Pl--lh)
and (ii) a semiconductor. Hence write an expression for p
the conductivity of a semiconductor. 1
or - = e(nl--le + PI--lh)
Mobility. The conductivity of any material is due to p
its mobile charge carriers. These may be electrons in But 1/ p is the electrical conductivity cr. Therefore,
metals, positive and negative ions in electrolytes; and
electrons and holes in semiconductors. cr=e(nl--le+Pl--lh)
CURRENT ELECTRICITY 3.23
Table 3.3 Mobilities in some materials at room Example 37. A semiconductor has the electron concen-
temperature, in cm2v-1s-1 tration 0.45 x 1012 m- 3 and hole concentration 5 x 1020m-3.
Find its conductivity. Given : electron mobility
Materials Electrons Holes =0.135 ~V-ls-l and hole mobility =0.048 ~V-ls-l ;
Diamond 1800 1200 e = 1.6 x 10-19 coulomb.
Silicon 1350 480 Solution. Here n = 0.45 x 1012m -3, P = 5 x 1020m -3,
R
+ 5 x 1020x 0.048) Sm-1
= 1.6 x 10- (0.06075 + 0.24 x 108) Sm-1
7
fl
SI
1. Mobility, 11 = vd = q't
E m roblems For Practice
2. Electric current, 1= enAvd = enA IlE
1. A potential difference of 4.5 V is applied across a
3. Conductivity of metallic conductor, (J = nelle conductor of length 0.1 m. If the drift velocity of
4. Conductivity of a semiconductor, (J = nell e + pell h electrons is 1.5 x 10-4 ms -1, find the electron
Units Used mobility. (Ans.3.33xlO-6m2V-1s-1)
3.15 TEMPERATURE DEPENDENCE OF Alloys have high resistivity. The resistivity of nich-
RESISTIVITY rome has weak temperature dependence [Fig. 3.20(b)]
while that of manganin is almost independent of
25. Explain the variation of resistivity of metals, temperature. At absolute zero, a pure metal has negli-
semiconductors, insulators and electrolytes with the gibly small resistivity while an alloy (like nichrome)
change in temperature. Define temperature coefficient of has some residual resistivity. This fact can be used to
resis tivi ty. distinguish a pure metal from an alloy.
Temperature dependence of resistivity. The resisti- I
As R = p- i.e., R oc p
vity of any material depends on the number density n A
of free electrons and the mean collision time 1:.
Thus equation (1) can be written in terms of resis-
m
R
P=-- tances as
2
ne 1:
R, = Ro (1 + a. t)
1. Metals. For metals, the number density n of free
where Rt = the resistance at tOC
electrons is almost independent of temperature. As
temperature increases, the thermal speed of free elec- Ra = the resistance at O°c, and
SI
trons increases and also the amplitude of vibration of t = the rise in temperature.
the metal ions increases. Consequently, the free elec- 2. Semiconductors and insulators. In case of insu-
trons collide more frequently with the metal ions. The lators and semiconductors, the relaxation time 1: does
mean collision time 1: decreases. Hence the resistivity of a
not change with temperature but the number density
metal (p oc 1/1:) increases and the conductivity decreases of free electrons increases exponentially with the
with the increase in temperature. increase in temperature. Consequently, the conductivity
For most of the metals, resistivity increases linearly increases or resistivity decreases exponentially with
with the increase in temperature, around and above the increase in temperature.
IT
the room temperature. In such cases, resistivity P at The number density of electrons at temperature T is
any temperature T is given by given by
_ -Eg /kBT
P = Po [1 + a. (T - To)] ...(1) n (T) - no e
where Po is the resistivity at a lower reference
where kB is the Boltzmann constant and Eg is the
.temperature to
(usually 20° q and a. is the coefficient
energy gap (positive energy) between conduction and
of resistivity. Obviously,
H
valence bands of the substance.
P -P 1 dp 1 .
a.= 0 As p oc - ,so we can wnte
Po (T - To) Po 'dT n
_1_ = ~ e-Eg /kBT
Thus, the temperature coefficient of resistivity a.
may be defined as the increase in resistivity per unit p (T) Po
O
the resistivity of a pure metal increases as a higher power At room temperature, kBT =0.03 eV. Whether the
of temperature, as shown for copper in Fig. 3.20(a). non-conducting substance is an insulator or a semi-
conductor, depends on the size of the energy gap, Eg:
E
~ 0.4 (i) If E ::;1 eV, the resistivity at room temperature
eo ~
a:
I;:: 1.20
is ~ot very high and the substance is a
a: .c semiconductor .
:~ 1.
.;;; (ii) If E > 1 eV, the resistivity at room temperature
~ 1.00 is v~ry high (-103 n m) and the substance is an
o 50 100 150 200 400 600 800 insulator.
Temperature T(K) ....• Temperature T (K) ....•
(a) (b) The coefficient of resistivity (u) is negative for
Fig. 3.20 (a) Variation of resistivity p of copper with carbon and semiconductors i.e., their resistivity
temperature. (b) Variation of resistivity p of decreases with temperature, as shown in
Fig. 3.21.
nichrome with temperature.
CURRENT ELECTRICITY 3.25
t=2.= 1 =2560C
a 3.9 x 10- 3
Fig. 3.21 Resistivity of a semiconductor decreases Example 39. The resistance of the platinum wire of a
rapidly with temperature. platinum resistance thermometer at the ice point is 50 and
at steam point is 5.39 O. When the thermometer is inserted
R
3. Electrolytes. As the temperature increases, the in a hot bath, the resistance of the platinum wire is 5.975 O.
interionic attractions (solute-solute, solvent- solute and Calculate the temperature of the bath. [ TERT]
solvent-solvent types) decrease and also the viscous
Solution. Here Ro = 5 0, RlOO = 5.23 0, Rt = 5.795 0
forces decrease, the ions move more freely. Hence
conductivity increases or the resistivity decreases as As R, = Ro (1+ at)
SI
the temperature of an electrolytic solution increases. Rt - Ro = Ro at ... (i)
26. Why alloys like constantan or manganin are used and RlOO - Ro = Ro a x 100 ... (ii)
for making standard resistors? On dividing (i) by (ii), we get
Use of alloys in making standard resistors. Alloys Rt - Ro = .L.
like constantan or manganin are used for making stan-
RlOO - Ro 100
dard resistance coils because of the following reasons:
R - R
(i) These alloys have high value of resistivity. or t= t "0 x 100
R100 - Ro
IT
(ii) They have very small temperature coefficient.
So their resistance does not change appreciably = 5.795 - 5 x 100 = 0.795 x 100 = 345.650C
even for several degrees rise of temperature. 5.23 -5 0.23
(iii) They are least affected by atmospheric Example 40. A nichrome heating element connected to a
conditions like air, moisture, etc. 220 V supply draws an initial current of2.2 A which settles
(iv) Their contact potential with copper is small. down after afew seconds to a steady value of 2.0 A. Find the
H
steady temperature of the heating element. The room
Examples based on temperature is 30° C and the average temperature coefficient
Tern erarure Variarion of Resisrance of resistance of nichrome is 1.7 x 10-4 per ° C.
Formulae Used Solution. Here V =220 V, II =2.2 A, I2 =2.0 A,
= 1.7 x
O
a = 0 or R, = ~ (1+ at)
Ro xt ~ = V =220 =1100
12 2.0
Units Used
R - R
Resistances are in 0, temperatures in °C or K. As a = ''2 1
Rl (t2 - t1)
Example 38. (i) At what temperature would the resistance _ _ ~ - Rl _ 110 -100 -5880C
t2 tl - - -
of a copper conductor be double its resistance at O°C ? Rl a 100 x 1.7 x 10- 4
tii) Does this temperature hold for all copper conductors
Steady temperature,
regardless of shape and size ?
t2 = 588 + tl = 588 + 30 = 618°C.
Given afor Cu =3.9·x 10-3 °C-1.
R-R 2R-R 1 Example 41. An electric toaster uses nichrome (an alloy of
Solution. (i) a = ''2 1 = 0"0 =- nickel and chromium) for its heating element. When a
R1(t2-t1) Ro(t-O) t
negligibly small current passes through it, its resistance at
3.26 PHYSICS-XII
R
the resistance of the material of the coil is 0.0042 °C-1.
_ _ ~ - R1 _ 85.8 -75.3 -8200C
t2 t1 -
R1 a
-
75.3 x 1.70 x 10- 4
- Solution. Rl = Ra (1 + a t1) and ~ = Ra (1 + a t2)
Rl = 1+ a tl
Steady temperature,
~ 1+ a t2
SI
t2 '" 820 + t1 =820 + 27 = 847°C.
Here, Rl =2 0, ~ =2.1180, t2 =30°C, tl =?
At the steady temperature, the heating effect due to
the current equals heat loss to the surroundings. 2 = 1 + 0.0042 x tl = 1 + 0.0042 x tl
Example 42. The resistance of a tungsten filament at 2.118 1 + 0.0042 x 30 1.126
150°C is 133 ohm. What will be its resistance at 500°C?
or 1 + 0.0042 t = 2 x 1.126 = 2.252
The temperature coefficient of resistance of tungsten is
1 2.118 2.118
0.0045 peri C.
Solution. Here R1S0 =1330, a =0.0045°C, Rsoo =? •• t = _1_ [2.252 -1] = 0.104 ::.150C
IT
1 0.0042 2.118 0.0042 x 2.118
Now R, = Ra (1 + at)
i.e., the marking will be correct at 15°C
R1S0 = Ra (1 + a x 150)
Example 45. A potential difference of 200 V is applied to a
or 133 = Ro (1 + 0.0045 x 150) ...(1)
coil at a temperature of 15°C and the current is 10 A. What
and Rsoo = Ra (1 + a x 500) will be the mean temperature of the coil when the current has
H
or Rsoo = Ra (1 + 0.0045 x 500) ...(2) fallen to 5 A, the applied voltage being same as before ?
Dividing (2) by (1), we get Given a =_1_oC-1 atO°e.
234
Rsoo = 1 + 0.0045 x 500 = 3.25
Solution. In the second case, the current decreases
133 1 + 0.0045 x 150 1.675 due to the increase in resistance on heating.
O
Example 46. The resistances of iron and copper wires at 3. The resistance of a silver wire at O°Cis 1.25 O. Up to
20°C are 3.9 0 and 4.1 0 respectively. At what temperature what temperature it must be heated so that its
will the resistances be equal ? Temperature coefficient of resistance is doubled? The temperature coefficient
resistivity for iron is 5.0 x 10-3 K-1 and for copper it is of resistance of silver is 0.00375 °C-1. Will the
4.0 x 10-3 K-1. Neglect any thermal expansion. temperature be same for all silver conductors of all
Solution. Let resistance of iron wire at tOe shapes? (Ans. 26~ C, Yes)
4. The resistance of a coil used in a platinum-resis-
= Resistance of copper wire at tOe
tance thermometer at O°C is 3.000 and at 100°C is
~o [1 + a (t -20)] = R;o [1 + a' (t -20)] 3.75 O. Its resistance at an unknown temperature is
measured as 3.15 O. Calculate the unknown
3.9[1 + 5.0 x 10-\t -20)] = 4.1 [1 + 4.0 x 10-3 (t -20)] temperature. (Ans.200q
R
[3.9 x 5 - 4.1x 4]x 10-3 x (t -20) = 4.1-3.9 5. The temperature coefficient of a resistance wire is
t -20 = 0.2 =64.5 0.0012.soC-1. At 300 K, its resistance is 10. At what
3.1 x 10- 3 temperature the resistance of the wire will be 20 ?
[lIT 80]
t = 64.5 + 20 = 84.5°C.
SI
(Ans. 1127 K)
Example 47. A metal wire of diameter 2 mm and length
6. The temperature coefficient of resistivity of copper
100 m has a resistance of 0.54750 at 20°C and 0.805 0 at
is 0.004°C-1. Find the resistance of a 5 m long
150°C. Find (i) the temperature coefficient of resistance
copper wire of diameter 0.2 mm at 100°C, if the
tii) resistance at O°C (iii) resistivities at 0° and 20°C.
resistivity of copper at O°C is 1.7 x 10-8 Om.
Solution. Here r = 1 mm = 10-3m, 1=100 m, (Ans. 3.80)
t1 =20°C, R1 =0.54750, t2 =150°(, ~ =0.8050
HINTS
(i) Temperature coefficient of resistance is
IT
~ - R1 0.805 - 0.5475 1. a = Rf - Ra = 20 - 10 = ~ 0C1.
a= =------ Ra x t 10 x 273 273
R1 (t2 - t1) 0.5475 (150 - 20)
2. Here t
300 - 273 = 27°C
=
= 3.6 x 10-30(-1.
Rz7 = Ra (1 + a x 27)
(ii) Resistance at OO( is
3.115= Ra(1+4.2x10-3 x27) ...(1)
R =~ = 0.5475 = 0.5475
H
o l+at1 1+3.6xl0-3x20 1.072 and 3 = Ra(I + 4.2 x 10-3 xt) ...(2)
Po - -1- - I - 100
This gives,
= 1.60 x 10-8 Om. t = 17.2°C = 17.2 + 273 = 290.2 K.
Resistivity at 20 0
( is 3, Proceed as in Example 38, page 3.25.
P20 = Po (1 + a t) 4. Rf = Ra (1+ at)
M
R
R=p it? = 3.14x(0.lx10 3)2 ':::3.BQ. becomes non-ohmic at higher currents, as shown in
Fig. 3.23.
SI
27. State the conditions under which Ohm's law is
not obeyed in a conductor. What are ohmic and
non-ohmic conductors ? Give examples of each type.
Limitations of Ohm's law. Ohm's law is obeyed by
many substances under certain conditions but it is not
a fundamental law of nature.
Fig. 3.23 V-I graphfor a metallicconductor.
Ohmic conductors. The conductors which obey Ohm's
law are called Ohmic conductors. For these conductors, (ii) Water voltameter. Here a back e.m.f. is set up
IT
the linear relationship between voltage and current due to the liberation of hydrogen at the cathode and
(Vex: I) holds good. The resistance (R = V / I) is oxygen at the anode. No current flows through the
independent of the current I through the conductor. In voltameter until the applied p.d. exceeds the back
these conductors, the current I gets reversed in direc- e.m.f. Vo (1.67 V for water voltameter). So V-I graph is
tion when the p.d. V is reversed, but the magnitude of a straight line but not passing through the origin, as
current changes linearly with voltage. Thus the V-I shown in Fig. 3.24. Hence the electrolyte (water
H
graph for ohmic conductors is a straight line passing through acidified with dil. ~S04) is a non-ohmic conductor.
the origin. A metallic conductor for small currents and
the electrolyte like copper sulphate solution with
copper electrodes are ohmic conductors, as shown in
Figs. 3.22(a) and (b) respectively.
O
+1 +1
Pure CuS04 solution
V
metal with Cu electrodes Vo
R
shown in Fig. 3.26. It can be easily seen that (a) the V-I
~ 0.08
relation is non-linear, (b) V-I relationship is different
for positive and negative values of V, and (c) in certain
portions, there are two or more values of current for
o 2 4 6
the same value of voltage, i.e., the V-I relationship is T(K)~
SI
not unique. The region AB is interesting because the
current carried by the device increases as the voltage Fig. 3.28 Mercury loses complete resistance at 4.2 K.
decreases, i.e., a is negative in this region.
The phenomenon of complete loss of resistivity by
certain metals and alloys when they are cooled below
a certain temperature is called superconductivity.
The temperature at which a substance undergoes a
transition from normal conductor to superconductor in
IT
a zero magnetic field is called transition or critical
-v +v
temperature (Te).
A current once set up in a superconductor persists
for a very long time without any apparent change in its
magnitude.
Cause of superconductivity. It is believed that near
H
-1
the transition temperature, a weak attractive force acts
Fig. 3.26 V-I curve for a thyristor. on the electrons which brings them closer to form
coupled pairs. Such coupled pairs are not deflected by
(v) Gallium arsenide. Fig. 3.27 shows the V-I graph ionic vibrations and so move without collisions.
O
for the semiconductor GaAs. It exhibits non-linear 29. What is Meissener effect in superconductors ?
behaviour. Moreover, after a certain voltage, the Meissener effect. In 1933, Meissner and Ochsenfeld
current decreases as the voltage increases. That is, if !1V observed that if a conductor is cooled in a magnetic
is positive then M is negative and hence the effective field to a temperature below the transition tempe-
resistance (= !1V / M) is negative. rature, then at this temperature, the lines of magnetic
M
~•... I
I
...•
;:J I B
u
Voltage V (V) ~
The expulsion of the magnetic flux from a supercon- is no change in the current in the circuit and in the potential
ducting material when it is cooled to a temperature difference between the terminals of the combination, then the
below the critical temperature in a magnetic field is single resistance is called the equivalent resistance of the
called Meissner effect. combination.
Meissner effect indicates that as the supercon- 32. When are the resistances said to be connected in
ductivity appears in a material, it becomes perfectly series? Find an expression for the equivalent resistance of
diamagnetic. a number of resistances connected in series.
30. What is high Tc superconductivity ? Mention Resistances in series. If a number of resistances are
important applications of superconductors. connected end to end so that the same current flows through
High Tc superconductivity. A current once set up in
each one of them in succession, then they are said to be
R
a superconducting loop can persist for years without
connected in series. Fig. 3.30 shows three resistances R1,
any applied emf. This important property of super- ~ and ~ connected in series. When a potential dif-
conductors can have important practical applications. ference V is applied across the combination, the same
A serious difficulty in their use is the very low tem- current I flows through each resistance.
IE;:.:JI
perature at which they must be kept. Scientists all over
SI
the world are busy to construct alloys which would be
superconducting at room temperature. Superconduc-
tivity at around 125 K has already been achieved and
efforts are being made to improve upon this temperature.
Material Tc (K)
IT
By Ohm's law, the potential drops across the three
Hg 4.2 resistances are
Pb2Au 7.0 VI = IR1, V2 = I~, V3 = l~
YBa2Cu307 90 If Rs is the equivalent resistance of the series
combination, then we must have
T12Ca 2Ba 2Cu3010 120
V = IRs
H
Applications of superconductors. The possible But V = Sum of the potential drops across the
applications of superconductors are individual resistance
1. For producing high magnetic fields required for or V=VI +V2+V3
research work in high energy physics.
or IRs = IRI + I~ + l~
O
5. In long distance power transmission without Thus when a number of resistances are connected in
any wastage of power. series, their equivalent resistance is equal to the sum of the
individual resistances.
3.18 RESISTANCES IN SERIES AND PARALLEL
Laws of resistances in series
31. What do you mean by equivalent resistance of a
(i) Current through each resistance is same.
combination of resistances ?
(ii) Total potential drop = Sum of the potential drops
Equivalent resistance of a combination of resis-
across the individual resistances.
tances. Sometimes, a number of resistances are
connected in a circuit in order to get a desired value of (iii) Individual potential drops are directly proportional
current in the circuit. Resistances can be connected in to individual resistances.
series, in parallel or their mixed combination can be used. (iv) Equivalentresistance= Slim of the individual resistances.
If a combination of two or more resistances in any electric (v) Equivalent resistance is larger than the largest
circuit can be replaced by a single resistance such that there individual resistance.
CURRENT ELECTRICITY 3.31
33. When are the resistances said to be connected in Laws of resistances in parallel
parallel ? Find the equivalent resistance of a number of (i) Potential drop across each resistance is same.
resistances connected in parallel. (ii) Total current = Sum of the currents through
Resistances in parallel. If a number of resistances are individual resistances.
connected in between two common points so that each of (iii) Individual currents are inversely proportional to the
them provides a separate path for current, then they are said individual resistances.
to be connected in parallel. Fig. 3.31 shows three resis-
(iv) Reciprocal of equivalent resistance = Sum of the
tances R1, ~ and R3connected in parallel between
reciprocals of the individual resistances.
points A and B. Let V be the potential difference
applied across the combination. (v) Equivalent resistance is less than the smallest
individual resistance.
R
R1
E •I b d
B ." Combfnation of Resistances in
,. . . Series and Parallel
SI
Formulae Used
1. The equivalent resistance Rs of a number of resis-
tances connected in series is given by
v Rs=R,.+~+~+·..
Fig. 3.31 Resistance in parallel. 2. The equivalent resistance Rp of a number of resis-
tances connected in parallel is given by
Let 11, 12 and 13 be the currents through the resis- 1 1 1 1
-=-+-+-+ ...
tances R1, R2 and R3 respectively. Then the current in
IT
Rp R,. ~ ~
the main circuit must be I = II + 12 + 13
3. For two resistances in parallel,
Since all the resistances have been connected
between the same two points A and B, therefore, poten- Currents through the two resistors will be
tial drop V is same across each of them. By Ohm's law, I = ~ I and I = R,. I
1 R,.+~ 2 R,.+~
the currents through the individual resistances will be
H
V V V Units Used
11 =- , 12 =- , 13 =-
R1 ~ ~ All resistances are in ohm (n).
If R is the equivalent resistance of the parallel Example 48. A wire of resistance 4 R is bent in theform of
combin~tion, then we must have a circle (Fig. 3.32). What is the effective resistance between
O
But 1=11 + 12 + 13
A~B
V V V V
or -=-+-+-
M
R
RI =180, Rz =100. bination. So resistance between the ends A and E is
Solution. As the resistances R3 and R4 are in series,
R' = 10 + 20 + 10 = 26.67 O.
their equivalent resistance 3
= 5 + 10 =150. Example 52. A set of n identical resistors, each of resis-
SI
tance R 0, when connected in series have an effective resis-
tance X 0 and when the resistors are connected in parallel,
their effective resistance is YO. Find the relation between
e=60V R,X and Y.
Solution. The effective resistance of the n resistors
connected in series is
Fig. 3.35 X = R + R + R + .....n terms = nR
IT
The series combination of R3 and R4 is in parallel The effective resistance Y of the n resistors
with Rz. Their equivalent resistance is connected in parallel is given by
1 1 1 1 n
R,=10x15 =150=60 - = - + - + - + ....n terms =-
10 + 15 25 Y R R R R
•. Voltage drop across R} are 10 0 and 120, find the third one.
= IRI = 2.5 x 18 V = 45 V. [Punjab 91; Haryana 94]
Example 51. A letter A consists of a uniform wire of Solution. Here Rp = V = ~ = 4 0
resistance 1ohm per em. The sides of the letter are each 20 em I 7.5
long and the cross-piece in the middle is 10 em long while the 1 1 1 1
M
But -=-+-+-
apex angle is 60°. Find the resistance of the letter between the Rp R} ». ~
two ends of the legs.
Solution. Clearly, 1 1 1 1
or -=-+-+-
AB= BC=CD= DE= BD=10 em 4 10 12 ~
1 1 11 1
R} = Rz = ~ = R4 = Rs = 10 0 or ---=- .. ~ =150.
As Rz and ~ are in series, their combined resis- ~ 4 60 15
tance = 10 + 10 = 20 O. This combination is in parallel Example 54. When a current of 0.5 A is passed through
with Rs (=100). two resistances in series, the potential difference between the
Hence resistance 'between points Band D is ends of the series arrangement is 12.5 V. On 'connecting
given by them in parallel and passing a current of 1.5 A, the potential
1 1 1 3 difference between their ends is 6 V. Calculate the two
-=-+-=- or
R 20 10 20 resistances.
CURRENT ELECTRICITY 3.33
R
R = ~ = 1.7 x 10- x 20 x 10- =1.7x 10-3D
Rl - Rz = 15 ...(2) 1 A 2 x 10-6
SI
:. Resistance,
are connected in series, as shown in Fig. 3.37. Find the ratio 8 2
RA / RB of the resistance of the two plates. Rz = 2.6 x 10- x 20 x 10- =2.6 x 10- 3 D
2 x 10-6
Example 59. Find, in the given network of resistors, the Finally, resistance between A and B
equivalent resistance between the points A and B, between A 1
and D, and between A and C. [lIT] 1 1 =20
--+-
Solution. The resistors 3 +3 3
0 70 C
AD (= 3 0) and DC (= 7 0) Thus the effective resistance between A and B is
are in series to give a total 20.
resistance R' =100. The 30 50
Example 61. Find the effective resistance of the network
resistance R' (= 10 0) and
shown in Fig. 3.41 between the points A and B when (i) the
the resistor AC(=100)
A B switch S is open (ii) switch S is closed.
are in parallel. Their equi-
R
valent resistance is Fig. 3.39 60 120
R" = 10 x 10 = 50
10 + 10 5
A B
Now R" (= 50) and CB (= 50) are in series, their
SI
total resistance R'" = 10 o. Finally, R'" (= 10 0) and 120 60
AB (= 100) are in parallel between A and B. Hence the
Fig. 3.41
equivalent resistance between points A and B is
R = 10 x 10 = 5 O. Solution. (i) When the switch S is open, the resis-
AB 10 + 10 tances of 60 and 12 0 in upper portion are in series,
39 15 the equivalent resistance is 18 o. Similarly, resistances
Similarly, RAD =-0 and RAe =-0.
16 4 in the lower portion have equivalent resistance of 18 o.
Now the two resistances of18 0 are in parallel between
Example 60. Find the effective resistance between points A
IT
points A and B.
andB for the network shown in Fig. 3.40.
.'. Effective resistance between points A and B
o 30 E
= 18 x 18 = 90.
18 + 18
c
F (ii) When the switch S is closed, the resistances of
H
6 0 and 12 0 on the left are in parallel. Their equivalent
resistance is
A 30 B
O
between A and D Example 62. Calculate the current shown by the ammeter
1 A in the circuit shown in Fig. 3.42. [CBSE 00 2000]
1 1 0 =30
--+- 50
3 +3 6
Similarly, resistance between A and E
1
1 1 =30
--+-
3 +3 6
Resistance between A and F
1
1 1 =30 10V
--+-
3 +3 6 Fig. 3.42
CURRENT ELECTRICITY 3.35
R
3Q c E
B
10V
Fig. 3.43
SI
A I----'\IV\.,----'---'\/V\r----l F
. . 10 x 10 o
equivalent resistance = --- = 50
10 + 10
For two such combinations connected in series,
equivalent resistance = 5 + 5 = 10 0 15 V
Fig. 3.46
c The equivalent resistance of the circuit is
6x3
R=--+3=50
6+3
M
5Q
Current drawn from the battery,
I=15V =3A
50
o
Current through the branch BCD,
Fig.3.44 I =_3_. x I=~x3=lA
1 6+3 9
1 1 1 1
Solution. --=-+-+-- Current through the arm DF = I =3 A
RBA 15 30 5+ 5
P.D. across the capacitor
6 1 = P.D. between points C and F
-=-
30 5 = P.D. across CD + P.D. across DF
RBA =50 =3x1+3x3=12V.
3.36 PHYSICS-XII
Example 65. A battery of emf 10 V is connected to Example 67. In the circuit shown in Fig. 3.50, Rl = 4 0,
resistances as shown in Fig. 3.47. Find the potential e
Rz = ~ = 150, R4 = 300 and = 10 V. Work out the equi-
difference between the points A and B. valent resistance of the circuit and the current in each resistor.
H1 A sn II RI A [eBSE D 2011]
c o
3n B In
lOV B
R
Fig. 3.50
Fig. 3.47 Solution. The resistances Rz, ~ and R4are in parallel.
4x4 Their equivalent resistance R' is given by
Solution. Total resistance, R = -- =20
4+4 111111151
-=-+-+-=-+-+-=-=-
SI
V lOV R' Rz ~ R4 15 15 30 30 6
Current I = - = -- =5A
, R 20
or R' =60
As each of the two parallel branches has same The resistance Rl is in series with R'. Hence total
resistance (40), so the current of 5 A is divided resistance of the circuit is
equally through them.
R=R1+R'=4+6=100
Current through each branch =5/2 =2.5 A
The current II is the current sent by the cell e in the
Now Vc -VA =2.5xl=2.5V
whole circuit.
IT
and Vc - VB= 2.5 x 3 = 7.5 V
I =-=-=lA
e
10
VA - VB=(Vc - VB)-(Vc - VA) 1 R 10
= 7.5 -2.5 = 5.0 V. Potential drop between A and B,
Example 66. What is the equivalent resistance between V = II R' = 1 x 6 = 6 V
points A and B of the circuit shown in Fig. 3.48? [lIT 97] This is the potential drop across each of the resis-
H
tances Rz, ~ and R4 in parallel. Therefore, currents
through these resistances are
A~B V 6 V 6
12 = - = - = 0.4 A; I3 = - = - = 0.4 A
Rz 15 . ~ 15
O
Fig. 3.48 V 6
and I4 = - = - = 0.2 A.
R4 30
Solution. Obviously, the points A and Dare
equipotential points. Also, the points B and C are equal Example 68. Find the equivalent resistance between the
potential points. So the given network of resistances points A and B of the network of resistors shown in Fig. 3.51.
reduces to the equivalent circuit shown in Fig. 3.49.
M
o>----+--c::::-\.~o
A ~c B
Their equivalent resistance
=3+3=60
A~----~~r------PB
The 6 0 resistance is in
parallel with ~, so that
Fig. 3.49 their equivalent resistance
The three resistances form a parallel combination. 30
=6x3=20
Their equivalent resistance Req is given by 6+3 Fig. 3.51
Similarly, total resistance of the lower portion Solution. As shown in Fig. 3.55, the given hexagon
=5Q has a line of symmetry C1 C C2• So all points on this
Now we have three 5 Q resistors connected in line have the same potential i.e., potential at C1 =
parallel between the points A and B. Hence the equi- potential at C = potential at C2. Hence the points Cl, C
valent resistance R of the entire network is given by and C2 can be made to coincide with each other.
1 1 1 1 3 5
- = - + - + - =- or R = - Q. r/2 : r/2
R 5 555 3
Example 69. Find the effective resistance between points A
and B of the network of resistors shown in Fig. 3.52.
Solution. By c
R
symmetry, the potential
A B
drops across GC and GD
are equal, so no current H
SI
B
flows in the arm DE. r/2 : r/2
Hence the resistances in K
the arms CD and DE are Fig. 3.55
ineffective. The given
After this is done, the circuit splits into identical parts,
circuit reduces to the E
joined in series between the points A and B. One such
equivalent circuit shown
Fig. 3.52 part between A and C is shown in [Fig. 3.56] which, in
in Fig. 3.53.
turn, is equivalent to the circuit shown in Fig. 3.57.
G~~VVV- __C~~VVV---oH
IT
0 r/2 4r/3
Cj Cj
AO---+'" H~-oB
r
C C
A A
E
H
4r/3
Fig. 3.53 C2 C2
G r/2
Resistance of arm GH = r + r =2r
rxr rx r Fig. 3.56 Fig. 3.57
Resistance of arm 1/ = -- + -- =r
r+r r+r
From Fig. 3.57, the equivalent resistance R' between
O
A B
Solution. By symmetry, potential drops across AC Their equivalent resistance =r / 2 + r / 2 =r. This
and AD are equal. So resistance in arm CD is ineffec- resistance is in parallel with the resistance r / 2 along
tive. The given circuit reduces to the equivalent circuit BC
shown in Fig. 3.59. Clearly the equivalent resistance R :. Effective resistance between points P and Q
between points A and B is given by
rx (r/2) r
1 1 1 1 4 2 r+(r/2) 3
-=-+-+-=-=-
R 2r 2r r 2r r
~ roblems For Practice
or R =~ =0.5 r.
2
1. Given the resistances of 1n, 2 n and 3 n. How will
R
C r
you combine them to get an equivalent resistance of
(i) 11nand (ii) 11n ?
r 3 5 [CBSE F 2015]
0
[Ans. (i) parallel combination of In and 2n in
series with 3 n (ii) parallel combination of
SI
2 nand 3 n in series with 1n
A B
2. Given three resistances of 30 n each. How can they
Fig. 3.59 be connected to give a total resistance of (i) 90 n
(ii) 10 o (iii) 45 o ?
Example 72. Find the equivalent resistance of the circuit [Ans. (i) in series (ii) in parallel (iii) two
shown in Fig. 3.60 between the points P and Q. Each resistor resistances in parallel and one in series]
has a resistance r. 3. A 5 n resistor is connected in series with a parallel
combination of n resistors of 6 n each. The equi-
IT
A
valent resistance is 7 n. Find n. (Ans. 3)
4. A uniform wire of resistance 2.20n has a length of
2 m. Find the length of the similar wire which
connected in parallel with the 2 m long wire, will
give a resistance of 2.0 n. (Ans. 20 m)
H
5. A wire of 15n resistance is gradually stretched to
do ble its original length. It is then cut into two
p B C Q equal parts. These parts are then connected in
Fig. 3.60 parallel across a 3.0 volt battery, Find the current
drawn from ~e battery. [CBSE OD 09]
O
8. Three resistances, each of 40, are connected in the 14. Find the current through the 50 resistor in the
form of an equilateral triangle. Find the effective circuit shown in Fig. 3.65, when the switch 5 is
resistance between its corners. (Ans.2.670) (i) open and (ii) closed. [Ans. (i) 0.2 A, (ii) 0.6 A]
9. Two resistors are in the ratio 1 : 4. If these are Ion
5n
connected in parallel, their total resistance becomes
20 O. Find the value of each resistance.
[Punjab 2000]
5
(Ans. 250,1000) 3V
10. Five resistors are connected as shown in Fig. 3.62.
Find the equivalent resistance between the points B
and C.
R
[Punjab 011 Fig. 3.65
(Ans. 70/190)
9n 15. The letter A consists of a uniform wire of resistance
A B
10 cm -1. The sides of the letter are 40 ern long and
the crosspiece 10 em long divides the sides in the
ratio 1 : 3 from the apex. Find the resistance of the
SI
3n 5n
letter between the two ends of the legs.
[Punjab 9SC]
o zo (Ans. 66.670)
16. Calculate the equivalent resistance between points
Fig. 3.62
A and Bin each of the followingnetworks of resistors:
11. Four resistors of 120 each are connected in parallel. [Ans. (a) 120 (b) 40/30 (c) 20
Three such combinations are then connected in (d) 10/30 (e) 160 if) 50]
series. What is the total resistance? If a battery of
IT
9 V emf and negligible internal resistance is
connected across the network of resistors, find the
current flowing through each resistor. [Haryana 02] sn
(Ans. 90, 0.25 A) sn
12. If the reading of the ammeter ~ in Fig. 3.63is 2.4 A, A 5n
H
what will the ammeters Az and ~ read? Neglect the 5n 5n
resistances of the ammeters. (Ans. 1.6 A, 4.0 A)
II 20n A B
~"""-'V\./\r---{Al
(a) (b)
IOn
O
C
A
IOn 10n
Fig. 3.63
M
6V
(Ans. 0.18 A, 1.5 A)
IOn Ion
Ao-"".;v~~N\r-"""'V'II'v"""---OB
Ion
Ion
2Qt87Q
A 10n B
(e) (j)
20n
17. Calculate the resistance between points A and B for 19. Find the potential difference between the points A
the following networks: and B for the network shown in Fig. 3.69.
2 4 R (Ans. 8.0 V)
[Ans. (a) "3 0 (b) "3 0 (c) "3 0 (d) 60]
60
20
2A 2.50
A A B
30
(a)
R
Fig. 3.69
SI
B
(Ans. 22.5 V)
A
(b)
2A
4000
60V
(c) B
IT
Fig. 3.70 Fig. 3.71
4 r
[Ans. (a) "3 r (b) 4 (c) r]
M
25. Find the resistance between the points (i) A and B 29. In the circuit shown in Fig. 3.79, the battery has an
and (ii) A and C of the network shown in Fig. 3.75. emf of 12.0 V and an internal resistance of 5 R/ 11. If
[Ans. (i) 27.50 (ii) 30 OJ the ammeter reads 2.0 A, what is the value of R ?
(Ans.60)
Ion Ion 10 n
112 V
1
IOn Ion 1
1 5 Rill 1
-- 1
R
Fig. 3.75
SI
Fig. 3.80. (Ans. 3 A)
(Ans. 14.4 V, 0.8 A, 1.6 A) llV
Ion p
2.4A
p 4n
sn
Q
IT
Fig. 3.76 Fig. 3.80
R
6. ~ + ~ =90 ...(1)
resistance in branch Be. Hence the equivalent
~~ =2 resistance between B and C is
~+~ R = 14 x 5 = 70 O.
14 + 5 19
or ~ ~ = 2 (~ + ~) = 2 x 9 = 18
SI
11. The circuit diagram is shown in Fig. 3.81.
~ _ ~ = ~(~ + ~)2 _ 4 ~ ~
120 120 120
= ~81-72 = 30 ...(2)
120 120 120
On solving (1) and (2),
~=60,~=30 120 120 120
RA 10-7
7. (i) p = - = 40 x -- = 1.0 x 10- 7 Om 120 120 120
a 1 40 I
10-7
IT
and Pb = 20 x -- = 5.0 x10-8 Om
40
(ii) Total resistance, R = Ra + ~ = 40 + 20 = 600
Fig. 3.81
The current in the wires, 1 = V = 60 = 1.0 A.
R 60
Effective resistance R' of four resistances of 120
:. Potential differences between the ends of wires a each connected in parallel is given by
H
and bare 1 1 1 1 1 4
-=-+-+-+-=-
Va = 1 x Ra = 1.0 x 40 = 40 V R' 12 12 12 12 12
and Vb = 1 x ~ = 1.0 x 20 = 20 V or R' = 30
Electric fields in the two wires are Total resistance of the network,
O
When the rheostat is adjusted at the rrurumum (d) All the three resistances are connected in
resistance of 00, current will be maximum. parallel between points A and B.
6V 1 1 1 1 3 10
Imax ----15A
40 - . - =- + - + - =- or R = - O.
R 10 10 10 10 3
When the rheostat is adjusted at the maximum (e) The given network is equivalent to the net-
resistance of 300, current will be minimum. work shown in Fig. 3.83.
6V 10 x 15
I . = = 0.18A R = 10+ -- =160.
rrun (4 + 30)0
10+ 15
14. (i) When switch 5 is open, resistances of 50 and IOQ
100 are in series.
3V
R
IOQ 5Q
Current, I= = 0.2 A
(5+ 10)0
A B
(ii) When switch 5 is closed, no current flows
Fig. 3.83
through 10 0 resistance.
3V
:. Current, I = - = 0.6 A
if) Resistance in branch +
ADC = 2 4 = 6 O. This
SI
50 resistance is in parallel with 60 resistance in
arm AC
15. Refer to Fig. 3.82.Clearly Their equivalent resistance
BC= CD= BD= 10em
=6x6=30
AB= DE=30cm 6+ 6
~=~=Rs=100
The series combination of this 30 resistance
and 1) = R4 =300
and 70 resistance in arm BCis in parallel with
C 100 resistance in arm AB.
IT
R = 10 x 10 = 5 O.
10 + 10
17. The corresponding equivalent circuit diagrams are
given below:
H
2Q
2Q 2Q
A
2Q 2Q B
O
(a)
Fig. 3.82
(b) R = 5 + 10 x 5 + 5 = 40 O.
10 + 5' 3
(c) R = (3 + 3) 3 = 20.
(3 + 3) + 3
Fig. 3.84
3.44 PHYSICS-XII
18. (a) The equivalent network for 3.68(a) is shown in 21. Current through each branch = 2/ 2 = 1A
Fig. 3.85(a). Vc-VA=lx2=2V
r 4
R=r+-=-r. Vc-VB=lx3=3V
3 3
VA-VB=(VC-VB)-(VC-VA)=3-2=1V.
r
r 111111151
22. --=-+-+-=-+-+-=-=-
r RAB ~ R:, R4 5 5 10 10 2
r r
r RAB = 20
A B A B
r R = 1\ + RAB = 4+ 2 = 60
e 6V
R
(a) (b) 11 =-=-=lA
R 60
Fig. 3.85
(b) The equivalent network for 3.68(b) is shown in 2
Fig. 3.85(b). 14=- =0.2 A.
10
SI
~=~+~+~+~=~ or R=~. 23. The equivalent circuit is shown in Fig. 3.87. The
Rrrrrr 4 effective resistance between points C and D
(c) The current divides symmetrically in the two 3x6
upper and the two lower resistances. So the = --+ 8=100
3+ 6
resistances in the vertical arm are ineffective. The
3n
given network reduces ~
to the equivalent r r c o
network shown in A B A 8n B
r r
Fig. 3.86.
IT
30n
R = 2r x2r =r. Fig. 3.86
2r + 2r
Fig. 3.87
3x3
19. R = -- + 2.5 = 1.5 + 2.5 = 4.00 Now the 100 and 300 resistances are in parallel.
3+3
The equivalent resistance between points A and B
H
V = R1= 4.0 x 2 = 8.0 V. 10 x30
=--=7.50.
20. p.o. across 4000 resistance = 30 V 10 + 30
PD. across 3000 resistance = 60 - 30 = 30 V 24. Proceed as in Example 51 on page 3.32.
This shows that potential drop is same across both 25. (i) The equivalent circuit is shown in Fig. 3.88.
O
resistances.
10n E io n H 10n
Let R be the resistance of the voltmeter. Then
equivalent resistance of Rand 4000 connected in
parallel should also be 300 O. 10n io o
R x 400
--- = 300 or R = 12000
M
R+ 400
F 10n G
When the voltmeter is connected across the 3000
resistance, their equivalent resistance is given by Fig. 3.88
R' = 1200x 300 = 2400 Resistance of the arm EFGH = 10 + 10 + 10 = 300
1200+ 300 This resistance is parallel to the 100 resistance of
Total resistance in the circuit = 240 + 400 = 6400 arm EH.
Equivalent resistance between points E and H
in tthee circuit,
. . Current ill circuit.T = -60 = -3A
640 32 = 10 x 30 = 7.50
Reading of the voltmeter 10 + 30
Hence total resistance between points A and B
= JR' = ~ x 240 = 22.5 V.
32 = 10 + 7.5 + 10 = 27.5 n
CURRENT ELECTRICITY 3.45
R
Resistance of arm EFG = 10 + 10 = 200 3 3
These two 200 resistances are in parallel. Current,
e 11
I = - = -- = 3 A.
R 11/3
:. Effective resistance between points E and G
= 20 x20 = 100 3.19 INTERNAL RESISTANCE OF A CELL
20+ 20
SI
34. What is internal resistance of a cell ? On what
Hence total resistance between points A and C factors does it depend?
= 10 + 10 + 10 = 30 O. Internal resistance. When the terminals of a cell are
26. The resistances of 40, 100 and 40 are in series. connected by a wire, an electric current flows in the
Their equivalent resistance = 18 o. This is in parallel wire from positive terminal of the cell towards the
with 90 resistance. negative terminal. But inside the electrolyte of the cell,
Equivalent resistance between P and Q, the positive ions flow from the lower to the higher
18 x9 potential (or negative ions from the higher to the lower
R=--=60
IT
18 + 9 potential) against the background of other ions and
neutral atoms of the electrolyte. So the electrolyte offers
P.D. between P and Q = IR = 2.4 x 6 = 14.4 V
some resistance to the flow of current inside the cell.
12 = V = 14.4 = 1.6 A The resistance offered by the electrolyte of a cell to the
R 9
flow of current between its electrodes is called internal
and II = 2.4 - 1.6 = 0.8 A. resistance of the cell.
H
27. P.D. across 60 = P.D. across 30 The internal resistance of a cell depends on following
6 x 0.5 = 3 x 12 factors:
Current through Z, 1. Nature of the electrolyte.
12 =1.0 A 2. It is directly proportional to the concentration of
Current through X = 0.5 + 1.0 = 1.5 A the electrolyte.
O
Terminal potential difference. The potential drop Thus the potential difference across the terminals of the
across the terminals of a cell when a current is being drawn cell is equal to its emf when no current is being drawn from
from it is called its terminal potential difference (V). the cell.
Relation between r, e and V.Consider a cell of emf e (ii) A real cell has always some internal resistance r,
and internal resistance r connected to an external so when current is being drawn from cell, we have
resistance R, as shown in Fig. 3.90. Suppose a constant v<e
current I flows through this circuit.
...... Thus the potential difference across the terminals of the
cell in a closed circuit is always less than its emf
R
Characteristic curves for a cell. When a cell of emf e
and internal resistance r is connected across a variable
R
load resistance R, its functioning can be represented by
: Ie YY
the following three graphs :
----------
Cell
(i) e versus R graph. The emf of a cell is equal to the
Fig. 3.90 Cellof emf e and internal resistance r. terminal p.d. of the cell when no current is drawn from
it. Hence emf e is independent of Rand e-R graph is a
SI
By definition of emf,
straight line, as shown in Fig. 3.91(a)
e = Work done by the cell in carrying a unit charge
along the closed circuit
= Work done in carrying a unit charge from A to B jl------
00,
The terminal p.d. of the cell that sends current I Hence V-R graph is as shown in Fig. 3.91(b).
through the external resistance R is given by
(iii) V versus I graph. As V =e - Ir
V=IR=~ ~ V=-rI+e <=:> y=mx+c
R+r Hence, the graph between V and I is a straight line
M
R
eR circuit shown in Fig. 3.93,
3. Terminal p.d. of a cell, V = IR = -- the voltmeter reads 1.5 V,
R+r
when the key is open. When
4. Terminal p.d. when a current is being drawn from the key is dosed, the
the cell, voltmeter reads 1.35 V and R
v =e-
SI
Ir ammeter reads 1.5 A Find
5. Terminal p.d. when the cell is being charged, the emf and the internal Fig. 3.93
V =e+ Ir resistance of the cell.
- V] Solution. When the key is open, the voltmeter
6. Internal resistance of a cell, r = R
[e---v- reads almost the emf of the cell.
e = 1.5 V
Units Used
When the key is closed, voltmeter reads the
EMF e and terminal p.d. V are in volt (V), internal terminal potential difference V.
resistance r and external resistance R in n and
IT
current I in ampere (A). V = 1.35 V, I = 1.5 A, r=?
r = e - V = 1.5 -1.35 = 0.1 Q.
Example 73. For driving a current of 3 Afor 5 minutes in
I 1.5
an electric circuit, 900 J of work is to be done. Find the emf of
the source in the circuit. Example 76. A cell of emf 2 V and internal resistance
0.1 Q is connected to a 3.9 Q external resistance. What will
H
Solution. The amount of charge that flows 'through
the circuit in 5 minutes is be the p.d. across the terminals of the cell? [CBSE D OlC]
and percentage error in the reading of the voltmeter. 2.2 V. When the terminals of the + -
Solution. Here e = 2 V, r = 2 Q cell are also connected to a
Cell resistance of 5 Q, the voltmeter
Resistance of voltmeter,
:2V reading drops to 1.8 V. Find the
R =998Q R=5Q K
I
internal resistance of the cell.
Current in the circuit is 2Q : [CBSEOD 10]
..».
Fig. 3.94
Voltmeter Solution. Here e =2.2 V, R = 5 Q, V = 1.8 V
R+r
Internal resistance,
2V
R=998Q
(998 + 2)Q
Fig. 3.92
r = R (e; VJ = 5 C.21~81.8)Q = 1.1 Q.
=2 x 10-3 A
3.48 PHYSICS-XII
Example 78. A dry cell of emf 1.6 V and internal resis- Solution. (a) Here e = 12 V, I = 90 A,
tance 0.10 n is connected to a resistance of R ohm. The
r = 5.0 x 1O-2n
current drawn from the cell is 2.0 A Find the voltage drop
across R. .'. Terminal voltage,
Solution. Here e = 1.6 V, r = 0.10 n, 1=2.0 A V= e - Ir = 12 - 4.5 = 7.5 V.
(b) The maximum current can be drawn from a
Voltage drop across R will be
battery by shorting it.
V =e - Ir
Then V =0
= 1.6 - 2.0 x 0.10 = 1.4 V.
Example 79. A battery of e.m.f 't', and internal resistance and I
e
=- =-
12
A = 24 mA
max r 500
R
'r', gives a current of 0.5 A with an external resistor of
12 ohm and a current of 0.25 A with an external resistor of Clearly, the battery is useless for starting the car
25 ohm. Calculate (i) internal resistance of the cell and and must be charged again.
(ii) emf of the cell. [CBSE D 02; OD 13C] (c) During discharge of the accumulator, the current
Solution. EMF of the cell, e = I (R + r) inside the cells (of the accumulator) is opposite to what
SI
it is when the accumulator discharges. That is, during
In first case, e = 0.5 (12 + r) charging, current flows from the + ve to - ve terminal
in second case, e = 0.25 (25 + r) inside the cells. Consequently, during charging
0.5 (12 + r) = 0.25 (25 + r) V = e + Ir
On solving, we get r = 1n Hence V must be greater than 12 V during charging.
Hence e = 0.5 (12 + 1) = 6.5 V.Example 82. A battery of emf 12.0 V and internal resis-
Example 80. A battery of emf 3 volt and internal resis- tance 0.5 n is to be charged by a battery charger which
tance r is connected in series with a resistor of 55 n through supplies 110 V d.c. How much resistance must be connected
IT
an ammeter of resistance 1 n. The ammeter reads 50 mA. in series with the battery to limit the charging current to
Draw the circuit diagram and calculate the value of r. 5.0 A ? What will be the p.d. across the terminals of the
[CBSE D 95 ; Haryana 02] battery during charging ?
Solution. Total resistance Solution. For charging, the positive terminal of the
charger is connected to the positive terminal of the battery.
= 55 + 1 + r n = 56 + r n
Net emf e' = 110 -12.0 = 98 V
H
Current 55Q
Battery charger
=50mA .------f 110 V t-----,
R
= 50 x 10-3 A
5.0A
emf r= SV r------------ ..
: 12.0 V 0.5 Q: R
O
.
R esistance = ---
emf ,
,
,
, IIII -
'-----',--'-1+ ,
Current
, 1
,
'
:
J
r = 60 - 56 = 4 n. will be
Example 81. (a) A car has a fresh storage battery of emf I=~=~A
12 V and internal resistance 5.0 x 10-2 n. If the starter R+ r R + 0.5
motor draws a current of90 A, what is the terminal voltage Given 1= 5.0 A, therefore
of the battery when the starter is on ?
(b) After long use, the internal resistance of the storage ~ = 5.0 or R =19.1n
R +0.5
battery increases to 500 n. What maximum current can be
drawn from the battery ? Assume the emf of the battery to The terminal p.d. of the battery during charging is
remain unchanged. V = e + Ir = 12.0 + 5.0 x 0.5 = 14.5 V
(c) If the discharged battery is charged by an external emf If the series resistor R were not included in the
source, is the terminal voltage of the battery during charging charging circuit, the charging current would be 98/0.5
greater or less than its emf12 V ? [NCERT] = 196 A, which is dangerously high.
CURRENT ELECTRICITY 3.49
Example 83. A cell of emf 1.5 V and internal resistance Solution. (I) The potential difference corresponding
0.5 Q is connected to a (non-linear) conductor whose V-I to zero current equals the emf of the cell.
graph is shown in Fig. 3.97(a). Obtain graphically the :. EMF of the cell, E. = 1.4 V.
current drawn from the cell and its terminal voltage.
(ii) Maximum current is drawn from the cell when
I I the terminal potential difference is zero.
Imax = 0.28 A.
(iii) Internal resistance,
E. 1.4 V
r=--=--=SQ.
Imax 0.28 A
R
Example 85. Find the current drawn from a cell of emf 1 V
and internal resistance 2 /3 Q connected to the network
given below. [CBSE D OlC)
Fig. 3.97
SI
Solution. Here E. = 1.5 V, r = 0.5 Q
Terminal voltage of the cell, V = E. - Ir
For different currents, terminal voltages can be
determined as follows: D c
I=O, V=1.5-0=1.5V
I = 1A, V = 1.5 -1 x 0.5 = 1.0 V
I = 2 A, V = 1.5 - 2 x 0.5 = 0.5 V
IT
Fig. 3.99
I = 3 A, V = 1.5 - 3 x 0.5 =0
The V-I graph for the cell is a straight line AB, as Solution. The equivalent circuit is shown below.
shown in Fig. 3.97(b). This straight line graph intersects Hl
A B
the given non-linear V-I graph at current = 1 A and at
voltage = 1 V. Hl In
=1A
H
:. Current drawn from the cell p
Terminal voltage of the cell = 1 V.
IV
straight line ABC, as shown in Fig. 3.98. Determine from
the graph
(i) emf of the cell Fig. 3.100
(ii) maximum current obtained from the cell, and
Resistance in arm AB = 1Q
(iii) internal resistance of the cell. [CBSE D 1IC]
M
lxl lxl 1 1
2.0
Resistance in arm PQ = -- + -- = - + - = 1Q.
1+1 1+1 2 2
1.6 Resistance in arm DC = 1Q
t
<. A These three resistances are connected in parallel.
% 1.2
~
I
I Their equivalent resistance R is given by
I
:::. 0.8 ---+------- 1 1 1 1 3 1
I
I
- = - + - + - =- or R =- Q
I I R 1 1 1 1 3
0.4 ---T-------r-------
I
I
I
I
I
I Current drawn from the cell,
I I I
Example 86. A uniform wire of resistance 120 is cut into of the cell (ii) value of R (iii) internal resistance of
three pieces in the ratio 1: 2 :3 and the three pieces are the cell. [Ans. (i) 1.53 V (ii) 1.030 (iii) 0.50OJ
connected to form a triangle. A cell of emf 8 V and internal
resistance 10 is connected across the highest of the three
resistors. Calculate the current through each part of the
circuit. [CBSE OD 13C]
Solution. In Fig. 3.101, RAB = 2 0, RBC = 4 0 and 5
RAC =60.
B Fig. 3.102
R
battery of emf 6.0 V and internal resistance 10
drops to 5.8 V when connected across an external
resistor. Find the resistance of the external resistor.
(Ans.290)
SI
BV r=10 6. The potential difference between the terminals of a
6.0 V battery is 7.2 V when it is being charged by a
current of 2.0 A. What is the internal resistance of
Fig. 3.101 the battery? (Ans. 0.60)
7. A battery of emf 2 V and internal resistance 0.50 is
The series combination of 20 and 40 (of equi-
connected across a resistance of 9.50. How many
valent resistance 60) is in parallel with the 60
electrons pass through a cross-section of the
resistance. The equivalent resistance is
resistance in 1 second? (Ans. 1.25 x 1018)
R=6x6=30 8. A cell of emf E and internal resistance r is connected
IT
6+6 across a variable load resistor R It is found that
when R = 40, the current is 1A and when R is
Current, I =_E_ = 8 V =2 A
R + r (3 + 1)0
increased to 90, the current reduces to 0.5 A. Find
the values of the emf E and internal resistance r.
The resistances RBAC and RBC of the parallel [CBSE D 15]
branches are equal. (Ans. 5 V, 10)
H
.. IABC = lAC = 1 A. 9. The emf of a battery is 4.0 V and its internal resis-
tance is 1.5 O. Its potential difference is measured
~roblems For Practice by a voltmeter of resistance 1000 O. Calculate the
percentage error in the reading of emf shown by
voltmeter. (Ans. 0.15 %)
O
when a current of 1.0A is drawn. Find the emf and the (Ans. 2 A, 4.8 V)
internal resistance of the cell. (Ans. 2.0 V, 0.40)
11. The two poles of a cell of emf 1.5 V are connected to
3. The potential difference of a cell in an open circuit the two ends of a 100 coil. If the current in the
is 6 V, which falls to 4 V when a current of 2 A is circuit is 0.1A, calculate the internal resistance of
drawn from the cell. Calculate the emf and the the cell. (Ans. 50)
internal resistance of the cell. (Ans. 6 V, 10)
12. The potential difference across the terminals of a
4. In the circuit shown in Fig. 3.102, the resistance of battery is 8.5 V, when a current of 3 A flows
the ammeter A is negligible and that of the through it from its negative terminal to the positive
voltmeter V is very high. When the switch 5 is open, terminal. When a current of 2 A flows through it in
the reading of voltmeter is 1.53 V. On closing the the opposite direction, the terminal potential
switch 5, the reading of ammeter is 1.00 A and that difference is 11V. Find the internal resistance of the
of the voltmeter drops to 1.03 V. Calculate: (i) emf battery and its emf. (Ans. 0.50, 10 V)
CURRENT ELECTRICITY 3.51
13. In the circuit shown in Fig. 3.103, a potential 12. When current flows through the cell from its
difference of 3 V is required between the points A negative to positive terminal,
and B.Find the value of resistance R,.. (Ans. 30) V = e- Ir
I
or 8.5 = e - 3r ...(i)
When current flows through the cell from its positive
to negative terminal, p.d. across r adds to its emf. So
12 V, 1 Q
-'-
V = e+ Ir
or 11=e+2r ...(ii)
On solving equations (i) and (ii), we get
r = 0.50 and = 10 V. e
R
13. Current in the main circuit,
Fig. 3.103
1= e
R,.+Rz+r
HINTS
Since a potential difference of 3 V is required across
- VJ = 14 (1.5 1.4
e----v-
SI
-1.4) R,., therefore
1. r= R
( =10.
IR,. = 3 volt
2. EMF of a cell, e = V + Ir eR,. = 3 or _12_R,.-'--.= 3
or
When 1= 0.5 A, V = 1.8 V .. e = 18 + 0.5 r (1) R,.+Rz+r R,.+8+1
When I = 1.0 A, V = 1.6 V .. e = 1.6 + 1.0 r (2)
or R,. = 30.
Solving (1) and (2), we get
e = 2.0 V and r = 0.4 O.
3.21 COMBINATIONS OF CELLS IN SERIES
e = P.D. measured
IT
3. in open circuit = 6 V
AND PARALLEL
e-
V 6-4
r=--=--=ln. 36. Why do we often use a combination of cells ?
I 2
4. (i) e = P.D. measured in open circuit = 1.53 V.
Combination of cells. A single cell provides a feeble
current. In order to get a higher current in a circuit, we
(II..) R ----V - 1.03 -.
- 1 03 0 •
often use a combination of cells, two or more cells. A
H
I 1.00 combination of cells is called a battery. The battery cells
(iii) r = R (e - VJ
V
= 1.03 (1.53 - 1.03) = 0.50 O.
1.03
are used in torches, transistor sets, automobiles, etc.
Cells can be joined in series, parallel or in a mixed way.
37. What do you mean by a series combination of
5. R= /V = lx5.8 = 5.8 =290. cells ? Two cells of different emfs and internal resis-
O
c- V 6.0 - 5.8 0.2 tances are connected in series. Find expressions for the
6. During charging, V = e + Ir equivalent emf and equivalent internal resistance of the
:. 7.2=6.0+ 2xr or r=0.60. combination.
7. 1=--=
e 2
=0.2A
Cells in series. When the negative terminal of one cell is
R + r 9.5 + 0.5 connected to the positive terminal of the other cell and so on,
M
10. Current, I = --
e =
6
= 2 A.
R+ r 2.4+ 0.6
P.D. between the two terminals of the battery is
V = IR"= 2 x 2.4 V = 4.8 V.
R
or VAC =(e1 +e2)-I(r1 +r2) As the two cells are connected in parallel between
the same two points 1\ and ~, the potential difference
If we wish to replace the series combination by a
V across both cells must be same.
single cell of emf s;and internal resistance ~q' then
The potential difference between the terminals of
VAC = s: - I~q first cell is
SI
Comparing the last two equations, we get
V = V~ - VB2 = e1 - I1r1
eeq = e1 + e2 and ~q = r1 + r2 e -
1 __ V
I1 __
-
We can extend the above rule to a series combi- r1
nation of any number of cells:
The potential difference between the terminals of
1. The equivalent emf of a series combination of n
cells is equal to the sum of their individual emfs.
e2 is
eeq = e1 + e2 + e3 + + ell
IT
2. The equivalent internal resistance of a series
combination of n cells is equal to the sum of
their individual internal resistances.
Hence
req=r+r2+r3+ +r"
e
3. The above expression for eq is valid when the n
H
cells assist each other i.e., the current leaves each
cell from the positive terminal. However, if one
e
cell of emf 2, say, is turned around 'in
opposition' to other cells, then or
or
38. What do you mean by a parallel combination of
cells? Two cells of different emfs and internal resistances
are connected in parallel with one another. Find the If we wish to replace the parallel combination by a
expressions for the equivalent emf and equivalent internal single cell of emf eeq and internal resistance ~q ,then
resistance of the combination.
M
V=eeq-I~q
Cells in parallel. When the positive terminals of all cells Comparing the last two equations, we get
are connected to one point and all their negative terminals to
another point, the cells are said to be connected in parallel.
e = e1r2 + er
2 1 and r = r1r2
eq r+1: eq r+r.
1 2 1 2
As shown in Fig. 3.105, suppose two cells of emfs
e 1 and e
2 and internal resistances r1 and r2 are connec- We can express the above results in a simpler way
ted in parallel between two points. Suppose the as follows:
currents Il and I2 from the positive terminals of the
two cells flow towards the junction B1, and current I
eeq _
---+-1
e e2
flows out. Since as much charge flows in as flows out, ~q r1 r2
we have 1 1 1
and -=-+-
~q r1 r2
CURRENT ELECTRICITY 3.53
For a parallel combination of n cells, we can write 40. Derive the condition for obtaining maximum
current through an external resistance connected to a
eeq _
---+-+1 e e2 +-
en parallel combination of cells.
'eq r1 r2 rn Condition for maximum current from a parallel
1 1 1 1 combination of cells. As shown in Fig. 3.108,suppose m
and -=-+-+ +-. cells each of emf e and internal resistance r be connec-
'eq r1 r2 ~J
ted in parallel between points A and B.Let R be the
39. Derive the condition for obtaining maximum external resistance.
current through an external resistance connected across a
series combination of cells.
R
Condition for maximum current from a series A
combination of cells. As shown in Fig. 3.106,suppose n
e
similar cells each of emf and internal resistance r be
connected in series. Let R be the external resistance.
R
SI
Fig.3.108 A parallel combination of m cells.
circuit I I
= r +r +r + n terms =nr
=R+ R' = R +!-.-
m
R
Total resistance in the circuit = R + nr
H
As the only effect of joining
The current in the circuit is m cells in parallel is to get a
single cell of larger size with Fig. 3.109 Equivalent circuit.
1= Total emf
the same chemical materials, so
Total resistance
total emf of parallel combination
O
ne R
= emf due to single cell =e
R + nr The current in the circuit is
Fig. 3.107 Equivalent
Special Cases circuit. 1=---
e me
R + rim mR+r
(i) If R » nr, then
M
Special Cases
1=ne
R (i) If R « !-.- , then
m
= n timesthe current (e I R) that can
be drawn from one cell. 1=me = m times the current due to a single cell.
r
(ii) If R « nr, then
(ii) If R » !-.- , then
1=ne =~ m
nr r
I =~ = the current given by a single cell.
= the current given by a single cell R
Thus, when external resistance is much higher than the Thus, when external resistance is much smaller than the
total internal resistance, the cells should be connected in net internal resistance, the cells should be connected in
series to get maximum current. parallel to get maximum current.
3.54 PHYSICS-XII
R
m
or External resistance
= Total internal resistance of the cells.
A,:-.q:-- m rows ~...;;, Thus, in a mixed grouping of cells, the current through
the external resistance will be maximum if the external
SI
I~I------------~ resistance is equal to the total internal resistance of the cells.
I~I------------~ Examples based on
R 0-- -_ •.. Grouping of Cells
Formulae Used
Fig.3.110 Mixed grouping of ceUs.
1. For n cells in series, 1=--
ne
R+ nr
IT
Total number of cells
=mn 2. For n cells in parallel, I = --
ne
Net emf of each row of n nR+ r
cells in series = ne mne
3. For mixed grouping, I = ---
As m such rows are R mR+ nr
connected in parallel, so net
H
where n = no. of cells in series in one row,
emf of the combination = ne Fig. 3.111 Equivalent circuit.
m = no. of rows of cells in parallel.
Net internal resistance of 4. For maximum current, the external resistance
each row of n cells = nr must be equal to the total internal resistance.
As m such rows are connected in parallel, so the
i.e., R
O
nT =
total internal resistance t' of the combination is given
m
by
or nr = mR
1 1 11m
- =- + - + - + m terms =-
r nr nr nr nr Units Used
=R+r=R+nr
m Example 87. (a) Three cells of emf2.0 V, 1.8 V and 1.5 V
The current through the external resistance R, are connected in series. Their internal resistances are
1= Total emf ne 0.0512,0.712 and In respectively. If the battery is connec-
Total resistance R + nr / m
ted to an external resistor of 412 via a very low resistance
ammeter, what would be the reading in the ammeter ?
mne
(b) If the three cells above were joined in parallel, would
mR+ nr they be characterised by a definite emf and internal
Clearly, the current I will be maximum if the resistance (independent of external circuit) ? If not, how will
denominator i.e., (mR + nr) is minimum. you obtain currents in different branches? [NCERT]
CURRENT ELECTRICITY 3.55
Solution. (a) The circuit diagram is shown in Resistance of external circuit = Total resistance of
Fig. 3.112. two resistances of 17 n connected in parallel
r----------, ~---------I r----------,
0.05 Q I: 0.7 Q I: 1Q or R= Rl~ = 17x 17 n =8.5n
I ~~I-'V\J'\r-""""" Rl + ~ 17 + 17
y- - - - - -: ~!.§-~ - - - --
~~-~------: ~!.~ 17Q
I
Fig. 3.112
R
As the three cells have been connected in series to
an external resistor of 4 n, therefore
Total emf = (2.0 + 1.8 + 1.5) V = 5.3 V Fig. 3.113
Total resistance
SI
Let l' be the total internal resistance of the two cells.
= (0.05 + 0.7 + 1 + 4) n = 5.75 n
Then
Current, 1= emf
resistance
= 5.3 A=O.92A.
5.75
As the two cells of internal resistance r n each have
(b) No, there is no formula for emf and internal been connected in parallel, therefore.
resistance of non-similar cells, joined in parallel. For
1 1 1 1 2
this situation, we must use Kirchhoff's laws. -=-+- or -=-
IT
l' r r 0.6 r
Example 88. A cell of emf 1.1 V and internal resistance
0.5 n is connected to a wire of resistance 0.5 n. Another cell
or r = 0.6 x 2 = 1.2 n.
Example 90. Four identical cells, each of emf 2 V, are
of the same emf is connected in series but the current in the
wire remains the same. Find the internal resistance of the joined in parallel providing supply of current to external
second cell. circuit consisting of two 15 n resistors joined in parallel.
The terminal voltage of the cells, as read by an ideal
H
Solution. In first case:
voltmeter is 1.6 volt. Calculate the internal resistance of each
Total emf, Eo = 1.1 V
cell. [CBSE 0 02]
Total resistance, R = 0.5 + 0.5 = 1 n Solution. As shown in Fig. 3.114, four cell are
:. Current, I=! = 1.1 = 1.1 A connected in parallel to the parallel combination of two
R 1 15 n resistors.
O
In second case :
Total emf,
Eo == 1.1 + 1.1 =2.2 V
Total resistance,
M
R = 0.5 + 0.5 + r = (1 + r) n
lSQ
where r is the internal resistance of the second cell.
2.2
C urrent, 1=--=1.1 or r=ln.
l+r
Example 89. Two identical cells of emf 1.5 Veach joined Fig. 3.114
in parallel provide supply to an external circuit consisting of
Here Eo =2 V, V =1.6 V
two resistances of17 n each joined in parallel. A very high
resistance voltmeter reads the terminal voltage of cells to be The external resistance provided. by two 15 n
1.4 V. Calculate the internal resistance of each cell. resistors connected in parallel is
[eBSE 0 9SC] 15 xIS
R=--=7.5n
Solution. Here Eo =1.5 V, V =1.4 V 15 + 15
3.56 PHYSICS-XII
If r' is the total internal resistance of the four cells This emf sends circuit I in the anti clockwise
connected in parallel, then direction.
l' = R (e -
V
VJ =7.5 (2 1.6
-1.6) = 15 n
8
Total resistance = R + r1 + r2 = 5 + 1 + 2 = 8 n
Current in the circuit
If r is the internal resistance of each cell, then Net emf 2
1 1 1 1 1 4 ----- = - = 0.25 A.
-=-+-+-+-=- Total resistance 8
l' r r r r r
(i) Current inside the cell e2 flows from -ve to +ve
15
or r = 41' = 4 x - = 7.5 n. terminal, so the terminal p.d. of this cell is
8
Va - Vb = e2 - Ir2
Example 91. In the circuit
R
= 4.0 -0.25 x 2.0 = 3.5 V.
diagram given in Fig. 3.115,
the cells £1 and £2 have emfs (ii) Current inside the cell e1 flows from +ve to -ve
4 V and 8 V and internal terminal. Hence the terminal p.d. of this cell is
resistances 0.5 n and 1.0 n 4.50 Va - Vc e
= 1 + Ir1
respectively. Calculate' the =2.0 + 0.25 x 1.0 = 2.25 V.
SI
current in each resistance. 60
[CBSE DISC] Example 93. In the two electric circuits shown in
Fig. 3.115 Fig. 3.117, determine the readings of ideal ammeter (A) and
Solution. Effective emf of the circuit the ideal voltmeter (V). [CBSE DISC]
= e2 - e1 =8 - 4 = 4 V
Total resistance of the circuit
3x6
= 1 + 0.5 + 4.5 n + -- =8 n
3+6
IT
:. Current in the circuit, I = ~ = 0.5 A
6
2n
Example 92. In Fig. 3.116, e1 and e2 are respectively
As the current I flows from -ve to +ve terminal
2.0 V and 4.0 V and the resistances r1, r2 and Rare
inside the cell of 6 V, the terminal p.d. of the cells is
respectively 1.0 n, 2.0 nand 5.0 n. Calculate the current in
the circuit. Also calculate (i) potential difference between the V = e-Ir=6-7.5x 1 =-1.5 V
M
points b and a, (ii) potential difference between a and c. :. Reading of ammeter = 7.5 A,
:------e---: :-e-------: Reading of voltmeter = -1.5 V.
c I lL~ 2 I b
I ~ I
In the circuit (b)
: Y1 :: Y2:
---------- ---------- Net emf =9-6 =3V, Total resistance =2n
3V
Current, I = - = 1.5 A
R 2n
As the current I flows from +ve to -ve terminal
Fig. 3.116 inside the 6 V cell, so the terminal p.d. of the cell is
Solution. As emfs e1 and e2 are opposing each V = e + Ir = 6 + 1.5 x 1 = 7.5 V
other and e2 > e1, so Reading of ammeter = 1.5 A,
Netemf=e2 -e1 =4-2 =2 V. Reading of voltmeter = 7.5 V.
CURRENT ELECTRICITY 3.57
Example 94. A network of resistances is connected to a Example 95. A 20 V battery of internal resistance 10 is
16 V battery with internal resistance of 10, as shown in connected to three coils of 12 0,6 0 and 4 0 in parallel, a
Fig. 3.118. resistor of5 0 and a reversed battery (emf = 8 V and internal
(a) Compute equivalent resistance of the network, resistance = 2 0), as shown in Fig. 3.119. Calculate the
(b) Obtain the current in each resistor, and current in each resistor and the terminal potential difference
across each battery. [CBSE 00 OlC]
(c) Obtain the voltage drops VAB, VBC and Vco'
[NCERT ; CBSE F 10]
10 20V
~---------I
40 120 r--l __ -!:--'\.fV'v~ 111-+:-"---'
I 10
R
B
50 c:
N
,....;
16 V
r---------.
.- 8V, 20
--------~
SI
1 --_
,
••
Fig. 3.119
Fig. 3.118
Solution. Equivalent resistance R' of 12 0,60,40
Solution. (a) As the two 4 0 resistances are in resistances connected in parallel is given by
parallel, their equivalent resistance is 1 1 1 1 6 1
-=-+-+-=-=-
R=4x4=20 R' 12 6 4 12 2
1 4+4
.. or R'=20
Also, the 12 0 and 6 0 resistances are in parallel,
IT
Total resistance = 1 + 5 + 2 + 2 = 10 0
their equivalent resistance is
Net emf =20 -8 =12
~ = 12 x 6 =40
12 + 6 Current in the circuit, 1= 12 = 1.2 A
10
Now the resistances R1, ~ and 10 are in series. So the current through each battery and 5 0 resistor
Hence the equivalent resistance of the network is is 1.2 A.
H
R = Rl + ~ + 1 =2 + 4 + 1 = 70. P.D. across the parallel combination of three
(b) The total current in the circuit is resistors is
I=_e_=~=2A V' = IR' =1.2 x 2 =2.4 V
R+r 7+1
Current in 2 0 coil = 2.4 = 0.2 A
O
II = 12 = 1 A 4
The potential difference between C and D is Terminal p.d. across 20 V battery,
or 13 + 213 =2 A
V' = e' + l r =8 + 1.2 x 2 = 10.4 V.
2 4 Example 96.36 cells each of internal resistance 0.5 0 and
13 ="3A and 14=-A
3 emfl.5 Veach are used to send current through an external
(c) VAB = 4 x II = 4 x 1 = 4 V, circuit of 2 0 resistance. Find the best mode of grouping
them and the current through the external circuit.
VBC = 1 x' I = 1 x 2 = 2 V,
2
Solution. Here e = 1.5 V, r = 0.5 0, R =2 0
Vco = 12 x 13= 12 x 3" = 8 V. Total number of cells, mn = 36 ...(1)
3.58 PHYSICS-XII
For maximum current in the mixed grouping, Solution. In the steady state (when the capacitor is
nr = R or n x 0.5 = 2 .... (2) fully charged), no current flows through the branch BE.
m m Net emf =2 V - V = V
Multiplying equations (1) and (2), we get Net resistance = 2 R + R = 3 R
0.5n2=72 or n2=i44 . I =- V
. th e CIrCUIt,
.
:. Current In
n = 12 and m = 36 = 3 3R
12 Potential difference across BE
Thus for maximum current there should be three V 4
=2V -Ix2R =2V --x2R =- V
rows in parallel, each containing 12 cells in series. 3R 3
:. Maximum current ±3
(i) Potential difference across C = V - V = V .
R
3
mne 36 x 1.5 = 4.0 A.
mR + nr 3 x 2 + 12 x 0.5 (ii) Charge on the capacitor, Q = Cx V = CV .
3 3
Example 97. 12 cells, each of emf 1.5 V and internal (iii) Energy stored in the capacitor
resistance of 0.5 0, are arranged in m rows each containing
SI
2
n cells connected in series, as shown. Calculate the values of = ~C(V)2 CV
nand m for which this combination would send maximum 2 3 18
current through an external resistance of1.5 O. ~roblem5 For Practice
[CBSESamplePaper 08]
1. Three identical cells, each of emf 2 V and internal
~R=1.5n~
resistance 0.2 n are connected in series to an
external resistor of 7.4 n. Calculate the current in
~~'----~~i
i 1 the circuit. (Ans. 0.75 A)
IT
1 1
1- - _ - - - - - - - - - - - - ~ m rows 2. Three identical cells each of emf 2 V and unknown
1 I'
internal resistance are connected in parallel. This
1 1 \
~ - - - -1111- - - -11- - - - _I
combination is connected to a 5 n resistor. If the
terminal voltage across the cells is 1.5V, what is the
(n cellsin eachrow) internal resistance of each cell? [CBSEOD 99]
Fig. 3.120 (Ans.5n)
H
Solution. For maximum current through the 3. Two cells connected in series have electromotive
external resistance, force of 1.5 Veach. Their internal resistances are
0.5nand 0.25n respectively. This combination is
External resistance connected to a resistance of 2.25n. Calculate the
= Total internal resistance of the cells current flowing in the circuit and the potential
O
7. A set of 4 cells, each of emf 2 V and internal resis- When the cells are s
tance 1.5 n are connected across an external load of connected in
10 n with 2 rows, 2 cells in each branch. Calculate parallel (Fig. 3.123), -JVVv-
the current in each branch and potential difference the current in the re
across 10 n. [Karnataka 91C] circuit is Ip i,
(Ans. 0.175 A, 3.5 V) .J\.N\r
I =_--::e_ r
P rxr
HINTS 1+--
nE. 3x2 6 r + r
R=lQ
1. 1=--= =-=0.75A. 2E.
R + nr 7.4 + 3 x 0.2 8
Fig. 3.123
2+ r
2. Here E. = 2V, V = 1.5 V, R = 5 n
R
If r' is the total internal resistance of the three cells
connected in parallel, then
Given '.>', .. 1+2r 2+r
or 1 + 2r = 2 + r or r = 1 n.
r' = R [E. -V V] = 5 [2 - 1.5] = ~ n
1.5 3 7. The circuit diagram is shown below.
SI
1 1 1 1 3 3
But - = - + - + - or - =- .. r = 5 n.
r' r r r 5 r
-JVVv-
3. Total resistance, R = 0.5 + 0.25 + 2.25 = 3.0 n r
Total emf, E. = 1.5 + 1.5 = 3.0 V e,
Current m . . 1 = -E = -3.0 = lOA
. th e circuit, .
R 3.0
P.D. across first cell,
VI = E - bi = 1.5 - 1.0 x 0.5 = 1.0 V R=lOQ
IT
P.D. across second cell, Fig. 3.124
V2 = E - 1r2 = 1.5 - 1.0 x 0.25 = 1.25 V.
4. For series combination. The current is Here e = 2 V, r = 15 n, R = 10 n, 11 = 2, m =2
lOE ne
1=--=
2 x2 4
=-=0.35A.
---=0.25A nr 2 x 1.5 11.5
59+ 10r R +- 10+--
H
m 2
For parallel combination. The current is
The two branches are identical.
lOE
----=25A :. Current in each branch = 0.35 = 0.175 A.
10 x 0.05 + r
2
On solving the above two equations, Potential difference across R
O
because during the course of their motion, the but with a steady drift velocity. This is because of the
electrons collide frequently with the positive metal collisions of electrons with ions and atoms during the
ions. The kinetic energy gained by the electrons during course of their motion. The kinetic energy gained by
the intervals of free acceleration between collisions is the electrons is shared with the metal ions. These ions
transferred to the metal ions at the time of collision. vibrate more vigorously and the conductor gets heated
The metal ions begin to vibrate about their mean up. The amount of energy dissipated as heat in
positions more and more violently. The average kinetic conductor in time t is
energy of the ions increases. This increases the
2
temperature of the conductor. Thus the conductor gets H = VIt joule = 12Rt joule = V t joule
heated due to the flow of current. Obviously, the R
electrical energy supplied by the source of emf is 2 2
H
VI cal = I Rt cal = ~
= _t cal
converted into heat. or
R
4.18 4.18 4.18 R
3.23 HEAT PRODUCED BY ELECTRIC The above equations are known as Joule's law of
CURRENT: JOULE'S LAW heating. According to this law, the heat produced in a resistor
43. Obtain an expression for the heat developed in a is
SI
resistor by the passage of an electric current through it. 1. directly proportional to the square of current for a
Hence state Joule's law of heating. given R,
Heat produced in a resistor. Consider a conductor 2. directly proportional to the resistance R for a given I,
AB of resistance R, shown in Fig. 3.125. A source of emf 3. inversely proportional. to the resistance R for a given
maintains a potential difference V between its ends A V, and
and Band sends a steady current I from A to B. Clearly, 4. directly proportional to the time for which the
VA > VBand the potential difference across AB is current flows through the resistor.
V= VA - VB>0
IT
For Your Knowledge
> The equation: W = VIt is applicable to the conversion of
electrical energy into any other form, but the equation:
H = [2 Rt is applicable only to the conversion of electrical
energy into heat energy in an ohmic resistor.
H
> Joule's law of heating holds good even for a.c. circuits.
Only current and voltage have to be replaced by their
Fig. 3.125 Heat produced in a resistor.
rms values.
The amount of charge that flows from A to B in > If the circuit is purely resistive, the energy expended
time tis by the source entirely appears as heat. But if the
O
R
1 joule 1 joule 1 coulomb V2
1 watt = = x ---- P = - = I2R = VI watt
I second 1 coulomb 1 second R
or 1 watt = 1 volt x 1 ampere Measurement of electric power. To measure the
electric power of an appliance, sayan electric lamp, we
The bigger units of electric power are kilowatt
connect a battery and an ammeter in series with the
SI
(kW) and megawatt (MW).
6
electric lamp and a voltmeter in parallel with it, as
1 kW = 1000 Wand 1 MW = 10 W shown in Fig. 3.126. Suppose the voltmeter reads V
The commercial unit of power is horse power (hp) volts and the ammeter reads I amperes, then power
Ihp=746 w. rating of the electric lamp will be
P = VI watt
3.25 ELECTRIC ENERGY
45. Define the term electric energy. State its 51 and
commercial units.
IT
Electric energy. The total work done (or the energy
supplied) by the source of emf in maintaining an electric
current in a circuit for a given time is called electric energy
consumed in the circuit. It depends upon the power of
the appliance used in the circuit and the time for which
this power is maintained.
H
Fig. 3.126 To measure electric power of
Electric energy,
an electric lamp.
W = P. t = VIt joule = 12Rt joule
The SI unit of electric energy is joule (J). 3.27 POWER CONSUMPTION IN A
1 joule = 1 volt x 1 ampere x 1 second COMBINATION OF APPLIANCES
O
= 1watt x 1second 47. Prove that the reciprocal of the total power con-
Commercial unit of electric energy. The commercial sumed by a series combination of appliances is equal to the
unit of electric. energy is kilowatt hour or Board of sum of the reciprocals of the individual powers of the
Trade (B.O.T.) unit. One kilowatt hour is defined as the appliances.
electric energy consumed by an appliance ofl kilowatt in one Power consumed by a series combination of
M
The resistances of the three bulbs will be The resistances of the three bulbs will be
V2
V2
V2
V2 V2 V2
RI=p:'
1
~=P:'2 ~=P:3 RI=p:'
123
~=P:' ~=p:
As the bulbs are connected in series, so their As the bulbs are connected in parallel, their
equivalent resistance is effective resistance R is given by
R=RI+~+~ 1 1 1 1
-=-+-+-
If P is the effective power of the combination, then R RI ~ ~
V2 V2 V2 V2 Multiplying both sides by V2, we get
-=-+-+-
R
P PI P2 P3 V2 V2 V2 V2
1 1 1 1 -=-+-+-
or -=-+-+- R RI ~ ~
PPI P2 P3
or P= PI+ P2+ P3
Thus for a series combination of appliances, the
Thus for a parallel combination of appliances, the effective
SI
reciprocal of the effective power is equal to the sum of the power is equal to the sum of the powers of the individual
reciprocals of the individual powers of the appliances. appliances.
Clearly, when N bulbs of same power Pare If N bulbs, each of power P, are connected in
connected in series, parallel, then
P
Peff = NP
Peff = N
The brightness of the three bulbs will be
As the bulbs are connected in series, the current I
through each bulb will be same. V2 V2 V2
IT
PI=~,P2= ~,P3= ~
1= V
Rl+~+~ As the resistance of the highest wattage (power)
bulb is minimum, it will glow with maximum bright-
The brightness of the three bulbs will be
ness. If the current in the circuit exceeds the safety
2R 2R
P"-I
1- I'
p'=PR
2 "2'
P'=I
3 "3 limit, the bulb with maximum wattage will be fused
H
first. For this reason, the appliances in houses are
As R ex ~ , the bulb of lowest wattage (power) will
P connected in parallel.
have maximum resistance and it will glow with maxi-
mum brightness. When the current in the circuit exceeds 3.28 EFFICIENCY OF A SOURCE OF EMF
the safety limit, the bulb of lowest wattage will be 49. Define efficiency of a source of emf Write an
O
appliances. As shown in Fig. 3.128, consider a parallel Then its efficiency will be
combination of three bulbs of powers PI'P2and P3,
Output power VI V IR
which have been manufactured for working on the 11
=
same voltage V. Input power el -- E I (R + r)
R
or 11=--
R+r
50. (a) A battery of emfe and internal resistance r is
connected across a pure resistive device (e.g., an electric
heater or an electric bulb) of resistance R. Show that the
power output of the device is maximum when there is a
~-----oVo-----~ perfect 'matching' between the external resistance and the
source resistance (i.e., where R = r). Determine the
Fig. 3.128 Parallel combination of bulbs. maximum power output.
CURRENT ELECTRICITY 3.63
R
.'. Current through the expression for the efficiency of an electric motor.
device, Efficiency of an electric device. The efficienClJof an
Total emf electric device is defined as the ratio of the output power to
1=----- the input power
Total resistance
Output power
SI
11= --"---"---
Input power
R+r Fig. 3.129 For an electric motor, we can write
.'. Power output of the resistive device will be Output mechanical power
11= --"--------=---
Input electric power
P = I 2R =(_e_J2 R
R+r Here, input electrical power
= Output mechanical power + Power lost as heat
eR
2
eR
2
IT
···(0 53. (a) An electric motor runs on a d.c. source of emfe
(R+r)2 (R-r)2+4Rr
and internal resistance r. Show that the power output of
Obviously, the power output will be maximum the source is maximum when the current drawn by the
when motor is e /2r.
R- r=0 or R=r (b) Show that power output of electric motor is maximum
Thus, the power output of the device is maximum when the back emf is one-half the source emf provided the
H
when there is a perfect matching between the external resistance of the windings of the motor is negligible.
resistance and the resistance of the source, i.e., when (c) Compare and contrast carefully the situation in this
R = r. This proves maximum power theorem. exercise with that in Q.SO(a) above. [NCERT)
Maximum power output of the source is (a) Output power from a source connected to an
O
(~r
Power dissipation inside the battery
Hence the power output of the source is maximum when
= I2r = r= er
2
.
the current drawn by the motor is /2 r. e
(b) Here, emf of source =e
51. Show that the efficiency of a battery when Internal resistance of source =r
delivering maximum power is only 50%. Back emf of motor = e'
Maximum efficiency of a source of emf. For a Resistance of motor =R"'O
source of emf,
As the external resistance R is negligible, therefore
Input power = eI . th . .
current ill e circuit = --
e-e'
.
Output power = VI r
3.64 PHYSICS-XII
And power output of the motor of it appears as light. A bulb gives nearly 1 candela of
= Power output of the source light energy for the consumption of every watt of
=eI - Pr electric power.
From part (a), this is maximum when 3. Electric fuse. It is a safety device used to protect
electrical appliances from strong currents. A fuse wire
1=-
s or
e -e' e or e' =~ must have high resistivity and low melting point. It is
2r r 2r 2 usually made from an alloy of tin (63%) and lead
(37%). It is put in series with the live wire of the circuit.
Hence the power output of electric motor is maximum
When the current exceeds the safety limit, the fuse
when the back emf is.one-half the source emf
wire melts and breaks the circuit. The electric
(c) The condition in Q. 50(a) is for a passive resistor installations are thus saved from getting damaged.
R
in which the entire electric energy is converted into
The fuse wire of suitable current rating (1 A, 2 A,
heat while the condition in Q. 53(a) is for a non-passive
3 A, 5 A, 10 A etc.) should be used in the circuit
resistor (e.g., electric motor) in which the supplied
depending on the load in the circuit. For example,
electric energy changes partly into heat and partly into
when we use an electric iron of 1 kW electric power
mechanical work. So the former is a special case of the
SI
with electric mains of 220 V, a current of (1000/220) A
latter.
i.e., 4.54 A flows in the circuit. This requires a fuse of
5 A rating.
3.30 APPLICATIONS OF HEATING
EFFECT OF CURRENT 4. Electric arc. It consists of two carbon rods with a
small gap between their pointed ends. When a high
54. Discuss some practical applications of the heating
potential difference of 40 - 60 V is applied between the
effect of current.
two rods, very intense light is emitted by the gap. We
Applications of heating effect of current. Some of know that E = - dV / dr. Clearly, E will be large if the
the important applications of Joule heating are as follows:
IT
gap is small. When the electric field exceeds the
l. Household heating appliances. Many electrical dielectric strength of air, ionisation of air occurs. This
appliances used in daily life are based on the heating causes a big spark to pass across the gap.
effect of current such as room heater, electric toaster, 5. Other devices. Many other devices are based on
electric iron, electric oven, electric kettle, geyser, etc. the heating effect of current such as electric welding,
The designing of these devices requires the selection of thermionic valves, hotwire ammeters and voltmeters.
H
a proper resistor. The resistor should have high 55. Explain why is electric power transmitted at high
resistance so that most of the electric power is voltages and low currents to distant places.
converted into heat. In most of the household heating
High -oltage po 'er transmission. Electric power is
appliances, nichrome element is used because of the
transmitted from power stations to homes and
following reasons :
O
when heated. across load R and I the current through it, then
2. Incandescent electric bulb. It is an important P= VI
application of Joule heating in producing light. It The power wasted in transmission cables is
consists of a filament of fine metallic wire enclosed in a
p2R
glass bulb filled with chemically inactive gases like P. = 12R = __ 1
1 1 V2
nitrogen and argon. The filament material should
have high resistivity and high melting point. Thus the power wasted in the transmission cables
Therefore, tungsten (melting point 33800q is used for is inversely proportional to the square of voltage.
bulb filament. When current is passed through the Hence to minimise the power loss, electric power is
filament, it gets heated. to a high temperature and transmitted to distant places at high voltages and low
emits light. Most of the power consumed by the currents. These voltages are stepped down by
filament is converted into heat and only a small part transformers before supplying to homes and factories.
CURRENT ELECTRICITY 3.65
For Your Knowledge Example 99. An electric current of 4.0 Aflows through a
12 0 resistor. What is the rate at which heat energy is
~ The emission of light by a substance when heated to a produced in the resistor? [NCERT]
high temperature is called incandescence.
Solution. Here I = 4 A, R = 12 0
~ A heater wire is made from a material of large resis-
tivity and high melting point while a fuse wire is made Rate of production of heat energy,
from a material of large resistivity and low melting point P = I R =42
2
x 12 =192 W.
~ The load in an electric circuit refers to the current Example 100. How many electrons flow through the
drawn by the circuit from the supply line. If the filament of a 120 V and 60 W electric lamp per second ?
current in a circuit exceeds the safe value, we say that Given e = 1.6 x 10- 19 C.
the circuit is overloaded.
Solution. Here P =60 W, V =120 V, t =1 s
R
~ The temperature upto which a wire gets heated (i.e.,
steady state temperature 9) is directly proportional to 1= P =~=O.5A
the square of the current and is inversely proportional V 120
to the cube of its radius but is independent of its length. But I=!1. = ne
]2 t t
SI
9ex:-
r3 No. of electrons flowing per second is
~ When the resistances are connected in series, the
current] through each resistance is same. Consequently,
I n=- =
It 0.5 x 1
19 = 3.125 x 10 .
18
e 1.6xlO-
Pex:R (':P=12R)
Example 101. A heating element is marked 210 V,630 W.
and V ex: R (.: ~ = ]R)
What is the current drawn by the element when connected to
Hence in a series combination of resistances, the
a 210 V d.c. mains? What is the resistance of the element?
potential difference, power consumed and hence heat
[NCERT]
produced will be larger in the higher resistance.
IT
Solution. Here P =630 W, V =210 V
~ When the resistances are connected in parallel, the
potential difference V is same across each resistance. P 630
Current drawn 1=- =- = 3 A.
Consequently, , V 210
.
Pex: -
1
Resistance of the element, R = -V = -210 = 70 O.
R I 3
1 Example 102. A 10 V storage battery of negligible internal
H
and I ex: -
R resistance is connected across a 500 resistor made of alloy
Hence in a parallel combination of resistances, the manganin. How much heat energy is produced in the resistor
current, power consumed and hence heat produced in 1 h ? What 'is the source of this energy ? [NCERT]
will be larger in the smaller resistance.
~,--------------~ Solution. Here V = 10 V, R = 50 0, t = 1 h = 3600 s
O
Example 104. An electric motor operating on a 50 V d.c. Rate of heat dissipation at 200 V is
supply draws a current of12 A. If the efficiency of the motor P2 = I~ R = (5 x 105)2 R =25 x 1010 R watt
is 30%, estimate the resistance of the windings of the motor.
Clearly, PI < P2 •
[NCERT]
Hence there will be lesser power wastage when the
Solution. Here V = 50 V, I = 12 A, Yl = 30%
power is transmitted at 20,000 V.
As the efficiency of electric motor is 30%, therefore, Example 107. Two ribbons are given with the following
power dissipated as heat is particulars:
70
P = 70% of VI = - x 50 x 12 W = 420 W I
100 Ribbon A B
But power dissipated as heat, P = [2 R Alloy I Constantan Nichrome
R
PR = 420 Length (m) 8.456 4.235
or R = 420 = 420 = 2.9 O. Width (mm) 1.0 2.0
12 144
Thickness (mm) 0.03 0.06
Example 105. (a) A nichrome heating element across
Temp. coefficient of egligible egligible
SI
230 V supply consumes 1.5 kW of power and heats up to a
resisti vity (OC- 1)
temperature of 750 C.A tungsten bulb across the same supply
0
operates at a much higher temperature of1600°C in order to Resistivity (Om) 4.9 x 10-7 I 1.1 x 10-6
be able to emit light. Does it mean that the tungsten bulb
For afixed voltage supply, which of the two ribbons corres-
necessarily consumes greater power? (b) Which of the two
ponds to a greater rate of heat production? [ CERT]
has greater resi~J!mce_: a 1kW heater or a ioo tv tungste..n
.bulb, both marked for 230 V ? [NCERT] Solution. Since R = p iA
Solution. (a) No, the steady temperature acquired
Resistance of constantan ribbon,
IT
by a resistor depends not only on the power consumed
but also its characteristics such as surface area, emissi- 7
R = 4.9 x 10- x 8.456 0 = 138.1 0
vity, etc., which determine its power loss due to radiation. 1 1.0 x 10-3 x 0.03 x 10-3
(b) Here V =230 V, PI =1 kW =1000 W, P2 =100 W
Let V be the fixed supply voltage. Then the rate of
2
R = V = 230 x 230 0 = 52.9 0 production of heat in constantan ribbon,
1 PI 1000 V2 V2
H
P, = - =-- watt
2 1 Rl 138.1
~ = V =230 x 230 0 =5290
P2 100 Resistance of nichrome ribbon,
Thus the 100 W bulb has a greater resistance. 6
~ = 1.1 x 10- x 4.235 0 =38.80
O
Example 106. An electric power station (100 MW) 2.0 x 10-3 x 0.06 x 10-3
transmits power to a distant load through long and thin
Rate of production of heat in nichrome ribbon,
cables. Which of the two modes of transmission would result
in lesser power wastage: power transmission of: (i) 20AJOOV V2 V2
P2 =-=--watt
or (ii) 200 V ? [ CERT] ». 38.8
M
Equivalent resistance when both parts are connected Example 111. The maximum power rating of a 20 Q
in parallel, resistor is 2.0 kW. (That is, this is the maximum power the
R' = 200 x 200 = 100 Q resistor can dissipate (as heat) without melting or changing
200 + 200 in some other undesirable way). Would you connect this
resistor directly across a 300 V d.c. source of negligible
Energy liberated per second when combination is internal resistance? Explain your answer.
connected to a source of 200 V [Haryana 97C ; NCERT]
V2 200 x 200 Solution. Maximum power rating of the given 20 Q
=- = =400 J.
R' 100· resistor,
Example 109. An electric bulb is marked 100 W,230 V. If P' =2.0 kW
the supply voltage drops to 115 V, what is the heat and light
R
When connected to 300 V d.c. supply, the power
energy produced by the bulb in 20 min? Calculate the consumption or rate of production of heat Would be
current flowing through it. [NCERT; CBSE F 94]
Solution. If the resistance of the bulb be R, then P=~ = 300 x 300 W = 4500 W = 4.5 kW
R 20
Rate of production of heat and light energy,
This power consumption exceeds the maximum
SI
V2 power rating of the resistor. Hence the 20 Q resistor
P=-
R must not be connected directly across the 300 V d.c.
2 source. For doing so, a small resistance of 10 Q should
R= V = 230 x 230 = 529 Q
P 100 be connected in series with it.
When the voltage drops to V' = 115 V, the total Example 112. An electric heater and an .electric bulb are
heat and light energy produced by the bulb in 20 min rated 500 W,220 V and 100 W,220 V respectively. Both are
will be connected in series to a 220 V d.c. mains. Calculate the
V,2 power consumed by (i) the heater and (ii) electric bulb.
IT
H=Pxt=--xt [CBSE D 97]
R
Solution. Resistances of heater and bulb are
115 x 115 2
= x 20 x 60 =30,000 J = 30 kJ. Rl = V = 220 x 220 = 484 =96.8 Q
529
PI 500 5
Current, I = V' = 115 = ~ A. 2
H
R 529 23 ~ = V = 220 x 220 =484Q
P2 100
Example 110. An electric bulb rated for 500 W at 100 V is
used in circuit having a 200 V" supply. Calculate the Total resistance of series combination is
resistance R that must be put in series with the bulb, so that Rl + ~ = 96.8 + 484 = 580.8 Q
the bulb delivers 500 W. [IIT 87]
O
1= V = 100 =5A
R 20 P2 = P~ =0.382 x 484 = 69.89 w.
For the same power dissipation, the current Example 113. Two heaters are marked 200 V, 300 Wand
through bulb must be 5 A. 200 V, 600 W. If the heaters are combined in series and the
When the bulb is connected to 200 V supply, the combination connected to a 200 V d.c. supply, which heater
safe resistance of the circuit should be will produce more heat ? [NCERT]
R
3 3
75 paise per unit, calculate the monthly bill for running
.. P{ =2 P; appliances for 6 hours a day. Take 1 month =30 days.
The first heater (of 300 W) produces more heat than Solution. Power of 20 lamps of 60 W each
the second heater.
= 20 x 60 = 1200 W
SI
Example 114. In a part of the circuit shown in the
Power consumed by 10 fans at 0.5 A current
Fig. 3.130, the rate of heat dissipation in 40 resistor is
100 J / s. Calculate the heat dissipated in the 3 0 resistor in = 10 x VI =10 x 220 x 0.5 =1100 W
10 seconds. [CBSE Sample Paper 03] Power consumed by electric kettle of110 0 resistance
2
R, R2 V =220x220 =440W
R 110
40 20
Total power of the appliances
= 1200 + 1100 + 440 =2740 W =2.74 kW
IT
Total time for which appliances are used
30
=6 x 30 =180 h
Fig. 3.130 Total energy consumed
= P. t =2.74 kW x 180 h
Solution. Let 11 be the current through the series
= 493.2 kWh or units
H
combination of ~ and ~ and 12be the current through
.'. Monthly bill = 493.2 x 0.75 = ~ 369.90.
~.
r.o. across (R1 + ~) = r.o. across ~ Example 117. There are two electric bulbs rated 60 W,
110 V and 100 W, 110 V. They are connected in series with a
.. (4 +2) 11=3 12 or 12=211
220 V d.c. supply. Will any bulb fuse? What will happen if
O
Rate of heat dissipation in 4 0 resistor they are connected in-parallel with the same supply?
= I~R1 = I~ x 4 =100 Js-1 Solution. Currents required by the two bulbs for
the normal glowness are
.. 4 =.J25 = 5 A
11= ~100
I = P1 =~ =0.55 A
1 V 110
and 12=211 =10 A
M
As II < I and 12> I, so that 60 W bulb will fuse Solution. Water equivalent of copper kettle is
while the 100 W bulb will light up dim. w = Mass x Specific heat = 1000 x 0.1 = 100 g
When the bulbs are joined in parallel, their
Also m = 900 g,
equivalent resistance is
9 = 02 - 91 =100 -20 =80°C
R' = Rl Rz = 202 x 121 =76Q
Rl + Rz 202 + 121 Heat required,
H = (m + w) c 9 = (900 + 100) x 1 x 80 =80,000 cal
Current drawn from the 220 V supply will be
Heat produced
I' = ~ = 220 :::.3 A
R' 76 = V I t = 210 x I x 12 x 60 cal =36000 I cal
4.2 4.2
In the two bulbs of resistances Rl (:::.202 Q) and
R
Useful heat
Rz (= 120 Q), the current of 3 A will split up into
roughly 1 A and 2 A respectively. Hence both the bulbs = 90% of 36000 I
will fuse. = 90 x 36000 [ = 32400 I cal
Example 118. The resistance of a 240 V and 200 W 100
electric bulb when hot is 10 times the resistance when cold. .. 32400 1=80,000
SI
Find its resistance at room temperature. If the working Current, I = 80000 = 2.469 A.
temperature of the filament is 2000°C, find the temperature 32400
coefficient of the filament.
Example 121. A coil of enamelled copper wire of resis-
Solution. Resistance of the hot bulb is given by tance 50 Q is embedded in a block of ice and a potential
R' = ~
= 240 x 240 = 288 Q difference of 210 V applied across it. Calculate the rate at
P 200 which ice melts. Latent heat of ice is 80 cal per gram.
Resistance of bulb at room temperature, Solution. Here R = 50 Q, V = 210 V, t = 1 s,
IT
R= !3'.. = ~~ = 28.8 Q L=80 cal g-1
10 10
Heat produced,
Since R'= R(l + at)
H= ~ = 210 x 210 x 1 = 210 cal
288 = 28.8(1 + a x 2000)
4.2 R 4.2 x 50
or - 9 0C-1 _-.4 5 x10-3 °C-1 .
a--- Suppose m gram of ice melts per second. Then
2000
H
mL=H
Example 119. A thin metallic wire of resistance 100 Q is
immersed in a calorimeter containing 250 g of water at 10° C or m = H = 210 = 2.62 gs ".
and a current of 0.5 ampere is passed through it for half an L 80
hour. If the water equivalent of the calorimeter is 10 g, find Example 122. An electric kettle has two heating coils,
the rise of temperature. when one of the coils is switched on, the kettle begins to boil
O
Solution. Here m = 250 g, l = 0.5 A, in 6 minutes and when the other is switched on, the boiling
begins in 8 minutes. In what time will the boiling begin if
t=30min=1800s, w=10g
both the coils are switched on simultaneously (i) in series
.'. Heat produced and (ii) in parallel ? lIlT]
= [2 Rt = (O.5l x 100 x 1800 J = 45000 J Solution. Let Rl and Rz be the resistances of the
M
(i) When the coils are connected in series, Useful heat produced
effective resistance = R1 + ~. = 80% 2000 t = 80 x 2000 t J
Let the boiling occur in time t1 min. 100
Then = 80 x 2000 t cal
V\ x 60 V 2 x 6 x 60 100 x 4.2
--~-- = H = -----
4.2(R1 + ~) 4.2 R1 Useful heat produced = Heat gained by water
80 x 2000 t = 80000
or t1 =6( R1 ;1 ~ ) =6 (1+ ~ J 100 x 4.2
t = 80000 x 100 x 4.2 = 210 s.
R
or
= 6 (1 + ~) min = 14 min. 80 x 2000
SI
R1 + ~ in the heater? (iv) How much heat in calories is produced
Let the boiling occur in time t2 min. Then per second? (v) How many grams of water at 100°C will be
converted per minute into steam at 100° C, with the heater?
V2t2 x 60 V2 x 6 x 60
----,------'''-------,
- H - ---- Assume that the heat losses due to radiation are negligible.
4.2 -~~
R1 ». J- -
4.2 R1 . Latent heat of steam = 540 cal per gram [liT)
( Rl + ~ Solution. Here P = 1 kW = 1000 W, V = 220 V
R 1
~J
R
or t2 = 6 x 1''2 =6 x -,---------,- (i) Current, I = £ = 1000 = 4.55 A.
+ (1+ V 220
IT
(R1 ~)R1
2
(ii) Resistance, R = V = 220 x 220 = 48.4 Q.
P 1000
6x ( \) min = 3.43 min.
1+- (iii) Power dissipated in heater = 1000 W.
4
(iv) Heat produced per second,
H
Example 123. The heater coil of an electric kettle is rated
H = VIt =~ = 1000 xl = 240 cal s-l.
at 2000 W, 200 V. How much time will it take in raising the J J 4.2
temperature of1litre of water from 20°C to 100°C, assuming
that only 80% of the total heat energy produced by the heater (v) Heat produced per minute,
coil is used in raising the temperature of water. Density of H = 240 x 60 = 14400 cal
O
The amount of heat conducted out per second Solution. Here e = 1.5 V, r = 0.10 Q, 1=2.0 A
through the walls of the cubical box is
(a) Rate of chemical energy consumption of the cell
H _ KA(92-91)_ 4x 10-4 x 15000 x 100
= eI = 1.5 V x 2.0 A = 3.0 W.
I - d - 0.1
(b) Rate of energy dissipation inside the cell
= 6000 cal = 6000 x 4.2 J
=12 r =(2)2 x 0.10 W = 0.40 W.
If R is the resistance of the heater, then heat produced
(c) Rate of energy dissipation inside the resistor
per second
2 =eI- I2r =3.0 -0.40 = 2.6 W.
. H2 = I2Rt = V = (400)2 [t =ls]
R R (d) Power output of the source
Temperature inside the box will be maintained by = Power input to the external circuit
R
the heater if =eI - I2r=2.6 W.
HI = H2 or (40~)2 = 6000 x 4.2 Example 129. A series battery of 10 lead accumulators,
each of emf 2 V and internal resistance 0.25 ohm, is charged
or R = 400 x 400 = 6.35 Q by a 220 V d.c. mains. To limit the charging current, a
6000 x 4.2
SI
resistance of 47.5 ohm is used in series in the charging
Example 126. A 10 V battery of negligible internal circuit. What is (a) the power supplied by the mains and
resistance is charged by a 200 V d.c. supply. If the resistance (b) power dissipated as heat? Account for the difference of
in the charging circuit is 38 Q, what is the value of charging power in (a) and (b). [CBSE Sample Paper 98]
resistance 0.10 Q is connected across a resistor in series with Solution. Here e = 2.0 V, r = 0.50 Q, V = 100 V,
a very low resistance ammeter. When the circuit is switched I =8.0 A
on, the ammeter reading settles to a steady value of2.0 A As the battery emf opposes the charging emf,
What is the steady
:. Effective emf =100 -2.0 x 6 =88 V
(a) rate of chemical. energy consumption of the cell,
(b) rate of energy dissipation inside the cell, Let the required series resistance be of R Q.
(c) rate of energy dissipation inside the resistor, Then
(d) power output of the source? [NCERT] total resistance = (0.50 x 6 + R) Q = (3 + R) Q
3.72 PHYSICS-XII
R
= P(R + r) =82(8 + 0.50 x 6) W emf 24
= 64 x 11 W = 704 W. I= =--A=3.0A.
resistance 4+4
(c) Power supplied by the d.c. energy stored in the
battery in 15 min Example 133. 4 cells of identical emf E, internal
resistance r, are connected in series to a variable resistor. The
SI
= (800 -704)W x 15 min
following graph shows the variation of terminal voltage of
= 96 W x 900 s = 86400 J. the combination with the current output.
Example 131. Power from a 64 V d.c. supply goes to (i) What is the emf of
charge a battery of 8 lead accumulators each of emf 2.0 V each cell used? 5.6
Es. Therefore, net current in the circuit must be The output power is maximum, when
I= Net emf = Es -Eb -Em externaJ resistance = internal resistance = 4 r
Net resistance r+ R Total emf 4E
Total resistance 4r + 4r
or 3.5 = 64 -16-E m
8
=~=~=lA.
or Em = 48 -28 =20 V. 2r 2xO.7
(a) Mechanical energy yielded by motor in 1 h
Example 134. Two batteries, each of emf E and internal
= Em. It =20 x 3.5 x 3600 J = 252000 J. resistance r, are connected in parallel. If we take current
(b) Chemical energy stored in the battery in 1 h from this combination in an external resistance R, then for
what value of R maximum power will be obtained? What
= Eb. It = 16 x 3.5 x 3600 J = 201600 J. will be this power ?
CURRENT ELECTRICITY 3.73
Solution. The situation is shown in Fig. 3.133. Now h is independent of Iand the values of r and p
are same for both wires, hence steady state tempe-
e rature e will depend only on I i.e., the two fuses will
melt at the same values of current.
Example 136. A fuse with a circular cross-sectional radius
of 0.15 mm blows at 15 A What should be the radius of
R cross-section of afuse made of the same material which will
blow at 30 A ? [NCERT]
Fig. 3.133 Solution. Here r1 =0.15 mm, II = 15 A, r2 =?
Net emf of the parallel combination of two cells = E, 12 =30 A
R
Total resistance in the circuit From Eq. (i), the heat lost per second per unit
surface area of the wire is
=rxr+R=!.+R
r+ r 2 Pp
h=--2 3
Hence current in the circuit is 21t r
SI
I=_E,_=~ For a fuse wire of the given material and the
!.+R r+2R given value of h,
2 r3 oc I2
Power dissipated in the resistance R is 3 2
1'2 _ 12
2 or
P= PR = (2 E,)2 R = 4 E, R 3--2
r1 II
(r+2R)2 (r-2R)2+8rR
Power P will be maximum when the denominator or r2"" = ~I 2
x r13 = (30)2
- x (0.15)3
has a minimum value. This happens when II - 15
IT
(r - 2 R)2 =0 or R = !. r2 = (4)1/3 x 0.15 mm
2
= 1.5874 x 0.15 mm = 0.24 mm.
E,2
P = (2E,l r/2
max (r+rl 2r 'Problems For Practice
Example 135. Two wires made of tinned copper having 1. Calculate the current flowing through a heater
H
identical cross-section ( = 1O-6m2) and lengths 10 em and rated at 2 kW when connected to a 300 V d.c.
15 em are to be used asfuses. Show that thefuses will melt at supply. [CBSEF 94 Cl
the same value of current in each case. [NCERT] (Ans. 6.67 A)
Solution. The temperature of the wire increases up 2. Calculate the amount of heat produced per second
to a certain temperature e where the heat produced per (in calories), when a bulb of 100 W - 220 V glows
O
second by the current equals heat lost (by radiation) assuming that only 20% of electric energy is
per second. converted into light. J = 4.2 J cal-1. [Haryana 011
But heat produced by the current (Ans. 19.05 call
3. An electric heating element to dissipate 480 watts
= I 2 R = l 2 P .!..- = I 2p/
on 240 V mains is to be made from nichrome ribbon
M
A 1t?
1 mm wide and thickness 0.05 mm. Calculate the
If h is heat lost per second per unit surface area of length of the ribbon required if the resistivity of
the wire and if we ignore the heat loss from the end nichrome is 1.1 x 10- 6 Om. (Ans. 5.45 m)
faces of the wire, then heat loss per second by the wire
4. 100 W, 220 V bulb is connected to 110 V source.
= h x curved surface area of the wire Calculate the power consumed by the bulb.
= h x 21trl [Roorkee 861
When the steady state temperature is attained, (Ans. 25 W)
[2 pI 5. How many electrons flow per second through an
hx21tr/=-.-2 electric bulb rated 220 V, 100 W ? [BIT Ranchi 98]
1tr
(Ans. 2.84 x 1018)
Pp 6. An ammeter reads a current of 30 A when it i.s
or h=-23 ...
(i)
21t r connected across the terminals of a cell of emf 1.5 V.
3.74 PHYSICS-XII
Neglecting the meter resistance, find the amount of 17. Two bulbs rated 25 W, 220 V and 100 W, 220 V are
heat produced in the battery in 10 seconds? connected in series to a 440 V supply. (i) Show with
(Ans. 107.14cal) necessary calculations which bulb if any will fuse.
(ii) What will happen if the two bulbs are connected
7. A coil of resistance 1000 is connected across a
battery of emf 6.0 V. Assume that .the heat deve- in parallel to the same supply?
loped in the coil is used to raise its temperature. If [Ans. (i) 25 W bulb will fuse
the thermal capacity of coil is 4.0 JK-1, how long (ii) Both the bulbs will fuse]
would it take to raise the temperature of the coil by 18. A servo voltage stabiliser restricts the voltage
15°C? (Ans. 2.8 min) output to 220 V ± 1%. If an electric bulb rated at
8. A generator is supplying power to a factory by 220 V, 100 W is connected to it, what will be the
minimum and maximum power consumed by it ?
R
cables of resistance 20 O. If the generator is
generating 50 kW power at 5000 V, what is the (Ans. 98.01W, 102.01W)
power received by the factory? [Punjab 96 C] 19. A room is lighted by 200 W, 124 V incandescent
(Ans. 48 kW) lamps fed by a generator whose output voltage is
9. Two bulbs are marked 220 V, 100 W and 220 V, 130 V. The connecting wires from the generator to
SI
50 W respectively. They are connected in series to the user are made of aluminium wire of total length
220 V mains. Find the ratio of heats generated in 150 m and cross-sectional area 15 mm2. How many
them. (Ans. 1 : 2) such lamps can be installed ? What is the total
power consumed by the user? Specific resistance of
10. In a house having 220 V line, the following applia-
aluminium = 2.9 x 10-8 Om. (Ans. 12, 2.4 kW)
nces are working: (i) a 60 W bulb (ii) a 1000W heater
(iii) a 40 W radio. Calculate (a) the current drawn by 20. Two wires A and Bof same material and mass, have
heater and (b) the current passing through the fuse their lengths in the ratio 1 : 2. On connecting them,
line. [MNREC 86]
one at a time to the same source of emf, the rate of
heat dissipation in B is found to be 5 W. What is the
IT
(Ans. (a) ~~ A (b) 5 A) rate of heat dissipation in A ? (Ans. 20 W)
21. Two electric bulbs rated as 100 W, 220 V and 25 W,
11. Three equal resistances connected in series across a 220 V are connected in series across 220 V line.
source of emf consume 20 W.If the same resistances Calculate (i) current through (ii) potential difference
are connectedin parallel acrossthe same source of emf, across and (iii) actual powers consumed in filament
what will be the power dissipated? [Punjab 99] of each bulb.
H
(Ans. 180 W)
(Ans. (i) 1\ A (ii) 44 V, 176V, (iii) 4 W, 16 W)
12. An 'electricheater consists of 20 m length of manganin
wire of 0.23 m2 cross-sectional area. Calculate the 22. The heater coil of an electric kettle is rated as 2000W
wattage of the heater when a potential difference of at 200 V. How much time will it take to heat one
200 V is applied across it. Resistivity of manganin
O
Find the rate at which energy is dissipated in the 30. The 2.0n resistor shown in Fig. 3.138is dipped into
resistor. If the total heat produced could be made a calorimeter containing water. The heat capacity of
available for heating water, how long would it take the calorimeter together with water is 2000JK-1.
to bring 1 kg of water from 15°C to the boiling (a) If the circuit is active for 30 minutes, what would
point? Specific heat of water =lcalg-IoC-1 and be the rise in the temperature of the water ?
1cal = 4.2 J. [MNREC 84] (b) Suppose the 6.0n resistor gets burnt. What
(Ans. 1350 J s -1, 264.4s) would be the rise in the temperature of the water in
26. In the circuit shown in Fig. 3.134, each of the three the next 30 minutes? (Ans. 5.8°C, 7.2°q
resistors of 4 n can have a maximum power of 20 W 6V lQ
(otherwise it will melt). What maximum power can
the whole circuit take? (Ans. 30 W)
R
H2
4Q
4Q
SI
2Q
Fig. 3.134
27. Find the heat produced per minute in each of the resis- Fig. 3.138
tors shown in Fig. 3.135. (Ans. 360 J, 720 J, 540 J)
31. Three resistors ~,Rz and Rs each of 240 o are
II 6Q
connected across a 120V supply, as shown in
Fig. 3.139. Find (i) the potential difference across
each resistor and (ii) the total heat developed across
the three resistors in 1 minute.
IT
9V i n [Ans. (i) VI = 80 V, (ii) V2 = V3 = 40 V (iii) 2400 JJ
RI
Fig. 3.135
120V
28. Calculate the current drawn from the battery of emf
H
15 V and internal resistance 0.5n in the circuit
shown in Fig. 3.136.Also find the power dissipated
in the 6 n resistor. [lIT] Fig. 3.139
15 V,
0.5 Q-=-
-=- difference of 200 V is applied and the temperature
lQ of the liquid is found to increase by 60°C in 20
minutes. If R is removed, the same rise in,
temperature is reached in 15 minutes. Find the
8Q value of R (Ans.22.14n)
M
Fig. 3.136 33. A house is fitted with two electric lamps, each of
100 W ; one heater of resistance 110n and two fans,
29. In the circuit shown in Fig. 3.137,the heat produced each consuming 0.25 A. If electric energy is supplied
by 4 n resistance due to current flowing through it at 200 V and each appliance works for 5 hours a day,
is 40 cal s -1. Find the rate at which heat is produced find the monthly bill at the rate of Rs. 3.0 per kWh.
in 2 n resistance. (Ans. 80 cal s -1) [Punjab 98C] (Ans. ~298.65)
35. A series battery of 6 lead accumulators, each of emf 5, n ~ !!.. = £!.. = 100 x 1 = 2.84 x 1018
2.0 V and internal resistance 0.25 n is charged by a e Ve 220x1.6x10-19
230 V d.c. mains. To limit the charging current, a
6. If r is the internal resistance of the cell, then
series resistance of 53 n is used in the charging
circuit. What is (i) power supplied by the mains 1= ~ or r = §. = 1.5 = 0.05 n
r 1 30
(ii) power dissipated as heat ? Account for the
difference in the two cases. [NCERT]
12 rt (30)2 x 0.05 x 10
H = -- = = 107.14 cal.
(Ans. 920 W, 872 W) J 4.2
36. A storage battery of emf 8 V, internal resistance 1n, is 7. Heat required by the coil = Thermal capacity
being charged by a 120 V d.c. source, using a 15 n x rise in temperature
resistor in series in the circuit. Calculate (i) the = 4.0 x 15 = 60 J
R
current in the circuit, (ii) terminal voltage across the Rate of production of heat,
battery during charging, and (iii) chemical energy V2 6 x6
stored in the battery in 5 minutes. [CBSE 01, 08] p=-=-- = 0.36 Js-1
R 100
[Ans. (i) 7 A, (ii) 15 V, (iii) 16800 JJ R . d . 60J 60 .
37. The following graph shows the variation of terminal .. eqUire time = 0.36 Js-l 0.36 x 60 rrun
SI
potential difference V, across a combination of three
cells in series to a resistor, '" 2.8 min.
versus the current, i: 6.0 8. Here P = SO:kW :0 50 x 103 W, V = 5000 V
4.2
Current drawn by bulb = ~ =~ =~ A
V2 V2 240 x 240 V 220 11
3. Power, P = - .. R = - = = 120 n
R p
Cu rrent d rawn b y radio -_!i~_
480 ,_40 -_--12 A
-
I Area of cross-section of the ribbon, V 220 11
A = 0.05 mm 2 = 0.05 x JQ-om 2 (b) Current passing through fuse for the line
M
Required length, 50 3 2
=-+-+-=5A.
11 11 11
RA 120 x 0.05 x 10-6
1= -
p
=
11 x 10-
6 m = 5.45 m. 11. Let R be the resistance of each resistor and e the emf
of the source.
Here P = 100 W, V = 220 V For series combillatioll: Rs = R+ R+ R =3 R
V2
220 x 220
:. Resistance of bulb, R = - = = 484 n V2 V2 V2
P 100 P=- or 20=- •. -=60W.
Rs 3 R R
When the bulb is connected to 110 V source, the
power consumed by the bulb is For parallel combination: Rp = R/3
2
V,2 110 x 110 V2 V2 3V
P'=-= =25W. p'=-=-- =--=3 x60=180W.
R 484 Rp R/3 R
CURRENT ELECTRICITY 3.77
200
12. First find R = P ~ and then P = ~ . Current through each lamp = - = 1.613 A
A R 124
13. Let P' be the power loss in the transmission line in :.No. of bulbs which can be used = 20.69 = 12.83.
1.613
the form of heat. Then
= 413.2 W = 0.4132 kW
x 0.2
No. of bulbs that should be installed = 12.
Power consumed = 12 x 200 = 2400 W = 2.4 kW.
20. As the two wires are of same material and mass,
their volumes must be equal.
Efficiency of transmission,
Power delivered by line :. ~~ = a212 or ~ x 1= a2 x 21 or ~ = 2a2
11 = Power supplied to line
If E, is the emf of the source, then rate of heat
dissipation in wire B is
R
Power delivered E,2 E,2
Power delivered + Power loss -=5 or --=5
~ P 12/ a2
10
---- = 0.96 = 96%. E,2 E,2 a
]0 + 0.4132 . or ---=5 or __ 2 =10
p.21/a2
SI
pi
14. Power supplied to line = VI = 120 x 2 = 240 W
Power loss in the form of heat Rate of heat dissipation in wire A is
E,2 €,2 E,2
=9cals-1 =9x4.2}s-1 = 37.8 W - = -- =-.2a2 =2 x10=20W.
~ p 1/ ~ pI
Power delivered by line = 240 - 37.8 = 202.2 W
21. Proceed as in Example 123 on page 3.70.
Effi . Power delivered by line 202.2
ICIency, 11 = -------"---
Power supplied to line 240
22. [= i = 2000 = 10 A
V 200
= 0.8425 = 84.25%. .. VIt 200 x 10 x t
IT
Heat produced m time t =- = cal
15. Here P = 500 W, V = 200 V
J 4.2
2 Heat gained by water = mdJ = 1000 x 1 x 80 cal
R = V = 200 x 200 = 80 n
P 500 2000t
.. -- = 1000 x80
4.2
When the voltage drops to 160 V, rate of heat
1000 x 80 x 4.2
production is or t = = 168 s.
H
,2 2000
P' = V = 160 x 160 = 320 W 2
R 80 R= V = 200 x 200 =20n.
P 2000
% Drop in heat production
23. Heat absorbed by water
P- pi 180 x 100
=-- x100= =36%. = 1 x 4200 x (100 - 15) = 4200 x 85 J
O
P 500
Heat produced by electric kettle
17. Proceed as in Example 117 on page 3.68.
= Pt = 500 x 15 x 60 J
(220)2
18. Resistance of the bulb, R = -- = 484 n . . 4200 x 85
100 Heat efficiency = x 100 = 79.3%.
500 x 15 x 60
Variation in voltage = ± 1% of 220 V = ± 2.2 V
M
Minimum voltage = 220 - 2.2 = 217.8 V 24. Proceed as in Example 120 on page 3.69.
. 120 - 30
.
Mi mmum -_(217.8)2 -_98.01 W. 25. Chargmg current, l = = 15
power R
484
Maximum voltage = 220 + 2.2 = 222.2 V
· .
:. Senes resistor, = -90 = 6 n R
15
Maximum power = (222.2)2 = 102.01 W. Rate of energy dissipation in the resistor,
484 P= [2R =(15)2 x 6 =1350 }S-l.
19. Resistance of aluminium wire, Heat produced in resistor in time t = Heat absorbed
R _ pi _ 2.9 x 10- 8 x 150 by water
- A-IS x 10- 6 = 0.29 n
1350 x t = 1 x 4200 x (100 - 15)
.. 130 -124 4200 x85
Current from the mam Ime = = 20.69 A or t= = 264.4 s.
0.29 1350
3.78 PHYSICS-XII
26. Let I be the current through a resistance of maximum Heat produced per second in 40 resistor,
power 20 W. Then ~ ex: l~ x 4
fR=W m fx4=W m f=5
11 112 x 2 (21/ 3)2 x 2
Effective resistance between A and C, .. -=--= =2
~ I~ x 4 (I/ 3) x 4
4x4
R' = -- + 4 = 2 + 4 = 60
4+ 4 or 11 = 2 ~ =2x40=SOcals-1.
The maximum power that can be dissipated by the 30 . ()a Tot al resi . th e CIrCUIt
resistance In . . =--6 x2 + 1 =-5 rv
><
circuit, 6+2 2
p= f R' = 5 x 6 = 30 W. 6V 12
Total current, I= =- A
27. The equivalent resistance of the circuit is (5/2)0 5
R
6x3
R=--+ 1=2+ 1=30 Current through 20 resistance
6+ 3
= 12 x _6_ = 1.8 A
Current drawn from the battery is 5 6+ 2
1= 9V =3 A
30 Heat produced in 20 resistance in 30 minutes
SI
= (1.8)2 x 2 x 30 x 60 = 11664 J
As the current through 10 resistor is 3 A, so heat
produced in this resistor in 1 minute (or 60 s) is Rise in temperature
R = f Rt = 32 x 1 x 60 = 540 J 11664 J
= --- = 5.S K or 5.S°C.
Current through 60 resistor, 2000 JK
3 (b) When the 60 resistor gets burnt,
II =--x3=IA
6+ 3 6V
Current = =2A
.. Heat produced in 60 resistor (2 + 1)0
IT
= 12 x 6 x 60 = 360 J. Heat produced in 20 resistor in 30 minutes
Current through 30 resistor, =(2)2 x 2 x 30 x 60 = 14400J
12= I - II = 3 - 1 = 2 A Rise in temperature
:. Heat produced in 30 resistor 14400 J
= I = 7.2 Kor 7.2°C.
= 22 x 3 x 60 = 720 J. 2000 JK-
H
28. The distribution of current is shown in Fig. 3.136. 31. (i) Total resistance of the circuit,
Applying Kirchhoff's second law to the loops 1 and
~ x~ 240 x 240
2, we get R= ~ + <'2 "3 =240+ =3600
~ + ~ 240 + 240
(I - II) x (7 + 1 + 10) - II x 6 = 0
and II x 6 + I x (8 + 0.5 + 2) = 15 Current drawn from the battery,
O
29. Resistance of the upper arm = 2 + 3 = 50 so current through each of these resistors
Resistance of the lower arm = 4 + 6 = 100 1 1 1
=-x-=-A
Let I be the total current in the circuit. Then current 2 3 6
flowing through the upper arm will be 1
P.D. across ~ or ~, V2 = V3 = 240 x (; = 40 V.
I x 10 21
II = 5 + 10 ="3 (ii) Total heat developed in three resistors in
1 minute,
Current flowing through the lower arm.
Ix5
I =--=-
2 5 + 10 3
I
R = 12Rt = ur x 360 x 60 = 2400 J.
Heat produced per second in 20 resistor, 32 Here m = 2 kg = 2000 g, C = 0.5 cal g -1 -c'. e = 60°C,
11 ex: 112 x2 tl =20min, t2 =15m, R=?
CURRENT ELECTRICITY 3.79
I=~
. Total emf 112
:. Current, (I) Current, 1= =- = 7 A.
R+r Total resistance 16
(ii) Terminal voltage during charging,
Heat dissipated in time tI,
V = e + Ir = 8 + -7 x 1 = 15 V.
(iii) Chemical energy stored in the battery in
R
5 minutes
In the second case, the resistance R is removed.
=eIt = 8 x 7 x(5 x 60) =16800 J.
:. Current, 1= V
r 37. (i) Total emf the three cells in series
V)2 V2 t = P.O. corresponding to zero current = 6.0 V
Heat dissipated in time t2, H2 = ( -;: rt2 = ~
SI
:. EMF of each cell = 6.0/3 = 2.0 V
(ii) When i = 1.0 A, V = 3.0/ 3 = 1.0 V
As the liquid is raised to same temperature in both
cases, so e - V 2.0 -1.0
.. r = -- = = 1.00
i 1.0
H= HI = H2
or (R~ rf
r
2
rtl =
15 3
(~r
t2
rt2
The output power is maximum, when
external resistance = internal resistance = 3r
.
1
max
=
Total emf
Total resistance
3E.
=---=-
e
3r + 3r 2r
IT
or
(R + r)2 = ~ = 20 ="4
=~=1.0A.
r.J3 R+r 2 2 x 1.0
or R+r 2 or -r- = .J3
or
R
-+1=-
2 3.31 KIRCHHOFF'S LAWS
r .J3 Introductory concepts. In 1942, a German physicist
H
or ~ = 1.155-1=0.155 or r=~ Kirchhoff extended Ohm's law to complicated circuits
r 0.155 and gave two laws, which enable us to determine
As H= H2 current in any part of such a circuit. Before under-
standing these laws, we first define a few terms.
5 (200)2 x 15 x 60 x 0.155
2.52 x 10 = -'--~------
1. Electric network. The term electric network is used
O
R
4 x 104 x 15 x60 x 0.155 for a complicated system of electrical conductors.
or R= 5 = 22.140. 2. Junction. Any point in an electric circuit where two
2.52 x 10
or more conductors arejoined together is a junction.
33. Proceed as in Example 116 on page 3.68.
3. Loop or Mesh. Any closed conducting path in an
34. Proceed as in Example 122 on page 3.69. electric network is called a loop or mesh.
M
35. EMF of the battery = 6 x 2.0 = 12 V 4. Branch. A branch is any part of the network that lies
Internal resistance of the battery = 6 x 0.25 = 1.50 between two junctions.
Total resistance = 1.5 + 53 = 54.5 n 56. State the two Kirchhoffe laws for electrical
Charging current circuits and explain them giving suitable illustrations.
Effective emf 230 - 12 Also state the sign conventions used.
---=4.0A
Total resistance 54.5 Kirchhoff's first law or junction rule. In an electric
(i) Power supplied by the mains circuit, the algebraic sum of currents at any junction is zero.
Or, the sum of currents entering a junction is equal to the
= VI = 230 x 4.0 = 920 W.
sum of currents leaving that junction.
(ii) Power dissipated as heat
Mathematically, this law may be expressed as
= 12 (R + r) = (4)2 x(53+ 1.5) = 872 W. L 1=0
3.80 PHYSICS-XII
1 +
Sign convention for applying junction rule:
1. The currents flowing towards the junction are
taken as positive.
----.
--+--'VV'v-
V=+IR
Fig. 3.144 Positive potential drop across a resistor.
2. The currents flowing away from the junction
are taken as negative. 5. The IR product is taken as negative if the
resistor is traversed in the opposite direction of
Figure 3.141 represents a assumed current.
junction J in a circuit where I + - I
four currents meet. The
currents II and 12 flowing
....--
--+--'VV'v-
V=-IR
R
towards the junction are
Fig. 3.145 Negative potential drop across a resistor.
positive, while the currents
13 and 14 flowing away from Illustration. Let us consider the circuit shown in
the junction are negative, Fig. 3.146.
therefore, by junction rule: Fig. 3.141 Junction rule: Fig. 3.146 An electrical circuit.
R3
SI
11 + 12 = 13 + 14•
o vv E
or
or
II + 12 - 13 - 14
II + 12
13 + 14
=0
= R2
J 12
e
1
2
L t.V or L IR
Sign convention for applying loop rule:
e
2 = I2~ +(11 + 12)~
Second law is also called Kirchhoffs voltage law (KVL).
1. We can take any direction (clockwise or anti-
clockwise) as the direction of traversal. Justification. This law is based on the law rof
conservation of energy. As the electrostatic force is a
2. The emf of cell is taken as positive if the
conservative force, so the work done by it along any
M
direction of traversal is from its negative to the closed path must be zero.
positive terminal (through the electrolyte).
Formulae Used
Fig. 3.142 Positive emf. Fig. 3.143 Negative emf.
1. L I = a (Junction rule)
3. The emf of a cell is taken as negative if the or Totalincoming current = Total outgoing current
direction of traversal is from its positive to the
2. Le=L IR (Loop rule)
negative terminal,
4. The current-resistance (IR) product is taken as Units Used
positive if the resistor is traversed in the same Current I is in ampere, resistance R in ohm and
direction of assumed current. e
emf in volt.
CURRENT ELECTRICITY 3.81
Example 137. Network PQRS (Fig. 3.147) is made as Solving equations (1), (2) and (3), we get
under: PQ has a battery of 4 V and negligible resistance 48 18 66
with positive terminal connected to P, QR has a resistance of II = - A, 12= - A, [3 = - A
31 31 31
60 n. PS has a battery of 5 V and negligible resistance with
positive terminal connected to P, RS has a resistance of Example 139. Find the potential difference across each cell.
200 n. If a milliammeter, of 20 n resistance is connected and the rate of energy dissipation in R. [Fig. 3.149(ti)}.
between P and R, calculate the reading of the milliammeter. [CBSE Sample Paper 11]
[NCERT]
200n e1 = 12 V '1 =2o
R
R
5V R=4n
60n
p Q
4V
SI
Fig. 3.147 Fig.3.149(a)
Solution. Applying Kirchhoff's second law to the Solution. Applying Kirchhoff's laws,
loop PRQP, we get For closed loop ADCBA
20[1 + 60[ = 4 ... (i) 12 = 4(Il + 12)+2 II =6 II + 4 12 ...(i)
Similarly, from the loop PSRP, we get
For closed loop ADEFA,
200 (l- II) -20[1 =- 5
6 = 4(Il + 12) + II = 4 11 + 5 12 ...(il)
or 40[ -4411 =-1 ... (ii)
IT
Multiplying (i) by 2 and (ii) by 3, we get
120[ + 4011 = 8 ... (iii)
and 1201 -132 II = -3 ...(iv)
Subtracting (iv) from (iii), we get R=4n
A•.......•..
-----'\N\r--___. 0
H
172[1 =11
or [ =~ =0.064 A
1 172 F '-- ...•..---l:1---..JVV'v----' E
12
Thus the milliammeter of 20 o will read 0.064 A.
Example 138. Using Kirchhoff's laws in theelectricalnetwork Fig. 3.149(b)
O
P'D, across R =V
M
= (II + I2)R
F E o
= (18-6)
-7- x 4 volt ='748 volt
Fig. 3.148 P'D. across each cell = p.o. across R = 48 V
7
Solution. Applying Kirchhoff's first law at junction B,
Energy dissipated in R = 4 n resistor
II + [2 =13 ... (1)
Applying Kirchhoff's second law to loops ABEFA
= (II + [2) R = 2 (12)2
'7 x 4}
and BCDEB, we get
213 + 511 = 12 (2)
= 576 J = 11.75 J.
- 213 - 3[2 = - 6 (3) 49
3.82 PHYSICS-XII'
e1~2V
Example 140. Two cells of emfs 1.5 V and 2.0 V and 11 '1 ~4Q
internal resistances 1 n and 2 n respectively are connected
in parallel so as to send current in the same direction
through an external resistance of 5 n. [CBSE OD 05] 12
e2~ 1 V
'2~3Q
)
(i) Draw the circuit diagram.
(ii) Using Kirchhoff's laws, calculate
(a) current through each branch of the circuit.
13
e3~4 V r3~2Q )
(b) p.d. across the 5 n resistance.
Fig. 3.151
Solution. (I) The circuit diagram is shown in Fig. 3.150.
Solution. By Kirchhoff's junction rule,
R
II + Iz
13 = ... (i)
From upper loop,
FQ- .....•
:
~---l
r----------
I 1Q
J 3Iz-4I1 =2-1=1 ...(ii)
SI
: I v v :
: __1~~": ~~ .: 3Iz+2I3=4-1=3 ...(iii)
J B
On solving equations (i), (ii) and (iii), we get
Fig. 3.150 Example 142. Apply Kirchhoffs rules to the loops ACBPA
and ACBQA to write the expression for the currents II' Iz
(ii) (a) Let II and Iz be the currents as shown in and 13 in the network shown in Fig. 3.152. [CBSE OD 10]
IT
Fig. 3.150. Using Kirchhoff's second law for the loop Solution. By Kirchhoff's e1 ~ 6 V
AFCBA, we get junction rule, P
2 Iz - 1Il= ~\ - E.l = 2 -1.5
13 = II + 12 ... (1)
or 2Iz - II = 0.5 ...(1)
From loop AQBP A,
For loop CFEDC, we have
H
0.5 II - 12 = 6 -10 = -4
1Il + 5(11 + Iz) = e,
= 1.5
... (ii)
or 5Iz + 611 = 1.5 ...(2) c
Solving equations (1) and (2), we get From loop ACBP A,
R~12Q
1 9 12 13 + 0.5 II =6
O
.'. Current through branch BA, On solving equations (i), (ii) and (iii), we get
1 I = - 84 A I = 106 A I = _ 22 A
II =-A
34 1 37 2 37 3 37
Current through branch CF,
M
::AL~
(b) P.D. across the 5 o resistance
10 R
= (11 + Iz) x 5 = - x 5 V = 1.47 V.
34
Example 141. Use Kirchhoffs rules to write the c
B
expressions for the currents I1,Iz and 13 in the circuit 6V 4V
diagram shown in Fig. 3.151. [CBSEOD 10]
Fig. 3.153 (a)
CURRENT ELECTRICITY 3.83
Solution. No current flows through the arm BE. Solution. Let I}, 12 and 13 be the currents as shown
Let I be the current along the outer loop as shown in Fig. 3.155. Kirchhoff's second rule for the closed
in Fig. 3.153(b). loop ADCA gives
3Q E
~:e
F.-~~~ __-.~~D 10 - 4( II - 12) + 2{ 12 + 13 - II) - II = 0
I or 711-612-213=10 ...(1)
R For the closed loop ABCA, we get
R1
3V
10 - 4 [2 - 2(12 + 13) - 11 =0
T ,
r---:----i or 11+612+213=10 ...(2)
A I c
R
6V B 4V For the closed loop BCDEB, we get
SI
I =2A 5 7
I} =2.5 A, 12 =8" A, 13 =18 A
From A to D along AFD,
VAD =2 x2 -1+3x2 =9 V. The currents in the various branches of the network
are:
Example 144. In the circuit Fig. 154, assuming point A to
5 1 7
be at zero potential, use Kirchhoffs rules to determine the IAB=-A; ICA =2- A; IDEB =1- A
potential at point B. 8 2 8
7 1
lAD = 1- A; ICD = 0; I
BC
=2- A.
4V
IT
lA 0 3A 8 2
II B
Example 146. In the circuit shown in Fig. 3.156(a), E, F,
R 2Q
e and H are cells of emf 2 V, 1 V, 3 V and 1 V, and their
R1
internal resistances are 2 Q, 1 Q, 3 Q and 1 Q, respectively.
2V Calculate (i) the potential difference between Band D and
3A
A
II C
(ii) the potential difference across the terminals of each of the
cells e and H.
H
[CBSE D 04C ; CBSE Sample Paper 08]
Fig. 3.154
2 x 2 + 3 Rl = 4 or Rl =0
O
D""-----~
Example 145. Determine the current in each branch of the
network shown in Fig. 3.155. [NCERT] Fig.3.156(a)
B
M
1 :~I---<II!-:-Il-H"""~2
1 VI
D
F I,
",,---
12
~:slll:
•.. _G-I
3V 3Q
C
(i) Applying Kirchhoff's first law at junction D, Applying Kirchhoff's second law to loop BCDB,
we get we get
I = II + 12 ... (i) (II-1 g )x2-(I-l1+1 . g )x3-1 g x4=0
Applying Kirchhoff's second law to loop ADBA, we 5I1 -31 -9Ig =0 ...(2)
get Applying Kirchhoff's second law to loop ADCEA,
2 1+ I + 2 II = 2 -1 we get
or 3I+2I1=1 ...(ii) 2(1 - II) + 3(I - II + Ig)+2I=2
Applying Kirchhoff's second law to loop DCBD or - 5 II + 71 + 3 Ig = 2 ...(3)
3 12 + 12 - 2 II =3 - 1 Adding (2) and (3),
R
or 412 -2I1 =2 ...(iii)
41-6Ig=2 ...(4)
On solving equations (i), (ii) and (iii), we get
1 6 5 Multiplying (1) by 5 and (2) by 3 and subtracting,
II = - 13 A, 12 = 13 A and I = 13 A we get
=a
SI
P.D. between the points Band D, -1 + 471g or
2 From (4),
VI = II x 2 = - V.
13 4 x 47Ig -6Ig =2 or 182 Ig = 2
(ii) PD. between the terminals of G (giving current), 2 1
6 I =-=-A.
V2 =
-
e-
I? r = 3 - - x 3 = 1.615 V
13
g 182 91
Example 148. The four arms of a Whetstone bridge
PD. between the terminals of H (taking current), (Fig. 3.158) have the following resistances:
V3 = e' + 12r' = 1 + ~ x 1 = 1.46 V.
IT
AB=100n, BC=10n, CD=5n and DA=60n.
13
A galvanometer of 15 n resistance is connected across
Example 147. In a Wheatstone bridge, p=ln, BD. Calculate the current through the galvanometer when a
Q =2 n, R =2 n, 5 =3 nand Rg = 4 n. Find the current potential difference ofl0 V is maintained across AC.
through the galvanometer in the unbalanced position of the [NCERT]
bridge, when a battery of2 V and internal resistance 2 n is
B
H
used.
Solution. The circuit for the given Wheatstone
bridge is shown in Fig. 3.157. Let I, II and Ig be the
currents as shown.
B
O
o
lOV
M
Fig. 3.158
R
On solving equations (i) and (ii), we get
Substituting the value of 12in Eq. (3), we get 5 18
13 (31.51g) + Ig =2 1[ = 146 A and 12 = 146 A
SI
2
or I = --- A =4.87 mA. I + I =2+~=E...A
g 410.5 1 2 146 146 146
Example 149. Two cells of emfs 1.5 V and 2 V and PD. across the ends of 8 0 resistance wire
internal resistances 2 0 and 10 respectively have their = E... x 8 = 1.26 V.
negative terminals joined by a wire of 60 and positive 146
terminals by a wire of 40 resistance. A third resistance wire
Example 150. AB, BC, CD and DA are resistorsof L, 1,2
of 80 connects middle points of these wires. Draw the
and2 0 respectively connected in series. Between A and Cis
circuit diagram. Using Kirchhoff laws, find the potential
a 1 volt cell of resistance 2 0, A being positive. Between B
IT
difference at the end of this third wire. [CBSE D 2000c]
and D is a 2 V cell af1 0 resistance, B being positive. Find
Solution. As shown in Fig. 3.159, the positive the current ill each branch of the circuit.
e e
terminals of cells 1 and 2 are connected to the wire
Solution. The circuit arrangement and current
AE of resistance 4 0 and negative terminals to the wire
distribution is shown in Fig. 3.160.
BD of resistance 6 O. The 80 wire is connected
between the middle points F and C of the wires AE and B
H
BD respectively.
4
Rl=~=2=20
6 A C
and R=R=-=30
342 20 I3 - II 20
O
RJ=30
Applying Kirchhoff's second law to loops BADB,
II BCDB and ADCEFA, we get
---"vv'\r--...•..
O- .•.... --{)C
[2 1 . 12 + 2 . 13 + 1. II =2
R4=30
or 11+12+213=2 ... (1}
or 1(/1-12}-2(13-Il}+11=2
or 4 II - 12- 2 13= 2 ...(2}
and 213+2(13-11}+2(13-12}=1
Fig. 3.159
or - 2 II - 2 12 + 6 13 = 1 ...(3}
Applying Kirchhoff's second law to the loop Solving equations (I), (2) and (3), we get
ABCF A, we get II =0.8 A, 12=0.2 A and 13 =0.5 A
3.86 PHYSICS-XII
Currents in different branches are Example 152. Two squares ABCD and BEFC have the
lAB = IZ = 0.2 A ; side BC in common. The sides are of conducting wires with
resistances as follows: AB, BE, FC and CD each 2 o ; AD,
IBC = II - IZ = 0.6 A; Be, EF each 1 n. A cell of emf 2 V and internal resistance
'co = II - 13 = 0.3 A ; 2 n is joined across AD. Find the currents in various
lAD = 13 = 0.5 A ; branches of the circuit. .
IEF = 13 - 1Z = 0.3 A. Solution. The current distribution in various branches
of the circuit is shown in Fig. 3.162.
Example 151 . Find the equivalent resistance between the
1 A 11 B 12
terminals A and B in the network shown in Fig. 3.161. r-~~-.~~~~-;~~vv~E
Given each resistor R is ofl0 n. 1 - 11
2Q
11 - 12
2Q
R
12
1 K 11 L 12
A~~~~~~~~~~~r-QM 2V 2Q
IQ IQ IQ
R R
1- 11 11-12 12 2Q 11 2Q 12
R
J R
J R 0 C
F
SI
1-11 1 - 12
P B Fig. 3.162
R o R N
U Applying Kirchhoff's second law to the loop
- - - - - - - - - - -11- - - - - - - - - - - - - - - - - - - - - ~ containing the cell and AD, we get
e 2 x I + 1 x (I - II) =2
Fig. 3.161 or 3 I - II = 2 ...(1)
From the loop ABCDA, we get
Solution. Imagine a battery of emf t, having no
2x II +lx(II-Iz)+2x II-1x(I-II)=0
IT
internal resistance, connected between the points A
and B. The distribution of current through various or - I + 6 II - Iz =0 ...(2)
branches is as shown in Fig. 3.161. Similarly, from the loop BEFCB, we get
Applying Kirchhoff's second law to loop KLOPK, 2 x Iz + 1 x Iz + 2 x Iz - 1 x (II - Iz) =0
we get or - II + 6 Iz =0 ...(3)
II R+(I1-Iz)R-2(I-II)R=O Solving equations (1), (2) and (3), we get
H
or 4 II - Iz =2 I ...(1)
I = 70 A I = 12 A I =~ A
Similarly, from the loop LMNOL, we have 99 ' 1 99 ' z 99
2IzR-(I-Iz)R-(II-Iz)R=0 Currents in different branches are
or - II + 4 Iz = I ... (2) 12 2
lAB = Ieo = II = 99 A, IBE = IEF = ICF = Iz = 99 A
O
c 61
I--+---~B
Fig. 3.163
R
(I - 11)200 + Ix 300 = 110
or 500 1-200 II = 110 ...(2)
E
Solving equations (1) and (2), we get
Fig. 3.164
3 1
I = 10 A and II = 5" A .. The equivalent resistance of the network is
SI
.'. Current through 100 0 resistance R' = Total emf = ~ = 5 IR =~ R
1 Total current 61 6I 6
= II =- A
5 But R =10
Current through 200 0 resistance
R'=~O
1 6
=I-Il=-A
10 Total current in the network is
Current through 300 0 resistance 10 e
= - = - = 12 A or I = 2 A
IT
6 I
=I=-2.A 5 R'
10 6
P.D. between A and C = PD. across 100 n resistor The current flowing in each branch can be read off
1 easily.
= II x 100 = - x 100 = 20 V
5 Example 155. Twelve wires each having a resistance of r 0
are connected toform a skeleton cube; find the resistance of
H
PD. between C and B = PD. across 300 0 resistor
the cube between the two corners of the same edge.
= I x 300 = 2.x 300 = 90 V.
10 Solution. Let a current x + 2 Y enter the junction A
Example 154. A battery of 10 V and negligible internal of the cube ABCDEFGH. From the symmetry of the
resistance is connected across the diagoYfallyopposite corners parallel paths, current distribution will be as shown in
O
R
points A and C so that a current of 1 A enters junction
or ( x + ~~ x ) R = xr or 12 R =r A. This current is divided equally along AB and AD.
7
The distribution of current in various branches is
7
Hence R=-rn. shown in Fig. 3.167.These currents finally add so that a
12 current of 1 A flows out of junction C.
SI
Example 156. Eleven equal wires each of resistance rform 1-2x-z
E H
the edges of an incomplete cube. Find the total resistance
I
from one end of the vacant edge of the cube to the other.
Solution. Let A and B be the vacant edges of the
,= y
B 1-2x-z+y
cube. Let an emf E. applied across AB send a current 2x X I
I
x-y
at B to give current 2x. Let R be the total resistance of
D
the cube between A and B.
Fig. 3.167
H
Applying Kirchhoff's second law to the loop
AEFDA, we get
x
x D~ ____ ~ - (1 - 2x) - z +y +x =0 ...(1)
C
X /
Y
O
A E. - y - (1- 2x - z + y) - (1- 2x + 2 y) + (x - y) =0
2x
...(2)
Again, from the loop FGCDF, we have
Fig. 3.166
M
-(y+z)-(1-2x+2y)+(x-y)-y=0 ...(3)
Applying Kirchhoff's second law to the loop,
On solving equations (1), (2) and (3), we get
ABCDA, we get
xr + yr + xr = E. 31·
x = 3" A, y =0, Z = 8' A
From Ohm's law,
E. =2x. R
Now VAC = VAB + VBC
2xr+ yr=2xR 3 3 6 3
=lx-+1x-=-=-V
Applying Kirchhoff's second law to the loop 8 8 8 4
EFGHE, we get Equivalent resistance between A and C,
yr -(x - y)r-2(x - y)r - (x - y)r =0 R - VAC .; 3/ 4 _ 3 o
--[---1--4 .
or y-x+y-2x+2y-x+y=0
CURRENT ELECTRICITY 3.89
Example 158. In the network as shown in Fig. 3.168, each Solution. Suppose a cell of emf is connected e
resistance r is of 2 n. Find the effective resistance between between A and B. Then the given circuit can be
points A and B. represented by an unbalanced Wheatstone bridge as
shown in Fig. 3.172. The distribution of current is also
shown.
Fig. 3.168 A B
R
Fig. 3.169. By symmetry, current in arm AE = current in
arm EB. As the current in arm CE is equal to the current
C 12 - 13 0
SI
Fig. 3.172
1 1 3 7 8r (3), we get
-=-+-=- or R=-
R 2 r 8r 8r 7 2
8 x 2 16 _ (R+r) 1+ (R-r) I+(R+r)l=e
Given r =2 n, therefore, R =-- =- n. R+3r R+3r
7 7
Example 159. Calculate the equivalent resistance between or 3rR+?I=e
the points A and B in the network shown in Fig. 3.171. R+3r
R Equivalent resistance between A and B,
R' =~ = 3r R +?
I R +3r
A B
r(3R + r)
Fig. 3.171 R r (R+3r)
3.90 PHYSICS-XII
~:=J
( 860 '215 '172 e e e en'n
L~~~---
l '1 2 '2 3'3
200n
R
5V
60n Fig.3.177
SI
4V parallel so as to send current in the same direction
through an external resistance of 5 Q.
2. Use Kirchhoff's rules to determine the value of the
(i) Draw the circuit diagram.
current 11 flowing in the circuit shown in Fig. 3.174.
[CBSE D 13C]
(ii) Using Kirchhoff's laws, calculate (a) the current
30n h through each branch of the circuit. (b) p.d.
(Ans. I} ": 0.75 A)
across the 5 Q resistance. [CBSE OD 95, 96 C]
11 20V
13 20n 13 (Ans. (a) ~17 A, ~17 A, ~17 A (b) 2.35 vJ
a d
b c
IT
12 7. In the electric network shown in Fig. 3.178, use
g f 20n Kirchhoff's rules to calculate the power consumed
e
12 by the resistance R = 4 Q. [CBSE D 14C]
80V
el
= 12V 'I = 2n (Ans. 9 W)
Fig.3.174 B I. C
II I'
24V
11
H
3. Using Kirchhoff's II + 12 R=4n
laws, determine the A 0
currents II' 12 and 13 27V 6n
for the network shown 13 e2~6 V
in Fig. 3.175. 4n E F
12 I'
O
[CBSE D 99C]
(Ans. 3 A, -1.5 A, 4.5 A) Fig.3.175 Fig.3.178
4. The circuit diagram shown in Fig. 3.176 has two 8. A network of resistors is connected to a battery of
cells e} and e2 with emfs 4 V and 2 V respectively, negligible internal resistance, as shown in Fig. 3.179.
each one having an internal resistance of 2 Q. The Calculate the equivalent resistance between the
M
external resistance R is of 8 Q. Find the magnitude points A and D, and the value of the current 13,
and direction of currents flowing through the two (Ans. 1.25Q, 0.5 A)
cells. [ISCE 98] 2 n 11- 13
B c
( Ans. 11 =~ A, 12 =- ~ AJ
2n
2n
R I=2A
~---+-I f-----J
Fig.3.176 Fig.3.179
CURRENT ELECTRICITY 3.91
r---I-: Fig.3.184
R
14. In the given electrical networks shown in
A B c Figs. 3.185(a) and (b), identical cells each of emf t,
9V 3V
are giving same current 1. Find the values of the
Fig. 3.180 resistors ~ and ~ in the network (b).
SI
10. Find the current flowing through each cell in the ( Ans. 9.9Q, ~1Q)
circuit shown in Fig. 3.181. Also calculate the
potential difference across the terminals of each
cell. (Ans. 0, - 3 A, 3 A, 3 V)
IT
Fig. 3.185
C 0
14V 5V
40
4Q 12
2V 4V II 2Q
B E
6Q
10V
M
4Q 13 Fig.3.186
E 50 B
6V
A F 16. In the circuit shown in
2Q Fig. 3.187, determine 5Q 50
the current in the
Fig. 3.182 Fig.3.183
resistance CD and C 0
equivalent resistance
12. In the network shown in Fig. 3.183, find (i) the 50 5Q
between the points A
currents II' 12 and 13 and (ii) the potential difference
and B. The internal
between the points B and E.
resistance of cell is F
[Ans. (i) II = 2 A, 12 = - 3 A, 13 = - 1A (ii) - 2 Vj negligible.
13. Calculate the potential difference between the (Ans. 7Q, 0.4 A) 14V
junctions Band D in the Wheatstone's bridge shown
in Fig. 3.184. [Roorkee 89] (Ans. 0.2 V) Fig.3.187
3.92 PHYSICS-XII
17. A certain length of a uniform wire of resistance 120 22. Twelve identical wires each of resistance 60 are
is bent into a circle and two points, a quarter of arranged to form a skeleton cube. A current of 40 mA
circumference apart, are connected to a battery of is led into cube at one comer and out at the diago-
emf 4 V and internal resistance 10. Find the current nally opposite comer. Calculate the potential diffe-
in the different parts of the circuit. rence developed across these comers and the effec-
tive resistance of the network. (Ans. 0.2 V, 50)
(Ans. 12 A ~ A)
13 '13 23. Twelve identical wires each of resistance 60 are
18. In Fig. 3.188, ABCDA is a uniform circular wire of joined to form a skeleton cube. Find the resistance
resistance 2 O. AOC and BOD are two wires along between the comers of the same edge of the cube.
two perpendicular diameters (Ans.3.50)
B
of the circle, each having 24. Find the currents II' 12 and 13 through the three
R
same resistance 10. A resistors of the circuit shown in Fig. 3.191.
battery of emf € and internal (Ans. Zero in each resistor)
resistance r is connected A 1----+---1 C
C D
between the points A and D. A ,----H--.----jll--,.--H-----,
Calculate the equivalent 3V
SI
resistance of the network. D
(Ans. 15
140) Fig. 3.188 Ion Ion Ion
I = 47 A I = 17 A I = 30 A 1 = _ J.. A) HINTS
IT
( Ans. 91' 1 91 '2 91 '3 91
1. By Kirchhoff's junction rule,
B
13 = II + 12 ...(i)
20. Determine the current flowing through the Applying Kirchhoff's loop rule to the loop ahdcba,
galvanometer G of the Wheatstone bridge shown in we get
Fig. 3.190. (Ans. 0.0454A) 3011 + 2013 = 20
B
or 3011 + 20(11+ 12) = 20
or 5011 + 2012 = 20
M
R
.. 11= I - 12= 2 - 1.25 = 0.75 A
13= ~ - 12= 3 + 1.5 = 4.5 A
From loop BCDB, we get
4. Applying Kirchhoff's second law to loop 1, we get
111 - 12'2= e e2l -
2 (II - 13) + 2 (II - 13) - 213 = 0
SI
or 11- 12= 1 ...(1) 4 4
or 13= "611="6 x 0.75 = 0.50 A
Similarly, from loop 2, we get
12'2 + (II + 12) R = e 2
9. Applying Kirchhoff's loop rule to the loop AFEBA,
(1+1)1+4xO=-6+9
or 212 + 8 (II + 12)~ 2 1= 1.5 A
or 411+ 512 =1 ...(2)
H2 E I
F D
On solving equations (1) and (2), we get
InJ 4nJ
IT
2
II = 3A R
e e e
11 + 1'2 + 1'3 + ... + lr., = 1 + 2 + 3 + ... + en VAD = (1+ 1)x1.5=3 V.
or
1= e1 + e2 + e2 + ... + en 10. Applying Kirchhoff's first law at the junction B, we
1 +'2+r3+"'+'n get
_ e1 + e2 + e3 + + en _ 1 ...(1)
M
1
= e i - I,; = e i - -.
a
a ei = O. 11xl- 12 x2=(10-4)
II - 2 12 = 6. ...(2)
8. Here RBCD = 2 + 2 = 4 n. It is in parallel with 2 n
resistance in BD. Their equivalent resistance Similarly, from the closed loop A~ BE3 A, we have
Thus, the current in the 10 V cell is zero. The Similarly, from the loop ADCEA, we have
current given by the 13 V cell to the circuit is 3 A, 12 x 1.5 + 12 x 1= 2
and the current taken by the 4 V cell from the circuit 2
is 3 A. 12 = -=0.8A
2.5
As there is no current in the 10 V cell, so the Potential difference between the points A and B is
potential difference across its ends is equal to its
VA - VB = 1.0 A x l 0 = 1.0 V
e.m.f. i.e., 10 V. Since all the three cells are in
Potential difference between the points A and D is
parallel, the potential difference across the
terminals of each is 10 V. VA - V D = 0.8 A x 1.50 = 1.2 V
11. (i) The distribution of current in various branches of :. Potential difference between the points B and 0 is
the circuit is shown in Fig. 3.193. VB-VD=(VA -VD)-(VA -VB)
R
A B = 1.2 - 1.0 = 0.2 V.
14. From the network of Fig. 3.194(a), E. = 11 I
2V 4V
SI
6V
Fig. 3.193
Fig. 3.194
Applying Kirchhoff's second law to loop 1,
In the network Fig. 3.194(b), the main current I
312 + (II - 12) = 2 I
passes through R,., a part 10 through the 110
or ~1 + 212 = 2 ... (i)
IT
From loop (2), we get .
resistor an d th e remmnmg
. . current, I - - I = -91
(II - 12) + 2 (II + 13) = 6 10 10
through the resistor Rz.
or 311 - 12 + 213 = 6 ... (ii)
From loop (3), we get Applying Kirchhoff's law to the loop 1, we get
413 + 2 (II + 13)= 4 I 9I 11
- x 11- - x Rz = 0 or Rz = - 0
H
or 211 + 613 = 4 10 10 9
Solving equations (1) and (2), 12 = ~ A If R' is the equivalent resistance of the network, then
11 l(r+R')=e
This will be the reading of the ammeter. 7 7
On interchanging the cell and the ammeter, the :. -12r+512R=I(r+R')=-I2(r+R')
3 3
circuit takes the form as shown in Fig. 3.196. Again,
we can show that or R' = 15 R = 15 x 0.5 = 15 o.
7 7 14
5
12 = 11 A. 19. Applying Kirchhoff's first law at the junction A,
I = II + 12 ...(i)
Applying Kirchhoff's second law to the loop ABDA,
we get
R
5V
4Q 211+ 413 - 12= 0 ...(ii)
6Q From the loop BCDB, we get
2Q
3 (II -13)-2(I2 + 13)-413 =0
SI
From the loop ABCEA, we get
16. Proceeding as in Example 151, we obtain equivalent 211+ 3 (II - 13) + 2 (II + 12)= 2
resistance between points A and B as
or 711+212-313=2 ...(iv)
R' = 'Z R = 'Z x 5 = 70 [.: R = 50] On solving equations (i), (ii) and (iii), we get
5 5
Main current, 17 30 1
II = 91 A, 12 = 91 A and 13 = - 91 A
I = ~ = 14 = 2 A
R 7 20. From the loop ABDA, we get
IT
Current through 50 resistance in arm CD 5~+101g-(I-II)15=0 [l=IA]
321 1 or 201I+lOIg=15
= II - 12= "5 1 - "5 I = "5 I ="5 x 2 = 0.4 A.
or 411 + 21g = 3 ...(i)
18. The current distribution is shown in Fig. 3.197. From the loop BCDB, we get
B 1O(II-1g)-20(1-1I + Ig)-10Ig=0
3011 - 401g = 20
H
or 311 - 41g = 2 ...(ii)
On solving equations (i) and (ii), we get
Ig = ~ A = 0.0454 A
22
O
Fig. 3.197 6
Total resistance of the circuit = 2.5 + 2.5 = 5.00
Applying Kirchhoff's law to different loops, we get
Current,
R(I-II)+ R(I-I2)+(R+r)I=e (1)
I= emf = 2 = 0.6 A.
RII + R (II - 13) - R (I - II) = 0 (2) total resistance 5.0
RI3 - R (I2 - 13) - R (II - 13) = 0 (3) 22. Proceeding as in Example 154, we obtain the
RI2 - R (I - 12) + R (I2 - 13) = 0 (4)
effective resistance, R' =~ R
On simplifying and solving these equations, 6
5x6
·27 But R = 60, therefore, R' = -- = 50
11= 12, 13 = '3 12, 1='3 12 6
P.D. developed = Resistance x Current
7
and - 12r + 5 12R =
3
e = 5 x(40 x 10-3) = 0.2 V.
3.96 PHYSICS-XII
23. Proceeding as in Example 155, we obtain effective Principle. The basic principle of a potentiometer is that
.
resistance, R = -7 r when a constant current flows through a wire of uniform
12 cross-sectionalarea and composition, the potential drop across
7x6 any length of the wire is directly proportional to that length.
But r = 60, therefore, R = -- = 3.5 o.
12 In Fig. 3.198, if we connect a voltmeter between the
24. From the loop ABGHA, we get end A and the jockey J, it reads the potential difference
lOll = 3 - 3 or II = o. V across the length Iof the wire AJ. By Ohm's law,
From the loop BCFG8, we get
1012 - lOll = 3 - 3 or 12 = O.
V = IR = l. p~ [-: R =p ~J
From the loop CDEFC, we get For a wire of uniform cross-section and uniform
R
1013 - 1012 = 3 - 3 or 13 = O. composition, resistivity p and area of cross-section A
are constants. Therefore, when a steady current Iflows
3.32 POTENTIOMETER through the wire,
SI
and principle.
Potentiometer. An ideal voltmeter which does not Hence V =k I or V oc I
change the original potential difference, needs to have This is the principle of a potentiometer. A graph
infinite resistance. But a voltmeter cannot be designed drawn between V and Iwill be a straight line passing
to have an infinite resistance. Potentiometer is one through the origin 0, as shown in Fig. 3.199.
such device which does not draw any current from the
circuit and still measures the potential difference. So it v
acts as an ideal voltmeter.
IT
A potentiometer is a device used to measure an unknown
emf or potential difference accurately.
Construction. As shown in Fig. 3.198, a potentio-
meter consists of a long wire AB of uniform cross-
section, usually 4 to 10 m long, of material having high
resistivity and low temperature coefficient such as Fig. 3.199 Potential drop V oc length I
H
constantan or manganin. Usually, 1 m long separate
pieces of wire are fixed on a wooden board parallel to Potential gradient. The potential drop per unit length
each other. The wires are joined in series by thick of the potentiometer wire is known as potential gradient. It is
copper strips. A metre scale is fixed parallel to the given by
wires. The ends A and B are connected to a strong k= V
O
K·
r- 300
shows the circuit diagram for comparing the ernfs of
two cells. A constant current is maintained in the
potentiometer wire AB by means of a battery of emf E
through a key K and rheostat Rh. Let E1 and E2 be the
Rh ernfs of the two primary cells which are to be
B 400
1"'!!!!lI""I"III""llIrd""I""I",,lr, compared. The positive terminals of these cells are
connected to the end A of the potentiometer wire and
Fig. 3.198 Principle of a potentiometer. their negative terminals are connected to a high
CURRENT ELECTRICITY 3.97
K1 •
1-
Rh B
R
Fig. 3.201 To determine the internal resistance
of a cell by a potentiometer.
SI
the potentiometer wire. With key Kz kept open, move
Fig. 3.200 Comparing emfs of two cells by a the jockey along ABtill it balances the emf E, of the cell.
potentiometer. Let 11 be the balancing length of the wire. If k is the
potential gradient, then emf of the cell will be
As the plug is inserted between a and c , the cell i\ E, = kl1
gets introduced in the circuit. The jockey I is moved
along the wire AB till the galvanometer shows no With the help of resistance box RB., introduce a
deflection. Let the position of the jockey be I] and resistance R and close key Kz. Find the balance point
IT
length of wire AI1 = 11, If k is the potential gradient for the terminal potential difference V of the cell. If 12is
along the wire AB, then at null point, the balancing length, then
potentiometer? How can we increase the sensitivity of a 2. For measuring internal resistance of a cell,
potentiometer? r = ~ -12 x R
A potentiometer is Lz
R
Sensitivity of a potentiometer.
sensitive if 3. Potential gradient of the potentiometer wire,
(i) it is capable of measuring very small potential
k= V
differences, and I
(ii) it shows a significant change in balancing length for 4. Unknown emf balanced against length I, e =k I
SI
a small changein thepotentialdifferencebeing measured.
Units Used
The sensitivity of a potentiometer depends on the
potential gradient along its wire. Smaller the potential e e
The emfs 1 and 2 are in volt, lengths ~ and 12 of
gradient, greater will be the sensitivity of the potentiometer. potentiometer wire in metre.
(ii) For a potentiometer wire of fixed length, the Current through the potentiometer wire,
potential gradient can be decreased by reducing
the current in the circuit with the help of a rheostat. 1= _e_ =_5_ =2. =.!. A
R+r 18 +2 20 4
H
For Your Knowledge
:. Potential gradient = IR =.!. x 18 = 0.45 Vrn -1.
~ A potentiometer can be regarded as an ideal voltmeter 1410
with infinite resistance because it does not draw any Example 161 . A potentiometer wire is supplied a constant
current from the source of emf at the null point. voltage of 3 V. A cell of emf108 V is balanced by the voltage
O
~ The principle of potentiometer requires that (i) the drop across 216 em of the wire. Find the total length of the
potentiometer wire should be of uniform cross- potentiometer wire.
section and (ii) the current through the wire should
remain constant.
Solution. Here e
=3 v.s, =1.08 V, 11 =216 ern, I =?
1 1
~ The balance point cannot be obtained on the potentio-
meter if the fall of potential along the potentiometer
Example 162. Two cells of emfs e1 and e2 (e1 > e2) are
connected as shown in Fig. 3.202.
wire due to the auxiliary battery is less than the emf of
the cell to be measured.
~~~
~ The positive terminals of the auxiliary battery and the e, e,
cell whose emf is to be determined must be connected
Fig. 3.202
to the zero end of the potentiometer.
~ Other uses of a potentiometer. Any physical quantity When a potentiometer is connected between A and B, the
that can produce or control a potential difference can balancing length of the potentiometer wire is 300 em. On
be measured using a potentiometer. Thus, a potentio- connecting the same potentiometer between A and C, the
meter can be used to measure and control stress, balancing length is 100 em. Calculate the ratio of 1 and 2. e e
temperature, radiation, pH, frequency, etc. [CBSE D94]
CURRENT ELECTRICITY 3.99
R
[CBSE D 04C)
or -=1--=- or ~---3'
E.2 - 2 - .. 2
E.1 3 3 Solution. (i) The circuit diagram is shown in
Fig. 3.204.
Example 163. In Fig. 3.203, a long uniform potentiometer
wire AB is having a constant potential gradient along its E.=5V
length. The null points for the two primary cells of emfs E.1 I----{.
SI
and E.2 connected in the manner shown are obtained at a
distance of 120 em and 300 cmfrom the end A. Find (i) 1 / 2 e e
and (ii) position of null point for the cell E.1. AQ----,---~B
Fig. 3.204
IT
(ii) Here k = 5 V = 5 V = _1_ V cm-1
t---- 300ern 5m 500 em 100
120 ern 1 In first case,
A B 1
E.1+ E.2= kl1 =- x 350
100
or E.1+ E.2= 3.50 V ...(i)
H
In second case,
1
Fig. 3.203 E.1-E.2 = kI2 =- x 50 =0.50 V ...(ii)
100
On solving (i) and (ii), we get
O
1 _ 7
Hence, emf E. is balanced at 600 em of this wire, calculate (i) the
e 2
- 3
potential gradient of the potentiometer wire and (ii) the
value of the unknown emfe. [CBSE D 06]
(ii) As E.1= 210k
5V 480Q
.'. Balancing length for cell E.1is
E. II R
11 =~=210 em
]
k
600 ern ~I
The sensitivity of a potentiometer wire can be AO-----------.-----O B
increased by decreasing potential gradient either
through increasing length of the potentiometer wire or
through increasing resistance put in series with the
E.
main cell. Fig. 3.205
3.100 PHYSICS-XII
Solution. Current in the circuit or through the Example 1 67. The length of a potentiometer wire is 5 m It
potentiometer wire is is connected to a battery of constant emf For a given
1= V 5V =0.01 A
Leclanche cell, the position of zero galvanometer deflection is
RAB +R (20 + 480)0 obtained at 100 em If the length of the potentiometer wire be
made 8 m instead of 5 m, calculate the length of wire for zero
Resistance of potentiometer wire, deflection in the galvanometer for the same cell.
=200
RAB [CBSE F 97]
R
1= 10 m =1,000 em In first case, E, = IR 11 ...(1)
:. Potential gradient, I
SI
Unknown emf balanced against 600 cm length is Comparing equations (1) and (2),
1= E,1 _2_
O
R + RAB R + 15
Resistance of the 30 em length of wire, which
A~---7r-------6B
balances the emf E,2'is
R' = ~ x 30 = 4.5 0
100
M
As the length of the potentiometer wire is just The same current flows through the potentio-
100 em, so this balance point cannot be obtained on the meter wire and through the external resistance R.
wire. Total resistance = (R + 10) 0
CURRENT ELECTRICITY 3.101
R
cell, its terminal p.d. is balanced by 150 em of potentiometer
wire and when a resistor of 100 resistance is connected
across the cell, the terminal p.d. is balanced by 175 em of the
potentiometer wire. Find the internal resistance of the cell.
= Rl J
SI
Solution. In the first case, r ( I~11
I
r _1 = 1-11 ...(1)
Rl
Fig. 3.208
In the second case,
Solution. (i) Total resistance of the primary circuit
= 15+ 10=250, emf = 2 V r= RzC ~212 J
:.Current in the wire AB,
IT
I
I = 3..- =0.08 A
r-.L=1-1 ...(2)
2
Rz
25
P.D. across the wire AB Subtracting (2) from (1),
= Current x resistance of wire AB
= 0.08 x 10 =O.B V r[ ~1 - ~]=I-II-I+12
H
Potential gradient
r= 12-11 = 175 -150 = 25 =20
= P.D. =~=0.008Vem-l. ~ _.!L 150 _ 175 12.5 .
length 100 Rl Rz 5 10
(ii) Resistance of secondary circuit
O
cells is 1.5, calculate the difference in the balancing 11. A standard cell of emf 1.08 V is balanced by the
length of the potentiometer wire in the two cases. potential difference across 91 cm of a metre long
[CBSE D 06C] (Ans. 10 em) wire supplied by a cell of emf 2 V through a series
e e
4. Two cells of emfs 1 and 2 are connected together resistor of resistance 2 O. The internal resistance of
the cell is zero. Find the resistance per unit length of
in two ways shown here. The 'balance points' in a
given potentiometer experiment for these two com- the potentiometer wire. (Ans. 0.030 em-I)
12. Potentiometer wire PQof 1 m length is connected to
e e
a standard cell 1. Another cell, 2, of emf 1.02 V is
binations of cells are found to be at 351.0 em and connected as shown in the circuit diagram with a
70.2 em respectively. Calculate the ratio of the emfs resistance 'r' and a switch,S. With switch 5 open,
of the two cells. [CBSE Sample Paper 08] (Ans. 3 : 2) null position is obtained at a distance of 51 em from
R
P. Calculate (i) potential gradient of the potentio-
5. A potentiometer has 400 em long wire which is
connected to an auxiliary of steady voltage 4 V. A
e
meter wire and (ii) emf of the cell 1. (iii) When
switch 5is closed, will null point move towards P or
Leclanche cell gives null point at 140 em and Daniel
towards Q? Give reason. [CBSE OD 04]
cell at 100 cm. (i) Compare emfs of the two cells.
(ii) If the length of wire is increased by 100 ern, find (Ans. 0.02 V em - \ 2 V, no effect)
SI
the position of the null point with the first cell.
[Ans. (i) 7 : 5, (ii) 175 em]
6. With a certain cell, the balance point is obtained at
60 cm from the zero end of the potentiometer wire.
With another cell whose emf differs from that of the P~------------.---~Q
first cell by 0.1 V, the balance point is obtained at
55 ern mark. Calculate the emf of the two cells.
(Ans. 1.2 V, 1.1 V)
IT
7. A potentiometer wire has a potential gradient of
Fig. 3.210
0.0025volt/em along its length. Calculate the length
of the wire at which null-point is obtained for a
1.025 volt standard cell. Also, find the emf of e
13. A battery 1 of 4 V and a variable resistance Rh are
another cell for which the null-point is obtained at connected in series with the wire AB of the
860 cm length. (Ans. 410 em, 2.15 V) potentiometer. The length of the wire of the
H
8. AB is a potentiometer wire of length 100 em. When
potentiometer is 1 metre. When a cell 2 of emf e
1.5 volt is connected between points A and C, no
e
a cell 2 is connected across AC, where AC = 75em,
e
current flows through 2. Length of AC = 60em.
no current flows from e2. Find (I) the potential
gradient along AB and (ii) emf of the cell 2. The e (i) Find the potential difference between the ends
e A and Bof the potentiometer.
O
Rh
Fig. 3.209
4V
9. A cell can be balanced against 110 ern and 100em of ~-----100cm-----~
C
potentiometer wire respectively when in open circuit A~------------~------~B
and in circuit shorted through a resistance of 10O.
Find the internal resistance of the cell. (Ans. 10)
10. A potentiometer wire of length 1 m has a resistance
of 10O. It is connected to a 6 V battery in series with
Fig. 3.211
a resistance of 5 O. Determine the emf of the
primary cell which gives a balance point at 40 em. 14. The potentiometer wire of length 200em has a
[CBSE D 14] resistance of 20 O. It is connected in series with a
(Ans. 1.6 V) resistance 100 and an accumulator of emf 6 V
CURRENT ELECTRICITY 3.103
having negligible resistance. A source of 2.4 V is (ii) Let e
be the emf of the auxiliary battery and Ibe
balanced against a length r; of the potentiometer
t the length of potentiometer wire. Then = 4 V and e
wire. Find the value of L [CBSE F 03) [= 400 em.
(Ans. 120 cm)
..
e, ~ or e
""[=[
i = 140 =2
R'=10Q K 4 400 20
~--J""f\r---i r----{ • .. e = 1.4 V
i
6V
When length is increased by 100 em, new length,
t.-----L~ l' = 400 + 100 = 500 em
A~----~rC~--~B
e i _~ 1.4 _ ~
Now ""[-i' or 4- 500
R
1.4 x 500
2.4 V :. New balancing length, ~ = = 175 em.
4
Fig. 3.212
6. Let the emf of the two cells be e and e - 0.1 Then
15. A potentiometer wire carries a steady current. The 60 e e =1.2 V.
potential difference across 70 em length of it balances 0.1 = 55e-
SI
the potential difference across a 2 0 coil supplied by
emf of the other cell = 12 - 0.1 = 1.1 V.
a cell of emf 2.0 V and an unknown internal
resistance r. When a 10 coil is placed in parallel 7. (i) I = ~ = 1025 = 410 em.
with the 20 coil, a length equal to 50 cm of the k 0.0025
potentiometer wire is required to balance the (ii) e' = kl' = 0.0025 x 860 = 2.15 V.
potential difference across the parallel combi- . ~ 2V 4
8. (1) k=-=--=0.02Vem .
nation. Find the value of r. (Ans. 0.5 0) ~ 100cm
(ii) e2 = kl2 = 0.02 x 75 = 1.5 V.
IT
HINTS
2. Resistance of the potentiometer wire, R = 200
Length of the potentiometer wire = 10 m = 104 mm
9. r = R(~ -/ J 12
2 = 10(110 -100) =10.
100
Required potential gradient, k = 1flV mm - 1
10. I = V 6V = 0.4 A
Potential drop along the potentiometer wire, RAE +R (10+ 5)0
V = kl = 10 flV mm-I x104mm = 104flV = 1O-2V
H
V = IRAE = 0.4 x 10 = 4.0 V
Current through the potentiometer wire,
2 k = V = 4.0 V = 4.0 V = 0.04 V em-I
I = V = 10- = 5 x 10-4A I 1m 100cm
R 20
If R' is the required resistance to be introduced in Unknown emf balanced against 40 em of the wire,
e =kl'
O
.:»: R+ R'
or 5xl0-4 = __ 2_
20+ R'
11. Let r ohm be the resistance
potentiometer wire. Then
per ern of the
3. Here i = 120 V, ~ = 30 em
As the emf of 1.08 V balances against a length of
Also .s. = i = 15 91 em, so
e2 12 k = 1.08 V em-I:. 2 x 100 r 108
I = l. = 30 = 20 em 91 100 (2 + 100 r) 91
2 15 15 On solving, r = 0.029::::'0.03 0 em -1.
Difference in the balancing lengths,
~ - 12 = 30 - 20 = 10 em. 12. (i) k=J
e 102 V
=--
1
= 0.02 V em- .
12 51em
4. Proceed as in Example 162 on page 3.98.
5. (i) Here ~ = 140cm, 12 = 100 em
(ii) ei = klPQ = 0.02 V cm -1 x 100 em = 2 V.
(iii) With switch S closed, the null point is not
.. e = i = 140 = Z = 7: 5.
i affected because no current flows through the
e, 12 100 5 e
cell 2 at the null point.
3.104 PHYSICS-XII
R
V AB = lAB RAB = 0.2 x 20 = 4 V Q, Rand S ; connected to form the arms of a
Potential gradient,
quadrilateral ABCD. A battery of emf e
is connected
between points ,A and C and a sensitive galvanometer
k= VAB'=~ = 0.02 V ern-I between Band D, as shown in Fig. 3.213.
1 200crn
SI
Let S be the resistance to be measured. The
. 1 h Potential difference resistance R is so adjusted that there is no deflection in
Ba 1ancmg engt , L = -------
Potential gradient the galvanometer. The bridge is said to balanced when
2.4 V the potential difference across the galvanometer is zero
----'1 = 120 ern. so that there is no current through the galvanometer.
0.02 V ern
In the balanced condition of the bridge,
15. In first case. Current sent by the 2.0 V cell through
2n coil, P R
-=-
I = e 2
Q S
IT
1 Total resistance 2+ r Unknown resistance,S = Q.R
P
Potential drop across 2 n coil,
Knowing the ratio of resistances P and Q, and the
2, 4
VI =Rll =2x--=-- resistance R, we can determine the unknown
2+r 2+r
resistance S. That is why the arms containing the
But VI ocZOern resistances P and Q are called ratio arms, the arm AD
H
4 containing R standard arm and the arm CD containing 5
--oc70 ...(i) the unknown arm.
2+r
B
In second case. The combined resistance of the
parallel combination of 2n and In coil,
O
2xl 2
~=2+1=3n
where G is the resistance of the galvanometer. Again (iii) The unknown resistance can be measured to a
applying Kirchhoff's second law to the loop BCDB, we very high degree of accuracy by increasing the
get ratio of the resistances in arms P and Q.
(II - Ig) Q -(I2 + Ig) 5 - GIg =0
For Your Knowledge
In the balanced condition of the bridge, I g = O. The
above equations become ~ When the Wheatstone bridge is balanced, the po-
IIP-I2R=0 or IIP=I2R (i} tential difference between the points B and D is zero.
~ The Wheatstone bridge is most sensitive when the
and IIQ - I2S =0 or IIQ = 125 (il)
resistances in the four arms are of the same order.
On dividing equation (i) by equation (ii), we get
R
~ Wheatstone bridge method is not suitable for the
P R measurement of very low and very high resistances.
Q 5 ~ In the balanced Wheatstone bridge, the resistance in
This proves the condition for the balanced arm BD is ineffective. The equivalent resistance of the
Wheatstone bridge. balanced Wheatstone bridge between the points A
SI
and C will be
63. What do you mean by sensitivity of a Wheatstone
R = (P + Q) (R + 5)
bridge? On what factors does it depend ?
eq P+Q+R+S
Sensitivity of a Wheatstone bridge. A Wheatstone
bridge is said to be sensitive if it shows a large deflection in ~ If the bridge is balanced, then on interchanging the
the galvanometer for a small change of resistance in the positions of the galvanometer and the battery there is
resistance arm. no effect on the balance of the bridge. That is why the
The sensitivity of the Wheatstone bridge depends arms BD and AC are called conjugate arms of the
on two factors : bridge.
IT
(i) Relative magnitudes of the resistances in the ~ The Wheatstone bridge is the simplest example of an
four arms of the bridge. The bridge is most arrangement, the variants of which are used for a
sensitive when all the four resistances are of the large number of electrical measurements. The
same order. important applications of Wheatstone bridge are
metre bridge, Carey-Faster's bridge and post office box.
(ii) Relative positions of battery and galvanometer.
H
According to Callender for the greater sensitivity of
3.36 METRE BRIDGE OR SLIDE WIRE BRIDGE
the Wheatstone bridge, the battery should be so connec-
ted that the resistance in series with the resistance to be 65. What is a metre bridge? With the help of a circuit
measured is greater than the resistance in parallel with it. diagram, explain how it can be used to find an unknown
resistance. Explain the principle of the experiment and
O
According to Maxwell for the greater sensitivity of give the formula used.
the Wheatstone bridge, out of the battery and the galvano- Metre bridge or slide wire bridge. It is the simplest
meter, the one having the higher resistance should be practical application of the Wheatstone bridge that is used to
connected between the junction of the two highest resistances measure an unknown resistance.
and the junction of the two lowest resistances. Principle. Its working is based on the principle of
M
R
at 1 em from the zero end, then
C
!.. = B. = _1_ or X = (100 - I) R
Q X 100-1 1
Units Used
SI
Fig. 3.214 Measurement of unknown resistance
by a metre bridge. All resistances are in ohm and distances in em.
Working. After taking out a suitable resistance R Example 172. Find out the magnitude of resistance X in
from the resistance box, the jockey is moved along the the circuit shown in Fig. 3.215, when no current flows
wire AC till there is no deflection in the galvanometer. through the 5 Q resistor. [ISCE 98]
cr I I or X = - x 18 = 6 Q.
18 6 6
o (100 -I) 100 -I
Example 173. P, Q, Rand 5 are four resistance wires of
where o is the resistance per unit length of the wire. resistances 2, 2, 2 and 3 ohms respectively. Find out the
Hence resistance with which 5 must be shunted in order that bridge
R I may be balanced.
M
Example 174. In a Wheatstone bridge arrangement, the In circuit 2, the interchange of the positions of the
ratio arms P and Q are nearly equal. The bridge is balanced battery and the galvanometer does not affect the
when R = 500 O. On interchanging P and Q, the value ofR balance condition of the Wheatstone bridge, so
for balancing is 505 O. Find the value of X and the ratio
PIQ. ~=~
12 8
Solution. For balanced Wheatstone bridge,
P R ~ = 6x8 =40
-=- 12
Q X
~=~=~
In the first case, R = 500 0
~ 4 2
P 500
R
-=- ...(1) = 3: 2
Q X
Example 1 76. Calculate the current drawn from the
In the second case when P and Q are interchanged,
battery by the network of resistors shown in Fig. 3.217.
R =5050 rCBSE OD 09, ISC]
Q = 505 2n
SI
...(2)
P X
Multiplying equations (1) and (2),
1 = 500 x 505
X2
2n
or X = ~500 x 505
4V
= 502.50
IT
Substituting the value of X in (1), we get Fig. 3.217
4V
Fig. 3.218
10 20 P R
Now --- i.e., -=-
20 40 Q 5
M
Resistances in AB and BC are in series, their Example 178. A potential difference of 2 V is applied
equivalent resistance = 1 + 2 = 3 n. between the points A and B shown in network drawn in
Resistances in AO and OC are in series, their Fig. 3.222. Calculate
equivalent resistance = 2 + 4 = 6 n
c
The resistances of 3 nand 6 n are in parallel.
The equivalent resistance R between A and C is
3x6
R=--=2n
. 3+6
V 4
Current, I=- = - = 2 A.
R 2
R
Example 177. Each of the resistances in the network D~----~~v-----~E
shown in Fig. 3.220 equals R. Find the resistance between
two terminals A and C. Fig. 3.222
D
(i) the equivalent resistance of the network between the
SI
points A and B, and
(ii) the magnitudes of currents flowing in the arms
AFCEB and AFOEB. [eBSE OD 981
Solution. (i) The equivalent network is shown in
Fig. 3.223. It is a balanced Wheatstone bridge because
2n 2n
-=-
2n 2n
IT
Fig. 3.220 c
D
H
D
A c Fig. 3.223
B
network reduces to the equivalent circuit shown in
Fig. 3.221 Fig. 3.224.
HI C 2Q
It is a balanced Wheatstone bridge because
R R
M
A B
2Q 2Q
R R
D
Hence the points Band 0 must be at the same
potential. The resistance R in arm BO is ineffective. Fig. 3.224
= 2Rx2R =Rn. = 4x 4 =2 n
2R+2R 4+4
CURRENT ELECTRICITY 3.109
Example 179. Find the value of the unknown resistance X, Fig. 3.227
in the following circuit, if no current flows through the
section AO. Also calculate the current drawn by the circuit Solution. The equivalent circuits are shown below.
from the battery of emf 6 V and negligible internal The resistance R in arm DE of the balanced Wheatstone
R
resistance. (Fig. 3.225) [CBSEOD 02] bridge is ineffective.
o
A
A R R
SI
R R
E
R
o R
B 1t<---U-...J\fV\.,.---"" C A B A B
(a) (b)
6V 2.4Q
6V
Fig. 3.228
IT
Fig. 3.225 Fig. 3.226 The equivalent resistance R' across AB is given by
1 1 1 1 4 2
Solution. The equivalent circuit for the given -=-+-+-=-=-
R' 2R 2R R 2R R
network is shown in Fig. 3.226.
or R' = R/2
As no current flows through the section AO, so the
e . e 2e
given circuit is a balanced Wheatstone bridge. Current through arm AB = - = -- =- .
H
R' R/2 R
Hence
Current through arm DE = o.
2 3 3x 4
or X=--.=6n Example 181. Calculate the ratio of the heat produced in
4 X 2 thefour arms of the Wheatstone bridge shown in Fig. 3.229.
. 40n 60n
O
Hence the ratio of the heats produced in the four X 1-20 1-20
...(2)
arms is 22.5 100 - (l - 20) 120 -1
HAB: HBC: HAD: HDC Dividing (1) by (2),
= 90 Ii t :22.5 Ii t : 60 Ii t: 15 Ii t 22.5 I 120-1
--=--x---
= 90 :22.5 :60 : 15 = 6: 1.5 : 4 : 1. 10 100 -1 I - 20
Example 182. In the following circuit, a metre bridge is On solving, we get 12-1201 + 3600 =0
shown in its balanced state. The metre bridge wire has a I =60 em
resistance of 10 em Calculate the value of the unknown From (1),
resistance X and the current drawn from the battery of X 60 60 x 10
negligible internal resistance. or X=---=150.
R
10 100 -60 40
Solution. In balanced condition, no current flows
through the galvanometer. Example 184.In metre bridge, the null point is found at a
distance of 60.0 em from A. If now a resistance of 50 is
Here P = Resistance of wire AJ = 40 0
connected in series with 5, the null point occurs at 50 em.
Q = Resistance of wire BJ = 60 0 Determine the values of Rand 5. [Fig. 3.231]. [CBSE D 10]
SI
R =X, 5=60
x
In the balanced con-
dition,
P R A B
---
Q 5 / A·~~------+- -r~B
40 X jlj'l'i1j'j,j,jij,j'jijijiiijljijljIPi'lIlijijijijljijijij'I'il
or -=- 6V
60 6 Fig. 3.231
IT
Fig. 3.230
or X=40
Total resistance of wire AB = 1000 Solution. In first case, -
R
= -60 = -3 ... (i)
5 40 2
Total resistance of resistances X and 6 0 connec-
ted in series = 4 + 6 = 100 R 50
In second case, -- =- ... (ii)
This series combination is in parallel with wire AB. 5+5 50
H
10 x 100 100 On diIV!'din g ("b Y(..) 5+5_3- 2
us, -5-
Equivalent resistance = =- 0 IJ
10 + 100 11
or 25+10=35
emf of the battery =6 V
m 5=WO
.'. Current drawn from the battery,
O
and R= ~ 5 = ~ x 10 = 15 0
I= emf =_6_=O.66A. 2 2
resistance 100/11
Example 185. In a metre bridge, the null point is found at
Example 183. With a certain resistance in the left gap of a a distance of 40 cm from A. If a resistance of 12 0 is
slide wire metre bridge, the balance point is obtained when a connected in parallel with 5, the null point occurs at 50.0 em
resistance of10 0 is taken out from the resistance box. On from A. Determine the values of Rand 5. [Fig. 3.232]
M
increasing the resistancefrom the resistance box by 12.5 0, the [CBSE D 10]
balance point shifts by 20 em. Find the unknown resistance.
1Hl
Solution. With unknown resistance X in the left .J\IV'y,
I
gap and known resistance of 100 in the right gap, I
R
3 3
Example 186. A resistance R =2 n is connected to one of Fig. 3.234
the gaps in a metre bridge, which uses a wire of length 1m.
Solution. In first case,
An unknown resistance X > 2 n is connected in the other gap
X 60 X 3
as shown in thefigure. The balancepoint is noticed at '1' from or
SI
the positive end of the battery. On interchanging R and X, it Y 40 Y 2
isfound that the balance point further shifts by 20 em (away In second case,
from end A). Neglecting the end correction, calculate the _X_=50=1
value of unknown resistance X used. [CBSE OD OS] Y + 15 50
X
X x Y + 15 = ~x 1
Y X 2
or 1+ 15 = ~
Y 2
IT
A~~~~~----------~B or Y=30n
-- (100 - /) em
X=~Y=~x30=45n
2 2
When a resistance of 30 n is connected in parallel
Fig. 3.233 with Y, the resistance in the right gap becomes
H
Solution. In first case, Y' = 30Y = 30 x 30 = 15 n
R I 30+Y 30+30
- --
X 100-1 Suppose the null point occurs at I em from end A
Then
In second case,
O
X I 45 I
X 1+20 1+20 or -=--
15 100 -I 15 100-1
R 100 -(I + 20) 80 -I
or 300-31=1
On multiplying the two equations,
or 41 = 300 or 1=75 em.
l=_I_x 1+20
M
Now X = I + 20 R = 40 + 20 x 2 = 3 n.
80-1 80-40
Example 187. The given figure shows the experimental
A B
setup of a metre bridge. The null point is found to be 60 em
IIj'jll'P,·I'lIl'lIl'lIiilijilijijljljljiltjljiiii'r'illliiiilj
away from the end A with X and Y in position as shown.
When a resistance of 15 n is connected in series with Y, the Fig. 3.235
3.112 PHYSICS-XII
the metre bridge wire. An unknown resistance X is now 4. Calculate the equivalent resistance between points
connected in parallel to the resistance 5 and the balance point A and B of the network shown in Fig. 3.238.
is now found at a distance 12from the zero end of the metre [CBSE 0 99] (Ans. 2 Q)
bridge wire. Obtain aformula for X in terms of 11,12and S.
[CBSE 0 04C, IOC ; 00 09]
R
XS/(X+S) 100-12
5. Calculate the equivalent resistance between the
Dividing (ii) by (i), we get points A and B of the network shown in Fig. 3.239.
(Ans. R)
X+5 =~ ( 100 -11 J or
X 11 100 -12
SI
X= 5
~ ( 100 -11 J-1
11 100 -12
Fig. 3.239
problems ForPractice
1. Four resistances of 15Q, 12Q, 4 Q and 10Q respec- 6. Calculate the resistance between the points A and B
tively are connected in cyclic order to form a of the network shown in Fig. 3.240. (Ans. 8 Q)
IT
Wheatstone bridge. Is the network balanced ? If
not, calculate the resistance to be connected in
parallel with the resistance of 10Q to balance the
network. (Ans. Bridge is not balanced, 10 Q)
value of R.
through the branch AO is zero. (Ans. 6 Q, 0.5 A)
(Ans.25Q)
A
Fig. 3.236
3. (i) Calculate the equivalent resistance of the given
electrical network between points A and B.
M
9. The potentiometer wire AB shown in Fig. 3.243 is 14. In a metre bridge, the length of the wire is 100 em.
50 cm long. When AD = 30 em, no deflection occurs At what position will the balance point be obtained
in the galvanometer. Find R. (Ans. 40) if the two resistances are in the ratio 2 : 3 ?
(Ans. 40 em)
x 15. In the metre bridge experimental set up, shown in
Fig. 3.248, the null point' D' is obtained at a distance
6Q R
of 40 cm from end A of the metre bridge wire. If a
B
resistance of 100 is connected in series with X, null
J point is obtained at AD = 60 cm. Calculate the
100 em
values of X and Y. [CBSE D 13]
(Ans. 80,120)
R
A 0 B 5V 16. In a metre-bridge experiment, two resistances P
Fig. 3.243 Fig. 3.244 and Q are connected in series in the left gap. When
the resistance in the right gap is 500, the balance
10. Calculate the value of unknown resistance X and point is at the centre of the slide wire. If P and Q are
the current drawn by the circuit, assuming that no connected in parallel in the left gap, the resistance
SI
current flows through the galvanometer. Assume in the right gap has to be changed to 120 so as to
the resistance per unit length of the wire AB to be obtain the balance point at the same position. Find
0.010/ cm. (Fig. 3.244) [CBSE D 01] P and Q. (Ans. P= 300, Q = 200)
(Ans. 60, 5.5 A) 17. In a metre bridge when the resistance in the left gap
11. In Fig 3.245, P = 30, Q = 20, R = 60, S =40 and is 20 and an unknown resistance in the right gap,
X = 5 n Calculate the current l. [CBSE D 921 the balance point is obtained at 40 cm from the zero
end. On shunting the unknown resistance with 20,
(Ans. 0.6 A)
find the shift of the balance point on the bridge
c
IT
wire. (Ans. 225 em)
18. Fig. 3.248 shows experimental set up of a metre
r r bridge. When the two unknown resistances X and Y
are inserted, the null point D is obtained 40 cm from
r the end A. When a resistance of 10 0 is connected in
I r r series with X, the null point shifts by 10 cm. 'Find
the position of the null point when the 100
H
o resistance is instead connected in series with
6V
resistance 'Y'. Determine the values of the resis-
2.0 V tances X and Y. [CBSE D 09]
as shown in Fig.
, 3.2491 i
15 10 c
As ~*-
12 4
I" Thp~ ':Yh~atstQne ~ridge i.~; I'
resistance R is connected in parallel with 10 n 7. As points A and 0 are at the same potential,
resistance. Then therefore
10 R 1 4
- = - or R = 4 x 1.5= 60
1.5 R
15 10+ R 10 R
or --=5 If R' is the equivalent resistance of the network
12 4 lO+R
between B and C, then
or R=100.
2.5 x 10
100 200 R'= +2=40
2. or 100R = 20 .. R = 250. 2.5 + 10
100 R 40 100+ R
100+ R Current in the circuit, I = 10 =2.5 A
4
R
3. Proceed as in Example 178 on page 3.108.
Current through R (= 60) = 2.5 x 2.5 = 0.5 A.
1 2 2.5 + 10
4. As -=-
2 4 8 AD I
:. The given circuit is a balanced Wheatstone bridge 8. -=-=-- 1=16cm.
12 DB 40-/
as shown in Fig. 3.250. The resistance of 100 is
6 _ AD _ 30 R 4 r.
SI
ineffective. 9 . R- DB - 50 - 30 :. = s z.
B
10. Resistance of wire AJ = 60 x 0.01= 0.600
Resistance of wire BJ = 40 x 0.01= 0.400
A B When no current flows through the galvanometer,
P R 0.60 X
-=- or -=-
Q 5 0.40 4
0.60 x 4
IT
Fig. 3.250 : X=--=60
0.40
We have (In + 20) and (2n + 40) combinations Total resistance of X and R in series = 6 + 4 = 100
in parallel. Total resistance of wire AB = 0.60+ 0.40= 1.00
3x6
R=--=20. The above two resistances are in parallel.
3+6
. . .
= --10 x I = -10 0
H
5. The given circuit is equivalent to the circuit shown :. Total resistance of the circuit
10 + 1 11
in Fig. 3.251.
EMF 5
Current, I= = -- = 5.5 A.
Resistance 10/ 11
11. The circuit is a balanced Wheatstone bridge. Its
O
R R
Here -=-
R R 12. As !. =!., so the given circuit is a balanced
r r
So it is a balanced Wheatstone bridge. We have
Wheatstone bridge and the resistance r in the
resistances (R + R) and (R + R) in parallel.
vertical arm is ineffective. The circuit is then
2R x2R equivalent to two resistances of 2r and 2r connected
.. Equivalent resistance = = R.
2R+ 2R in parallel.
R
When resistance X is shunted with 2 Q resistor, the
Fig. 3.252 effective resistance becomes
B
Xx2 3x2
The resistances in arm AB are ineffective. X'=--=--=1.2Q
X+2 3+2
SI
1 1 1 1
:. -=---+---=- or R=lOQ. Now if the balance point is obtained at distance l '
R 10 + 10 10 + 10 10
from the left end, then
14. For a balanced metre bridge, X = _/- 2 l' 2 I'
R 100-/ or -=---
X' 100 -I' 1.2 100 -I'
X 2 2 /
But .. -=--
/'= 62.5cm
R 3 3 100-/
200 Shift in the balance point
or 200- 2/ = 31 or / =- = 40 em.
X + 10
In second case: --
Y
X
Y
= ---
60
100-60
40
100-40
IT
5
2
3
X
Y
= /' -/ = 62.5 - 40 = 22.5 em.
18. With the unknown resistances X and Y,the balance
point is 40 cm from the end A .
40
100-40
2
3
or
X 10 3
or -+- =- With 10Q resistance in series with X, the balance .
H
Y Y 2 point is at 40+ 10 = 50 em from the end A.
10 3 X 3 2 5
or - =- - - =- - - =- X+lO 50 =1
Y 2 Y 2 3 6
Y 100-50
10x6 2 2
., Y=--=12Q and X=-Y=-x12=8Q. or Y=X+lO
533
O
or ~X=X+lO
16. When P and Q are connected in series in the left gap, 2
P+ Q 50 or X = 20Q. and Y = 20+ 10 = 30Q.
50 100- 50
When 10Q resistance is connected in series with Y,
.. P+ Q= 50Q ...(1) let the balancing length be I'.
M
R
3.1. The storage battery of a car has an emf of 12 V. If the is the resistance of the resistor? What is the terminal voltage of
SI
internal resistance of the battery is 0.4 0, what is the maximum the battery when the circuit is closed ?
current that can be dra'i/{,nfrom the battery ? Ans.As
Ans. Here e = 12 V> r = 0.40 J=_e_
The current drawn £tom the battery will be maximum R+r
IT
when the external resistance in the circuit is zero i.e.,
R=O. or R+ r =-
s
I
H
I = §. J12
max r 0.4
R
e
=- - r
10
=- - 3 = 170
O
I 0.5
= 30 A.
3.3: (i) Three resistors of 1 0, 20 and 3 0 are combined in 3.7. A silver wire has a resistance of 2.10 at 27.5°C and a
series. What is the total resistance of the combination? (ii) If the resistance of 2.7 0 at 100° C Determine the temperature
combination is connected to a battery of emf 12 Vand negligible coefficient of resistivity of silver.
internal resistance, obtain the potential drop across each Ans.Here ~ = 2.10,t1 = 27.5°(, R; = 2.70,t2 = 100 0
(
R
V2 = I ~ = 2 x 2 = 4 V,
V3 = I ~ = 2 x 3 = 6 V.
3.8. A heating element using nichrome connected to a 230 V
supply draws an initial current of3.2 A which settles after afew
3.4. (i) Three resistors 2 0, 40 and 50 are combined in
seconds to a steady value of 2.8 A. What is the steady
parallel. What is the total resistance of the combination? (ii) If
temperature of the heating element if the room temperature is
the combination is connected to a battery of emf 20 V and
27° C ? Temperature coefficient of resistance of nichrome averaged
SI
negligible internal resistance, determine the current through
over the temperature range involved is 1.70 x 1O-4°C-1.
each resistor, and the total current drawn from the battery.
. 1 1 1 1 1 1 1 19 Ans. Here V = 230 V, II = 3.2 A,
Ans. (1)- = - + - + - = - + - + - =- 12 = 2.8 A, a = 1.70 x 10-4 0(-1
Rp ~ ~ ~ 2 4 5 20
Resistance at room temperature,
R = 20 O.
p 19 ~ = V = 230 = 71.8750
II 3.2
(ii) Currents drawn through different resistors are
Resistance at steady temperature,
t 20 t 20
IT
II = ~ = 2 = 10 A, 12 = ~ = 4 = 5 A, ~ = V = 230 = 82.1430
12 2.8
13 = -=-
t 20
=4A Now a= ~ -~
~ 5 ~ (t2 -t1)
Ans. Let I, II' 12, 13 be the currents as shown in above if X and Yare interchanged. (iii) What happens if the
Fig. 3.314. We apply Kirchhoff's second rule to different galvanometer and cell are interchanged at the balance point of
loops. the bridge? Would the galvanometer show any current?
B [CBSE D 05]
Ans. Here I = 35.9 ern, R = X = 7, 5 = Y = 12.5 n
- 100 - I R . 2 _ 100 - 39.5 R
As 5 - -- x .. 1 .5- x
I 39.5
R
LIon accounted for in the above formula.
(ii) When X and Yare interchanged,
Fig. 3.314 R = Y = 12.5n, 5= X = 8.16n, I =?
For loop ABDA, 100 -I 100-1
As 5= -1- x R :. 8.16=-1- x 12.5
SI
lOll + 513 - 512 = 0
For loop BCDB, or 8.161 = 1250 - 12.51
5(11 -13) -10(12 + 13) - 513 = 0 1250
or 1 = -- = 60.5 n , from the end A.
For loop ADCFGA, 20.66
512 + 10(12 + 13) + 10(11 + 12) = 10 (": 11+ 12 = I) (iii) When the galvanometer and cell are interchanged
or 1011 - 512 + 513 = 0 ...(1) at the balance point, the conditions of the balanced bridge
are still satisfied and so again the galvanometer will not
511 - 1012 - 2013 = 0 ...(2)
show any current.
IT
1011 + 2512 + 1013 = 10 ...(3)
3.11. A storage battery of emf B.O Vand internal resistance
Solving equations (1), (2) and (3), we get 0.5 n is being charged by a 120 V de supply using a series
4 6 2 resistor of 15.5 n. What is the terminal voltage of the battery
11 = 17 A, 12 = 17 A, 13 = - 17 A
during charging ? What is the purpose of having a series
Currents in different branches are resistor in the charging circuit ?
4 6 Ans. When the storage battery of 8.0 volt is charged
lAB = 11 =17A, IBC=11-13=17A,
H
with a de supply of 120 V, the net emf in the circuit will be
4 E.' = 120 - 8.0 = 112 V
I DC = 12 + 13 = - A
17
Current in the circuit during charging
6 2
I AD = 12 = - A, I BD = 13 = - - A
17 17 1=~= 112 =7 A
O
12.5 n.Determine the resistance of X. Why are the connections external source. In its absence, the current will be dange-
between resistors in a Wheatstone or metre bridge made of thick rously high.
copper strips ? (jj) Determine the balance point of the bridge 3.12. In a potentiometer arrangement, a cell of emf 1.25 V
x y gives a balance point at 35.0 em length of the wire. If the cell is
replaced by another cell and the balance point shifts to 63.0 em,
what is the emf of the second cell ?
Ans. Here E.1= 1.25 V, 4 = 35.0 em, 12 = 63.0 em,
E.2 =?
E.2 12
A~--~~~----------~C As
E.1 = 1;
e 12 e 63 x 1.25
C,2 = - x C,1= = 2.25 V.
Fig. 3.315 4 35
CURRENT ELECTRICITY 3.151
R
3 -1 is lighter ? Hence explain why aluminium wires are preferred
= ms
1.6 x 10-19 x 8.5 x 1028 x 2 x 10-6 for overhead power cables.
8
Given PAl = 2.63 x 1O- 0m, PCu = 1.72 x 10-8 Om,
3 ms-1 = 1.1 x 10-4 ms-1
16 x 85 x 2 x 10 relative density of Al = 2.7 and that of Cu = 8.9.
Ans. Mass = volume x density = Al d
Required time,
SI
I 3 4
= Pi .Id = pd 12 I
[.,'R=P-]
t =- = 4 S = 2.73 x 10 s = 7.57 h. R R A
vd 1.1 x 10-
As the two wires are of equal length and have the
3.14. The earth's surface has a negative surface charge same resistance, their mass ratio will be
density of10-9 Cm-2. The potential difference of400 kV between ncu PCu dcu 1.72 x 10-8 x 8.9
the top of the atmosphere and the surface results (due to the low -=---= 8 =2.1558=2.2
mAl PAl dAl 2.63 x10 x2.7
conductivity of the lower atmosphere) in a current of only
1800 A over the entire globe. If there were no mechanism of i.e., copper wire is 2.2 times heavier than aluminium wire.
IT
sustaining atmospheric electric field, how much time (roughly) Since aluminium is lighter, it is preferred for long sus-
would be required to neutralise the earth's surface? (Radius of pension of cables otherwise heavy cable may sag down
the earth = 6.37 x 106 m ). due to its own weight.
An . Surface charge density, 3.17. What conclusion can you draw from the following
c = 10 -9Cm-2 observations on a resistor made of alloy manganin :
Radius of the earth, Current Voltage Current Voltage
H
R = 6.37 x 106 m I(A) V I(A) V
Current, I = 1800 A 0.2 3.94 3.0 59.2
0.4 7.87 4.0 78.8
Total charge of the globe,
q = surface area x c = 41t R2cr 0.6 11.8 5.0 98.6
0.8 15.7 6.0 118.5
O
Since the V-I graph is almost a straight line, therefore, (b) Given the resistance of Elf 2 Of 30, how will you
manganin resistor is an ohmic resistor for given ranges of combine them to get an equivalent resistance of:
votlage and current. As the current increases from 0 to 8 A, (i) 110 (ii) 110 (iii) 60 (iv) i.. 0 ?
the temperature increases but the resistance of manganin 3 5 11
does not change. This indicates that the temperature [CBSE F 15]
coefficient of resistivity of manganin alloy is negligibly (c) Determine the equivalent resistance of the following .
small. networks:
3.18. Answer the following questions:
(a) A steady current flows in a metallic conductor of
non-uniform cross-section.Say which of thesequantities
is constant along the conductor: current, current
R
density, electric field, drift speed? [CBSE DISC]
(b) Is Ohm's law universally applicablefor all conducting (a)
elements ? If not, give examples of elements which
do not obey Ohm's law.
(c) A low voltage supply from which one needs high
SI
current must have very low internal resistance.
Why?
(d) Why a high tension (H. TJ supply of say 6 kV must
have a very large internal resistance?
R
Ans. (a) Only current is constant because it is given to (b)
be steady. Other quantities: current density, electric field
and drift speed vary inversely with area of cross-section. Fig. 3.317
(b) No, Ohm's law is not universally applicable for all Ans. (a) For maximum effective resistance, all the n
IT
conducting elements. Examples of non-ohmic elements resistors must be connected in series.
are vacuum diode, semiconductor diode, thyristor, gas
:. Maximum effective resistance,
discharge tube, electrolytic solution, etc.
Rs = nR
(c) The maximum current that can be drawn from a
voltage supply is given by For minimum effective resistance, all the n resistors
Imax =-
e must be connected in parallel. It is given by
1 1 1 1
H
r
n
- =- +- +- + n terms = -
Clearly, Imax will be large if r is small. Rp R R R R
(d) If the internal resistance is not very large, then the :. Minimum effective resistance,
current will exceed the safety limits in case the circuit is R =~
short-circuited accidentally. p n
O
3.19. Choose the correct alternative: Ratio of the maximum to minimum resistance is
(a) Alloys of metals usually have (greater! lesser) ~ = nR = ~ =n2: 1.
resistivity than that of their constituent metals. Rp R/ n 1
(b) Alloys usually have much (lower/higher) temperature
(b) Here R,.= 10, ~ = 20, R:,= 30
coefficients of resistance than pure metals.
M
(c) The resistivity of the alloy manganin is nearly (i) When parallel combination of 1 0 and 2 0 resistors
independent of/increases rapidly with increase of is connected in series with 30 resistor [Fig. 3.318(a)], the
temperature. equivalent resistance is
(d) The resistivity of a typical insulator te.g., amber) is R=R +R:,= R,.~ +R:,
greater than that of a metal by afactor of the order of P R,.+~
(1022/103 ). 1x2 2 11
=--+3=-+3=-0.
Ans. (a) greater (b) lower (c) is nearly independent of 1+ 2 3 3
(d) 1022.
(ii) When parallel combination of 20 and 30 resistors
3.20. (a) Given n resistors each of resistance R, how will you is connected in series with 1 0 resistor [Fig. 3.318(b)], the
combine them to get the ti) maximum, (ii) minimum effective equivalent resistance is
resistance ? What is the ratio of the maximum to minimum ~R:, 2x3 6 11
R= +R,.=--+1=-+1=-0.
resistance ? ~+R:, 2+3 5 5
CURRENT ELECTRICITY 3.153
lQ 2Q lQ 10 10 10 10 A
~3AQA_ ~AQA_
~vvv-- ~vvv--
10
(a) (b) 10 B
lQ
Fig. 3.320
2Q
Ans. Let the equivalent resistance of the infinite
3Q network be X. This network consists of infinite units of
three resistors of 1 0, 1 0, 1 O. The addition of one more
R
(c) (d) such unit across AB will not affect the total resistance. The
network obtained by adding one more unit would appear
Fig. 3.318
as shown in Fig. 3.321.
SI
R = ~ + ~ + R:J = (1 + 2 + 3) 0 = 6 o. x 10
(iv) When all the resistances are connected in parallel 10
[Fig. 3.318(d)], B
111111111
R = ~ + ~ + R:J = 1 + "2 + "3 = 6 Fig. 3.321
Resistance between A and B
Equivalent resistance, R = ~ n.
11 = Resistance equivalent to parallel
combination of X and 10
IT
(c) The network shown in Fig. 3.317(a) is a series
combination of four identical units. One such unit is X xl X
--=--
shown in Fig. 3.319(a) and it is equivalent to a parallel xi t X+1
combination of two resistances of 20 and 40 as shown in
Resistance between P and Q
Fig.3.319(b).
X X
=1+--+1=2+--
X+1 x r i
H
This must be equal to the original resistance X.
X
X=2+--
1+ X
or X2 - 2X - 2 = 0
O
(a) (b)
or X=l±.J3
-=-+-=--=-
R 2 4 4 4 Total resistance X + r 2.732 + 0.5
= 3.713 A
or R=io
3 3.22. Figure 3.322 shows a potentiometer with a cell of
.. Resistance of the total network (4 such units) 2.0 V and internal resistance 0.40 0 maintaining a potential
drop across the resistor wire AB. A standard cell which
=4 xi = 16 o. maintains a constant emf of 1.02 V (jor very moderate currents
3 3
upto afew A) gives a balance point at 67.3 cm length of the wire.
(ii) The network shown in Fig. 3.319(b) is a series To ensure very low currents drawn from the standard cell, a
combination of 5 resistors, each of resistance R. very high resistance of 600 k 0 is put in series with it, which is
:. Equivalent resistance = 5 R. shorted close to the balance point. The standard ceil is then
3.21. Determine the current drawn from a 12 V supply with replaced by a cell of unknown emf E. and the balallce point found
internal resistance 0.50 by thefollowing infinite network. Each similarly turns out to be at 82.3 em length of the wire.
resistor has 1 0 resistance. (a) What is the value ofE. ?
3.154 PHYSICS-XII
R
600 kfl have?
(c) Is the balance point affected by this high resistance?
(d) Is the balance point affected by the internal
resistance of the driver cell ? Fig. 3.323
(e) Would the method work in the above situation if the
SI
driver cell of the potentiometer had an emf of 1.0 V Ans. Here R = 10.00, ~ = 58.3 em, X =? ,12 = 68.5em
instead of 2.0 V ? e e
Let 1 and 2 be the potential drops across R and X
respectively and I be the current in potentiometer wire.
if> Would the circuit work well for determining
extremely small emf, say of the order of a few m V Then
e IX X
2
,
(e) No, the arrangement will not work. If is greater e @ ,,
,
than the emf of the driver cell of the potentiometer, there
will be no balance point on the wire AB.
if> The circuit as it is would be unsuitable, because the
e
balance point (for of the order of a few m V) will be very
close to the end A and the percentage error in measure- Fig. 3.324
ment will be very large. The circuit is modified by putting
a suitable resistor R in series with the wire AB so that Ans. Here ~ = 76.3 em, Iz = 64.8 em, R = 9.50
potential drop across AB is only slightly greater than the The formula for the .internal resistance of a cell by
emf to be measured. Then the balance point will be at potentiometer method is
larger length of the wire and the percentage error will be
much smaller.
r = R(~ -12)
12
= 9.5(76.3 - 64.8) = 9.5 x 11.5 ~ 1.70.
64.8 64.8
CURRENT ELECTRICITY 3.155
1. Define electric current. What is the 51unit of electric 25. Define the term' drift velocity' of charge carriers in
current? a conductor and write its relationship with the
2. Write the relation between a coulomb and an current flowing through it. [CBSE D 14]
ampere. [ISCE 96] 26. Write the expression for the drift velocity of charge
R
3. What does the direction of electric current signify in carriers in a conductor of length 'I' across which a
an electric circuit? potential difference 'V' is applied. [CBSE OD 14C]
4. What is electromotive force? State its 51 Unit. 27. How does one explain increase in resistivity of a
[Punjab 2000] metal with increase of temperature? [CBSE OD 14C]
5. State the condition in which terminal voltage across 28. Define the term mobility of charge carriers in a
SI
a secondary cell is equal to its emf? conductor. Write its 51unit. [CBSE D 14; OD 15]
[CBSE D 2000] 29. Plot a graph showing variation of resistivity of a
6. Define an emf of one volt. conductor (copper) with temperature.
[CBSE D 14; F 15]
7. State Ohm's law. [ISCE 95]
30. Plot a graph showing variation of current versus
8. Name the colours corresponding to the digits 4 and
voltage for the material GaAs. [CBSE D 14]
7 in the colour code scheme for carbon resistors.
[CBSE SP 15] 31. Sketch a graph showing variation of resistivity of
carbon with temperature. [CBSE D 06]
9. Define resistance and state its 51unit. [CBSE D 92C]
IT
Or
10. Define Ohm.
11. Define conductance of a material. Give its 51unit. Show on a graph, the variation of resistivity with
[CBSE D 02]
temperature for a typical semiconductor Si.
[CBSE DOS, 12,14]
12. Define electrical conductivity of a material. Give its
51unit. [CBSE D 03,14] 32. Name two materials whose resistivity decreases
H
with the rise of temperature.
13. How much is the resistance of an air-gap?
33. How does the conductance of a semi-conducting
14. How much is the resistance of a closed plug-key?
material change with rise in temperature?
15. Which metal has the lowest resistivity ?
34. Of copper and nichrome, which one has possibly
16. Define resistivity of a material. State its 51unit.
larger value of temperature coefficient of
O
[ISCE 93]
resistance? [CBSE D 95C]
17. What is the order of resistivity of an insulator? 35. How does resistivity of alloy manganin change
[Punjab 97C]
with temperature?
18. What is the ratio of the resistivity of a typical . 36. Bow does the resistance" of an insulator change
insulator to that of a metal? with temperature? I
M
19. What is the average velocity of free electrons in a 37. Name two parameters which determine the
metal at room temperature? resistivity of a material.
20. Give the order of magnitude of the number density 38. How is the conductivity of an electrolyte affec-
of free electrons in a metal. ted by the increase of temperature? [CBSE D 95]
21. Give the order of magnitude of thermal velocity 39. If potential difference V applied across a conductor
and drift velocity of free electrons in a conductor is increased to 2 V, how will the drift velocity of the
carrying current at room temperature. electrons change? [CBSE OD 2000C]
22. What is the order of resistivity of conductor? 40. What is a non-ohmic device? State one example.
23. Define temperature coefficient of resistivity. [Punjab 02]
24. How does the random motion of free electrons in a 41. What is a linear resistor?
conductor get affected when a potential difference 42. Give an example of non-ohmic device which shows
is applied across its ends? [CBSE D 14C] up negative resistance.
3.156 PHYSICS-XII
43. A cell of emf 'e' and internal resistance 't' draws a 62. How many kilowatt hours (kWh) are there in one
current '1'. Write the relation between terminal joule? [CBSE on 99C]
e,
voltage 'V' in terms of I and r. [CBSE aD 13] 63. The applied p.d. across a given resistance is altered
e,
44. Two identical cells, each of emf having negligible so that heat produced per second increases by a
internal resistance r, are connected in parallel with factor of 9. By what factor does the applied p.d ..
each other across an external resistance R. What is change? [CBSE on 99C]
the current through this resistance? [CBSE aD 13] 64. Two electric bulbs are rated at 220 V - 100W and
45. A 4 n non-insulated resistance wire is bent 1800 in 220 V - 60 W. Which one of these has greater
the middle and the two halves are twisted together. resistance and why? [CBSE Sample Paper 03]
What will be its new resistance? [CBSE D 10C] 65. The maximum power dissipated in a 10,000n
46. Can Kirchhoff's laws be applied to both d.c. and a.c. resistor is 1W. What is the maximum current?
R
circuits? [ISCE 93]
47. On what conservation principle is the Kirchoff's 66. What is the safest voltage you can safely put across
first law based ? a 98 n. 0.5 W resistor? [ISCE 97]
48. On what conservation principle is the Kirchhoff's 67. How much charge flows through a 250 V, 1,000W
second law based ? heater in one minute? [ISCE 96]
SI
49. Name the device used for measuring the emf of a 68. A heating element is marked 210 V, 630W. What is
cell. [CBSE D 96] the value of the current drawn by the element when
50. Name the device used for measuring the internal connected to a 210 V de source? [CBSE D 13]
resistance of a secondary cell. [CBSE D 96] 69. A heating element is marked 210 V, 630 W. Find the
51. Define potential gradient. Give its 51 unit. resistance of the element when connected to a 210 V
dc source. [CBSE D 13]
52. Name the principle on which a metre bridge works.
70. Two resistors of 2 nand 4 n are connected in
53. What is a Wheatstone bridge? [CBSE D 03]
parallel to a constant d.c. voltage. In which case
IT
54. The given graph shows the variation of resistance more heat is produced? [CBSE D 98C ]
of mercury in the temperature' range 0 < T < 4 K
71. Two bulbs whose resistances are in the ratio 1 :2 are
Name the phenomenon shown by the graph.
connected in parallel to a source of constant
[CBSEOD 03]
voltage. What will be the ratio of power dissipation
in these bulbs? [CBSE D 2000C]
c
H
72. Distinguish between kilowatt and kilowatt hour.
~ 0.16
u
73. Which has a greater resistance-kW electric heater
§
or a 100 W filament bulb both marked for 220 V ?
'~0 0.08 [CBSE D OlC]
74. The coil of a heater is cut into two equal halves and
O
82. The plot of the v 86. A resistance R is connected across a cell, of emf e
variation of potential 6V and internal resistance r. A potentiometer now
difference across a measures the p.d., between the terminals of the cell,
combination of three as V. State the expression for 'r' in terms ofe, V and
identical cells iri series, R [CBSE011]
versus current is as 87.: A parallel combination of two cells of emf's el and
shown in Fig. 3.326. o 1A
e2, and internal resistances, 1. and r2, is used to
What is the emf of each Fig. 3.326 supply current to a load of resistance R Write the
cell ? [CBSE0 08] expression for the current through the load in terms
83. A (i) series (ii) parallel combination of two given of el, e2, 1. and r2· [CBSESamplePaper 2011]
R
resistors is connected one-by-one, across a cell. In 88. Under what condition can we draw maximum
which case will the terminal potential difference, current from a secondary cell? [CBSEF 10]
across the cell, have a higher value?
89. Write any two factors on which the' internal
[CBSE00 08C]
resistance of a cell depends. [CBSE00 10]
84. The I-V charac-
SI
teristics of a resistor 90. Write two factors on which the sensitivity of a
are observed to potentiometer depends. [CBSE0 13C]
deviate from a 91. Graph showing the variation of current versus
straight line for higher voltage for a material GaAs is shown in the figure.
values of current as Identify the region of
shown in Fig. 3.327. V~ (i) negative resistance
Why? (ii) where Ohm's law is obeyed. [CBSE0 15]
Fig. 3.327
[CBSESP 08]
85. Two identical slabs of given metal are joined
IT
together, in two different ways, as shown in t
Figs. 3.328(i) and (ii). What is the ratio of the c
resistances of these two combinations? ~
::l
U
[CBSE0 10C]
A~--~~--~------
VoltageV ~
H
Fig. 3.329
(i) (ii)
O
Fig. 3.328
Answers
•
M
1. The electric current is defined as the rate of flow of 4. The work done per unit charge by a source in
electric charge through any section of a conductor. taking the charge once round the complete circuit is
Total charge flowing called electromotive force or emf of the source. SI
Electric current = -----'~---=
Time taken unit of emf is volt.
5. When no current is drawn from the cell, its terminal
or I=i
t voltage is equal to its emf.
The SI unit of electric current is ampere (A). 6. If an electric cell supplies an energy of 1 joule for the
1coulomb flow of 1 coulomb of charge through the whole
2. 1 ampere = ------
I second circuit (including the cell), then its emf is said to be
1 volt.
3. The direction of conventional current in an electric
circuit tells the direction of flow of positive charges 7. Ohm's law states that the electric current I passing
in that circuit. through a conductor is proportional to the potential
3.158 PHYSICS-XII
difference V applied across its ends, provided, the 24. Random motion gets partially directed towards the
temperature and other physical conditions remain higher potential side.
unchanged, i.e., V oc I or V = RI 25. Refer to point 17 of Glimpses on page 3.168.
where R is called resistance of the conductor. eV,
8. Yellow and violet respectively.
26. vd =-;;;z
9. Resistance of a conductor is the property by virtue 27. With the increase in temperature, the relaxation
of which it opposes the flow of current through it. It
is equal to the ratio of the potential difference
time r decreases and hence resistivity (p = ~
ne,
J
applied across the conductor to the current flowing increases.
through it. SI unit of resistance is ohm (n).
28. Refer to point 21 of Glimpses on page 3.169.
R
R=V 29. See Fig. 3.20(a).
I
30. See Fig. 3.27.
10. The resistance of a conductor is said to be 1 ohm if 31. See Fig. 3.21 on page 3.25.
1 ampere of current flows through it on applying a
potential difference of 1 volt across its ends 32. Germanium and silicon.
SI
1 volt 1V 33. With the rise in temperature, the conductance of a
1 ohm = or 1n = - .
semi-conducting material increases exponentially.
1ampere 1A
34. Copper.
11. The ease with which a conductor allows a current to
flow through it is called its conductance. It is equal 35. The resistivity of alloy manganin is nearly
to the reciprocal of resistance. independent of temperature.
1 36. The resistance of an insulator decreases with the
Conductance (G) = -----
increase of temperature.
Resistance (R)
37. The resistivity of a material depends on (i) its
IT
SI unit of conductance is ohm -lor mho.
number density of free electrons, (ii) the relaxation
12. The conductivity of a material is equal to the time.
reciprocal of its resistivity.
38. The conductivity of an electrolyte increases with
Conductivity ( 0') =, 1 the increase in its temperature.
Resistivity (p)
39. Drift velocity,
SI unit of conductivity is ohm -1m -lor mho m -1. eE, eV,
H
13. Infinity. vd = ----;;;= -;;;z
14. Negligibly small. Clearly, when V is increased to 2 V, drift velocity
15. Silver. also gets doubled.
16. The resistivity of a material is the resistance offered 40. A device which does not obey Ohm's law is called a
O
by a unit cube of that material. Its SI unit is n m. non-ohmic device. Semiconductor diodes, ther-
17. The resistivity of an insulator like glass or rubber is mistors, etc. are non-ohmic devices.
of the order of 108 - 1015 nm. 41. A linear resistor is one which obeys Ohm's law or
18. The ratio of 1022. for which voltage-current graph is a straight line'
19. Zero. ' passing through origin. .
M
R
SI unit is volt per metre (Vm -1).
68. [=!.. = 630 = 3 A.
52. The working of a metre-bridge is based on the V 210
principle of Wheatstone bridge. 2
69. R = V = 210 x210 = 700.
53. A Wheatstone bridge is an arrangement of four P 630
SI
resistances used to determine quickly and accu-
V2t
rately one of these resistances in terms of other 70. Heat produced, H = - i.e., n «-;1
R R
three resistances. '
Thus heat produced in 2 0 resistor is more than that
54. Superconductivity. in 40 resistor.
55. Here P = H1, Q = 20, R = 360, 5 = ?
For a balanced Wheatstone bridge, 71. Ii = V~ / R, = Rz = ~ = 2 : 1.
Pz V/Rz R, 1
P R
Q 5 72. Kilowatt is the unit of electric power while kilowatt
IT
hour is the unit of electric energy.
5 = R x Q = 36 x 2 = 720.
P 1 - 1 kilowatt = 1000 W = 1000 Js-1
56. Rate of production of heat is P = VI 1 kilowatt hour = 3.6 x 106 J.
57. Yes, it is valid. V2 220 x 220
73. Resistance of heater = - = = 48.40
58. Electric energy = Electric power x time. Ii 1000
H
59. Kilowatt hour is the unit of electrical energy. V2 220 x 220
Resistance of bulb ---=4840
60. 1 unit of electric energy = 1kWh. This means that Pz 100
when an appliance of power 1000 watt is operated
Thus the 100 W bulb has a greater resistance.
on mains for 1 hour, it consumes 1 unit of electric
74. Let original heat produced,
energy.
O
2V2t H
V2 H2 = R/2 =R .. _2 =2:1
63. Heat produced per second, P = - HI
R
75. The maximum power rating of a resistor is the
maximum power that it can dissipate in the form of
or
heat without undergoing melting.
As the heat produced per second increases 9 times, 76. P= [2R= pi ;[2.
so the applied p.d. must increase 3 times the A
original p.d. Volume, V = Al
V2 1 :. Power transferred per unit volume
64. R = - . For a given voltage, R ex: - .
P . P = !.. = rZpl/ A = (.!..)2 P = J2(J
So 60 W bulb has greater resistance than 100 W V Al A
bulb. where J is the current density.
3.160 PHYSICS-XII
80.
while that of a fuse wire is very low.
Total power P dissipated by the series combination
86. Internal resistance, r -_ (t -VV] R.
is given by
1 1 1 1 1 1
R
-=-+-=--+--=- t1'2 + t21
P ~ ~ 1000 1000 500
R('t + '2)+ 1'2
or P= SOOW.
81. The resistivity p at any temperature T is given by
88. When the external resistance in the circuit is zero,
the current drawn from the secondary cell is max.
SI
p =Po [1+ a(T -1Q))
1. Distinguish between electromotive force and 5. Explain the term 'drift velocity' of electrons in a
terminal potential difference of a cell. What are their conductor. Hence obtain the expression for the
units? [CBSE 00 14C] current through a conductor in terms of 'drift
2. Explain how the average velocity of free electrons velocity'. [CBSE 00 13, 13C, lSC]
M
in a metal at constant temperature, in an electric 6. Prove that the current density of a metallic
field, remains constant even though the electrons
conductor is directly proportional to the drift speed
are being constantly accelerated by this electric
of electrons. [CBSE 0 08]
field?
3. Define the terms resistivity and conductivity and 7. What is meant by drift velocity of free electrons?
state their 51 units. Draw a graph showing the Derive Ohm's law on the basis of the theory of
variation of resistivity with temperature for a electron drift. [CBSE 0 03; Haryana 94]
typical semiconductor. [CBSE 0 05] 8. Are the paths of electrons straight lines between
4. Define the electrical resistivity of a material. How it successive collisions (with positive ions of the
is related to the electrical conductivity ? Of the metal) in the (i) absence of electric field (ii) presence
factors, length, area of cross-section, nature of of electric field ? Establish a relation between drift
material and temperature - which ones control the velocity 'v/ of an electron in a conductor of cross-
resistivity value of conductor? [CBSE F 98] section' A', carrying current' l' and concentration
CURRENT ELECTRICITY 3.161
'n' of free electrons per unit volume of conductor. 18. Draw a plot showing the variation of resistivity of a
Hence obtain the relation between current density (i) conductor and (ii) semiconductor, with the
and drift velocity. [CBSE aD 03] increase in temperature.
9. Define relaxation time of electrons in a conductor. How does one explain this behaviour in terms of
Explain how it varies with increase in temperature number density of charge carriers and the
of a conductor. State the relation between resistivity relaxation time ? [CBSE D 14C]
and relaxation time. [CBSE D 2000] 19. Define conductivity of a conductor and state its S1
unit. State and explain the variation of conductivity of
10. A conductor of length 'I' is connected to a d.c.
(a) good conductor (b) ionic conductor with
source of potential 'V'. If the length of the
temperature. [CBSE D 01, 08]
conductor is tripled, by stretching it, keeping 'V'
R
constant, explain how do the following factors vary 20. Establish the relation between drift velocity of
in the conductor: electrons and the electric field applied to the
conductor. [Punjab02]
(i) Drift speed of electrons, (ii) Resistance and
Or
(iii) Resistivity. [CBSE D 2000]
Derive an expression for drift velocity of free
SI
11. Write the mathematical relation between mobility
and drift velocity of charge carri~rs in a conductor. electrons in a conductor in terms of relaxation time.
Name the mobile charge carriers responsible for [CBSE D 09, on 15]
conduction of electric current in 21. Establish a relation between current and drift
(i) an electrolyte (ii) an ionised gas. [CBSE D 06] velocity. [Himachal03; CBSE on 15C]
12. Define the term current density of a metallic 22. Define the term resistance. Give physical expla-
conductor. Deduce the relation connecting current nation of the opposition offered by a conductor to
density 0) and the conductivity (o) of the the flow of current through it. [Haryana94]
conductor, when an electric field E, is applied to it.
IT
23. Explain the colour code for carbon resistors with
[CBSE D 06] illustrations. [Haryana95, 98]
13. Define ionic mobility. Write its relationship with 24. Three resistances ~, Rz and ~ are connected in
relaxation time. Give its S1 unit. How does one series. Find their equivalent resistance. [CBSE D 92]
understand the temperature dependence of resis- 25. Three resistances ~, Rz and ~ are connected in
tivity of a semiconductor. [CBSE F 10; oo 13C]
parallel. Find the equivalent resistance of the
H
14. Definethe terms (I) drift velocity, (ii) relaxation time. parallel combination.
A conductor of length L is connected to a de source [CBSE D 92 ; Himachal98C ; Punjab03]
of emf e. If this conductor is replaced by another 26. What is superconductivity ? Explain. State two
conductor of same material and same area of applications of superconductors. [Punjab03]
cross-section but of length 3L, how will the drift
27. What are superconductors? Give two applications
O
16. Define the term resistivity of a conductor. Give its example of each. [Haryana02]
S1unit. Show that the resistance of a conductor is 30. State the conditions under which Ohm's law is not
given by obeyed in a conductor. [CBSE D 92]
e
34. Figure 3.330shows a cell of emf and internal resis- consume powers ~ and Pz respectively. Deduce the
tance r, connected to a voltmeter V and a variable expressions for the power of their combination
resistance R Deduce the relationship among V, R e, when they are, in turn, connected in (i) series and
and r. How will V vary when R is reduced. (ii) parallel across the same voltage supply.
[ISCE98] [CBSE0011]
45. Give four reasons why nichrome element is
,,, commonly used in household heating appliances.
,,,
R v 46. What is a safety fuse? Explain its function.
Y',
, , [Punjab99]
47. State the two Kirchhoff's rules used in electric
R
networks. How are these rules justified ?
Fig. 3.330 [CBSE0 14,00 15]
35. Define internal resistance of a cell. Prove that 48. State the working principle of a potentiometer.
Explain, with the help of a circuit diagram, how the
emfs of two primary cells are compared by using a
SI
potentiometer. How can the sensitivity of a
where R is the external resistance used. [Himachal99] potentiometer be increased ?
36. A cell of emf 'e' and internal resistance 'r is [CBSE0 05,06C; 00 15C]
connected across a variable load resistor R Draw 49. State the principle of a potentiometer. With the help of
the plots of the terminal voltage V versus (i) Rand a circuit diagram, describe a method to find the inter-
(ii) the current I. [CBSE0 15] nal resistance of a primary cell.
37. Distinguish between emf (e) and terminal voltage [CBSE0 03 ; 00 13]
(V) of a cell having internal resistance 'r . Draw a 50. You are required to find the internal resistance of a
plot showing the variation of terminal voltage (V)
IT
primary cell in the laboratory. Draw a circuit diagram
vs. the current (I) drawn from the cell. Using this of the apparatus you will use to determine it. Explain
plot, how does one determine the emf and the the principle of the experiment. Give the formula
internal resistance of the cell? [CBSE00 14,14C]
used. [CBSE0 08C]
38. A cell of emf e and internal resistance r is connected
51. Why is the use of a potentiometer preferred over
across a variable resistance R Plot graphs showing
that of a voltmeter for the measurement of emf of a
e
the variation of (i) and R, (ii) terminal p.d. V with
H
cell ? [Himachal01]
R Predict from the second graph under which V
becomes equal to e. [CBSE0 09] 52. Use Kirchhoff's rules to obtain conditions for the
balance condition in Wheatstone bridge.
39. Two identical cells, each of emf e and internal
[CBSE015]
resistance r are connected in parallel to an external
53. For the circuit diagram of a Wheatstone bridge
O
55. Draw a circuit diagram which can be used to deter- 57. Draw a circuit diagram of a metre bridge to
mine the resistance of a given wire. Explain the compare two resistances. Write the formula used.
principle of the experiment and give the formula Why is this method suitable only for two resistances
used. [eBSE 0 03C] of the same order of magnitude? [eBSE F 99]
56. Draw a circuit diagram using a metre bridge and 58. Derive an expression for the heat produced in a
write the necessary mathematical relation used to resistor R when voltage drop across it is V.
determine the value of an unknown resistance. [eBSE F 93]
Why cannot such an arrangement be used for
measuring very low resistances? [eBSE 0 06]
Answers
R
••
1. 11. Mobility = Drift velocity v
Jl =--.!L
or
Electric field E
EMF Terminal Voltage
(i) The charge carriers in an electrolyte are
SI
(i) It is the potential dif- It is the potential dif-
ference between two ference between two positive and negative ions. ,
terminals of the cells terminals when a
current passes through (ii) The charge carriers in an ionised gas are
when no current is
drawn from it. it. electrons and positively charged.
(ii) It is a cause. It is an effect. 12. Refer answer to Q. 12 on page 3.7
(iii) The SI unit is volt. The SI unit is volt. 13. Refer to point 21 of Glimpses.
8. Refer answer to Problem l(e) on page 3.122 and 15. Refer answer to Q. 19 on page 3.16.
Q. 19 on page 3.16. 16. Refer answer to Q. 19 on page 3.16.
9. The average time that elapses between two successive 17. Resistivity of a material is the resistance of a
collisions of an electron in a conductor is called conductor of that material having unit length and
relaxation time (r), It is rE!latedto resistivity p as unit area of cross-section. The SI unit of resistivity is
M
The conductivity of an ionic conductor increases 41. Alloy like manganin has a small value of tempe-
with the increase of temperature. As the tempe- rature coefficient of resistivity. It is used for making
rature Increases, the electrostatic attraction between standard resistances.
cations and anions decreases, the ions are more free 42. Refer answer to Q.44 and Q. 45. on pages 3.60 & 3.61.
to move and so the conductivity increases. 43. Refer answer to Q. 44 on page 3.60.
20. Refer answer to Q. 18 on page 3.15. 44. Refer to answers of Q. 47, 48 on page 3.61 & 3.62.
21. Refer answer to Q. 19 on page 3.16. 45. Refer answer to Q. 54 (Application 1) on page 3.64.
22. Refer answer to Q. 21 on page 3.17.
46. Refer answer to Q. !?4(Application 3) on page 3.64.
23. Refer answer to Q. 15 on page 3.9.
47. Refer to point 47 of Glimpses on page 3.172.
24. Refer answer to Q. 32 on page 3.30.
48. Refer answers to Q. 57 and Q. 58 on page 3.96.
25. Refer answer to Q. 33 on page 3.31.
R
49. Refer answer to Q. 59 on page 3.97.
26. Refer answer to Q. 28 on page 3.29.
• 50. Refer answer to Q. 59 on page 3.97.
27. Refer answer to Q. 30 on page 3.30. 51. Refer answer to Q. 60 on page 3.97.
28. Refer answer to Q. 29 on page 3.29.
52. Refer answer to Q. 62 on page 3.104.
29. Refer answer to Q. 27 on page 3.28.
SI
53. Refer answer to Q. 62 on page 3.104. ...--- .
30. Refer answer to Q. 27 on·page 3.28.
54. See Fig. 3.213. The working of a metre bridge is
31. Refer answer to Q. 34 on page 3.45.
based on the principle of Wheatstone bridge. When
32. Refer answer to Q. 35 on page 3.46.
the bridge is balanced i.e., no current flows through
33. Refer answer to Q. 35 on page 3.46. P R
the galvanometer arm, - =-
34. Refer answer to Q. 35 on page 3.46. Q 5
eR e Error in determination of resistance can be
We get V = R + r = 1 + (r / R) minimised by adjusting the balance point near the
middle of the metre bridge wire.
IT
Clearly V decreases when R is reduced. 55. Refer answer to Q. 65 on page 3.105.
35. Refer answer to Q. 35 on page 3.46. 56. Refer answer to Q. 65 on page 3.105.
36. (i) See Fig. 3.91(b) (ii) See Fig. 3.91(c) on page 3.46. Metre bridge becomes insensitive for very low
37. Refer answer to Q. 35 on page 3.46. resistance. Moreover, the end resistances become
38. Refer answer to Q. 35 on page 3.46. comparable to the unknown low resistance and
cannot be neglected.
H
e e 2Re
39. 1= =--=-- 57. Refer answer to Q. 65 on page 3.105 and Problem 71
R+ _r_x_r R+ r 2R+ r on page 3.120.
r+ r 2
58. Refer answer to Q. 43 on page 3.60.
40. Refer to the answers of Q. 37, 38 on pages 3.51 & 3.52.
O
electrons and relaxation time. [CBSE D 05] Each cell has emf e and iriternal resistance r.
2. What do you understand by the resistivity of a Show that the current in the circuit will be
conductor ? Discuss its temperature dependence maximum when R = nr / m.
for a (i) conductor (ii) semiconductor, and 6. State Kirchhoff's laws for an electrical network.
(iii) electrolyte. [CBSE D92C]
Using Kirchhoff's laws, find the relation between
3. A battery of n cells, each of emf e
and internal
the resistances of four arms of a Wheatstone bridge
resistance r, is connected across an external
resistance R. Find the current in the circuit. Discuss when the bridge is balanced.
the special cases when (i) R» nrand (ii) R« nr. Draw a circuit diagram to determine the unknown
e
4. n cells, each of emf and internal resistance rare resistance of a metallic conductor using a metre
bridge. [CBSE OD 03C ; D 13]
connected in parallel across an external resistance
R.Determine the condition for maximum current in 7. Define the term potential gradient. Using this
the circuit. concept, explain the method for comparison of
CURRENT ELECTRICITY 3.165
emfs of two primary cells using a potentiometer. resistance of a wire in the laboratory. Draw the
Establish the relation used. Write two possible circuit diagram and write the formula used. Write
causes of potentiometer giving only one-sided any two precautions you would observe while
deflection.· [CBSED13] performing the experiment. [CBSED 04]
8. (a) State the working principle of a potentiometer. 10. (a) State, with the help of a circuit diagram, the
Draw a circuit diagram to compare the emfs of two working principle of a. metre bridge. Obtain
primary cells. Derive the fonriula used. (b) Which. the expression used for determining the
material is used for potentiometer wire and why? unknown resistance.
(c) How can the sensitivity of a potentiometer be (b) What happens if the galvanometer and cell are
increased? [CBSED 11C] interchanged at the balance point of the bridge?
9. Deduce the condition for balance in a Wheatstone (c) Why it is considered important to obtain the
R
bridge. Using the principle of Wheatstone.bridge, balance point near the midpoint of the wire?
describe the method to determine the specific· [CBSED 11C]
Answers
•
SI
1. Refer answer to Q. 19 on page 3.16. 8. (a) Refer answer to Q. 57 and Q. 58 on page 3.96.
2. Refer answer to Q. 25 on page 3.24. (b) Referto the solution of Problem 49 on page 3.119.
3. Refer answer to Q. 39 on page 3.53. (c) Referto the solution of Problem 56 on page 3.119.
4. Refer answer to Q. 40 on page 3.53. 9. Refer answers to Q. 62 on page 3.104 and Q. 65 on
5. Refer answer to Q. 41 on page 3.54. page 3.105.
6. Refer answer to Q. 62 on page 3.104 and see 10. (a) Refer answer to Q. 65 on page 3.105.
IT
Fig. 3.214 on page 3.106. (b) There is no change in the position of the
7. Refer answer to Q. 58 on page 3.96 and Problem 42 balance of the bridge.
on page 3.142. (c) Referto the solution of Problem 65 on page 3.119. ,
noticed that the car battery has been discharged as Alongwith some of their friends and some respon-
the headlights were left on for a long time. He sible representatives of that area, they visited house
brought another battery from his garage and to house of that colony and made people aware of
connected its terminals to the terminals of the car the risks involved in short circuiting. They also
battery. He succeeded in starting the engine and explained the people the importance of paying
then disconnected his battery. This is called 'jump electricity bills. They succeeded in changing the
M
starting', Mrs. Sharma felt happy and thanked both mindset of the people. Answer the following
Rohit and Ramu. Answer the following questions questions based on the above information:
based on the above information: (a) What according to you, are the values of
(a) What values were displayed by Rohit ? displayed by Manish and Rajnish ?
(b) A household circuit has a fuse of 5 A rating.
(b) A storage battery of emf 12 V and internal
resistance 0.5 n is to be charged by a battery Find the maximum number of bulbs of rating
60 W - 220 Veach which can be connected in
charger which supplies 110 V de. How much
this circuit.
resistance must be connected in series with the
battery to limit the charging current to 5 A. 3. Abhishek went to meet his grandfather who lived
What will be the p.d. across the terminals of in a village. Both were resting and gossiping under
the battery during charging ? What is the a fan to get relief from the scorching heat of
purpose of having a series resistor in the summer. The lights suddenly went off. On seeing
charging circuit? that, that all their neighbourers had electricity,
3.166 PHYSICS-XII
grandfather told Abhishek the fuse might have his friend Rohit, an electrical engineer by
blown up. Abhishek immediately changed the fuse. profession, visited his house. When he pointed out
Grandfather blessed Abhishek and told him that if his anxiety about this to Rohit, his friend found that
had he not come to the village, he would have to Ameen was using traditional incandescent lamps
sleep the whole night without fan. Abhishek realised and using old fashioned air conditioner. In addition
his grandfather's problem and decided to replace there was no proper earthing in the house. Rohit
the fuse with a circuit breaker which uses a solenoid advised him to use CFL bulbs of 28 W instead of
with a core. When the current exceeds a safety limit, 1000 W - 220 V and also advised him to get proper
the breaker is activated and thus breaks the circuit. earthing in the house. He made some useful
The circuit can be closed by a manual switch. suggestion and asked him to spread this message to
R
Answer the following questions based on the above his friends also. reBSE DISC]
information: (a) What qualities/values, in your opinion did
(a) What were thevalues displayed by Abhishek? Rohit possess ?
(b) A low voltage supply from which one needs (b) Why CFLs and LEDs are better than traditional
high current must have very high internal incandescent lamps?
SI
resistance. Why ? (c) In what way earthing reduces electricity bill ?
4. Ameen had been getting huge electricity' bill for the
. : past few months. He was upset about this. One day
Answers
1. (a) Helpful, aware of his limits, ability to take quick 3. (a) Empathy, dutifulness, determination, responsi-
IT
decisions. bility and compassion.
(b) Net emf, e = 110-12 = 98V (b) The maximum current that can be drawn from
If R is the series resistor, then the charging current a voltage supply is given by
will be E
Imax
r
I = _E_ = ~ A = 5 A (given) :. R = 19.1 n Clearly, Imax will be large if r is small.
H
R+ r R+ 0.5
Terminal p.d. of the battery during charging, 4. (a) Helpfulness, co-operative attitude and scientific
temperament.
V = E + Ir = 12 + 5 x 0.5 = 14.5 V
(b) CFLs and LEDs have following advantages:
If the series resistor R were not included in
(i) Low operational voltage and less power
O
R
GLIMPSES
SI
1. Current electricity. The study of electric charges round the complete circuit. It is equal to the
in motion is called current electricity. terminal p.d. measured in open circuit.
2. Electric current. The flow of electric charges EMF = Work done or e= W.
through a conductor constitutes electric current. Charge q
Quantitatively, electric current across an area
held perpendicular to the direction of flow of 6. SI unit of emf is volt. If an electrochemical cell
charge is defined as the amount of charge supplies energy of 1joule for the flow of 1coulomb
flowing across that area per unit time. of charge through the whole circuit (including
IT
the cell), then its emf is said to be one volt.
For a steady flow of charge, I =Q
t 7. Ohm's law. The current flowing through a con-
If the rate of flow of charge varies with time, ductor is directly proportional to the potential
then difference across its ends, provided the tem-
1= lim ~Q = dQ perature and other physical conditions remain
M~O M di . unchanged.
H
SI unit of current is ampere (A).
v«I or v = RI or V=R
1 coulomb I
1 ampere = or
1 second Here R is called the resistance of the conductor.
8. Resistance. It is the property by virtue of which
O
10. Resistivity or specific resistance. It is the 15. Colour code for carbon resistors.
resistance offered by a unit cube of the material
Colour Number Multiplier
of a conductor.
Black 0 10°
RA
p==- Brown 1 HY
I
Red 2 102
SI unit of p == nm Orange 3 103
It depends on the nature of the material of the Yellow 4 104
conductor and the physical conditions like Green 5 105
temperature, pressure, etc. Blue 6 106
R
11. Current density. It is the amount of charge Violet 7 107
flowing per second per unit area normal to the Grey 8 108
flow of charge. It is a vector quantity having the White 9 109
same direction as that of the motion of the
How to remember colour code:
SI
positive charge.
B B R 0 Y of Great Britain had Very Good Wife
For normal flow of charge, -I. -I. -I. -I. -I. -I. -I. -I.' -I. -I.
. q/ t I
o 1 2 3 4 5 6 7 8 9
t=rr=>: Tolerence:
A A
-> ->
In general, I == jA cas e == J. A Gold
±5%
Silver
± 10%
No colour
± 20%
SI unit of current density ==Am -2. A set of coloured co-axial bands is printed on
IT
the resistor which reveals the following facts:
12. Conductance. It is the reciprocal of resistance.
(1) The first band indicates the first significant
1
Conductance figure.
Resistance
(2) The second band indicates the second
significant figure.
H
or G==~
R (3) The third band indicates the power of ten
with which the above two significant figures
SI unit of conductance == ohm -1 == mho
must be multiplied to get the resistance
== siemen (S).
value in ohms.
O
13. Conductivity or specific conductance. It is the (4) The last band indicates the tolerence in per
reciprocal of resistivity. cent of the indicated value.
R
--> For a semiconductor,
field E, Ohm's law may be expressed as
I=eAE(nlle+ Pllh)
--> --> --> -->
j = (J E or E =p j and (J = e (n Ile + P Ilh )
where n and P are the electron and hole
SI
--> -->
The equation E = P j leads to another state- densities.
ment of Ohm's law i.e., a conducting material 23. Ohmic conductors. The conductors which obey
obeys Ohm's law when the resistivity of the Ohm's law are called Ohmic conductors. For
material does not depend on the magnitude and these conductors, V-I graph is a straight line
direction of the applied electric field. passing through the origin. For example, a
19. Temperature coefficient of resistance (a). It is metallic conductor for small currents is an
defined as the change in resistance per unit Ohmic conductor.
IT
original rcsi.tance per degree rise m 24. Non-ohmic conductors. The conductors which
temperature. It is given by do not obey Ohm's law are called non-ohmic
a =IS. - Rl conductors. The non-ohmic situations may be of
s, (t2 - t1) the following types:
(i) The straight line V-I graph does not pass
If tl =ooe and t2 = t=C, then
H
through the origin.
Rt -Ro (ii) V-I relationship is non-linear.
a = or R/ = Ra (1+ at)
Ro x t
(iii) V-I relationship depends on the sign of V
The unit of a is oe-1 or rc-I. for the same absolute value of V.
O
20. Effect of temperature on resistance. For metals a (iv) V-I relationship is non-unique.
is positive i.e., resistance of metals increases Examples of non-ohmic conductors are water
with the increase in temperature. voltameter, thyristor, a p-n junction, etc.
For semiconductors and insulators, a is negative 25. Superconductivity. The phenomenon of
i.e., their resistance decreases with the increase complete loss of resistivity by certain metals
M
27 ... Resistances in series. When a number of 32. Cells in series. If, n cells of emf ~d internale
resistances are connected in series, their resistance r each are connected in series, then
equivalent resistance (Rs) is equal to the sum of current drawn through external resistance R is
the individual resistances.
I=~
Rs =Rl +~ +~ +... R + nr
28. Resistances in parallel. When a number of
33. Cells in parallel. If m cells are connected in
resistances are connected in parallel, the
parallel, then current drawn through external
reciprocal of their equivalent resistance (Rp) is
resistance R is
equal to the sum of the reciprocals of the
individual resistances. I=~
R
111 1 mR+r
-=-+-+-+ ...
Rp Rl ~ ~ 34. Cells in mixed grouping. If n cells are connected
For two resistances in parallel, in series in each row and m such rows are
connected in parallel, then current drawn
R = Rl~
SI
through an external resistance R is
P Rl + ~
mne
29. Division of current in resistors joined in 1=---
mR+ nr
parallel. The current is divided in resistors,
connected in parallel, in the inverse ratio of For maximum current, the external resistance
their resistances. must be equal to the total internal resistance, i.e.,
I = ~ . I R= I1r
1 Rl + ~ m
IT
or mR = nr.
1= Rl .1
2 Rl + ~ 35. Heating effect of current. The phenomenon of
the production of heat in a resistor by the flow
30. Internal resistance (r). The resistance offered by
of an electric current through it is called heating·
the electrolyte of a cell to the flow of current
effect of current or Joule heating. It is an
between its electrodes is called internal
H
irreversible process.
resistance of the cell. It depends on (i) nature of
the electrolyte, (ii) concentration of the 36. Joule's law of heating. It states that the amount
electrolyte, (iii) distance between the electrodes, of heat H produced in a resistor is
(iv) common area of the electrodes dipped in the (i) directly proportional to the square of
O
37. Electric power. It is the rate at which an electric power is equal to the sum of the reciprocals of
appliance converts electric energy into other the individual powers of the appliances which
forms of energy. Or, it is the rate at which work is have been manufactured for working on the
done by a source of emf in maintaining an same voltage.
electric current through a circuit.
1 1 1 1
Electric power, -=-+-+-+ .
PP1 P2 P3
P=-
W = VI = [2 R = _V2
43. Power consumed by a parallel combinat on of
t R
appliances. The effective power is equal to the
38. 51 unit of power is watt. The power of an sum of the powers of the individual appliances.
R
appliance is one watt if one ampere of current
flows through it on applying a potential P=P1+P2+P3+······
difference of 1 volt across it. 44. Efficiency of a source of emf. It is the ratio of the
1 joule output power to the input power. If a source of
1 watt=~'---
e
emf and internal resistance r is connected to an
SI
1 second
external resistance, then its efficiency will be
= 1 volt x 1 ampere
Output power
or 1 W =1 Js-1 =1 VA 11=
Input power
1 kilowatt (kW) = 1000 W.
VI V R
39. Electric energy. It is the total work done in e[ I R +r
maintaining an electric current in an electric
45. Maximum Power Thea ·em. It states that the
IT
circuit for a given time.
output power of a source of emf is maximum
Electric energy = Electric power x time when the external resistance in the circuit is
W =Pt equal to internal resistance of the circuit i.e.,
= VI t joule = [2 Rt joule when R =r.
e2
H
40. Units of electric energy. The commercial unit of
4r
electric energy is kilowatt-hour (kWh) or Board
The efficiency of a source of emf is 50% when it
of Trade (RO.T.) unit. It is the electric energy
delivers maximum power.
consumed by an appliance of power 1000 watt
in one hour. 46. Efficiency of an electric device. It is the ratio of
O
2
V 2 The power output of an electric motor is
P=-=[ R = VI.
R maximum when its back emf is one-half the
source emf, provided the resistance of the
42. Power consumed by a series combiriation of
windings of the motor is negligible.
appliances. The reciprocal of the effective
3.172 PHYSICS-XII
47. Kirchhoff's laws. These laws enable us to (i) To compare the emfs of two cells. If 11 and 12
determine the currents and voltages in different are the balancing lengths of the potentiometer
parts of the electrical circuits. e e
wire for the cells of emfs 1 and 2 respectively,
First law or junction rule. In an electric circuit, then
the algebraic sum of currents at any junction is
zero. Or, the sum of currents entering a junction
is equal to the sum of the currents leaving that
junction. (ii) Tofind the internal resistance r of a cell. If 11
Mathematically, is the balancing length of the potentiometer
R
wire without shunt and 12 the balancing length
LI=O
with shunt R across the cell, then internal
Justification. This law is based on the law of resistance of the cell will be
conservation of charge. When the currents in a
r=--x
e-v 1-/
R=_I__ 2 x R
circuit are steady, charges cannot accumulate or
SI
V 12
originate at any point of the circuit.
50. Wheatstone bridge. It is an arrangement of four
Second law or loop rule. Around any loop of a
resistances P, Q, R and S joined to form a
network, the sum of changes in potential must
quadrilateral ABCD with a battery between A
be zero. Or, the algebraic of the emfs in any loop
and C and a sensitive galvanometer between B
of a circuit is equal to the sum of the products of
and D. The resistances are so adjusted that no
currents and resistances in it.
current flows through the galvanometer. The
IT
bridge is then said to be balanced. In the
balanced condition,
or
P R
Justification. This law is based on the law of
Q S
conservation of energy. As the electrostatic
force is a conservative force, the total work done Knowing any three resistances, the fourth
H
by it along any closed path must be zero. resistance can be computed. A wheatstone
bridge is most sensitive when the resistances in
48. Gilvanometer. It is a sensitive device to detect its four arms are of the same order.
current in a circuit. It produces a deflection
proportional to the electric current flowing 51. Slide wire bridge or metre bridge. It is an
O
Vex: I
or S =( 1001-I J R
or V=kl
SA
where k is the potential drop per unit length Resistivity, p=-
I
which is called potential gradient. Poten-
S x nr2
tiometer has two main uses.